You are on page 1of 210

TPO 1

TPO1-

4
TOPIC 4 A mandatory policy take the option.

The shortened workweek would increase company

1. Force companies to spend more;


+ workers = putting more money into providing training and
medical benefits;
More office space & more computers;
Additional costs would cut into company profits.

profits.

For the country, the unemployment rates will be


reduced.

The option of a four-day workweek would be better for


individual employees.

2. Four-day workweek more jobs;


Hire new workers = more cost;
other ways: work overtimeraise expectations;
additional jobs & unpleasant current jobs.

3. Risks: reduce their quality of life;


A shorter workweek stability & promotion chances;
For continuous coverage and consistent supervision,
five-day employees are more likely to have the
management positions.

Sample Answer
The lecture completely refutes the reading which presents three benefits of a mandatory policy offering employees the
option of working a four-day workweek for four-fifths of their normal pay. In the lecture, the professor believes that this
policy wont provide the benefits in the ways the reading suggests.
To begin with, the author in the reading claims that since the employees would feel more rested and alert, they would
concentrate more on work, increasing the company profits in the end, whereas the professor counters that argument by
asserting that offering the option of a four-day workweek might cost a lot more, for companies need to spend more
money in training and medical benefits. More employees also require more office space and more computers. These costs
would cut into profits.
Besides, in contrast to the reading, which claims that this policy would reduce the unemployment rates, the lecturer insists
that instead of hiring new workers, which costs a lot, companies have other options such as asking their employees to

work overtime. What is worse, companies might raise their expectations. In this way, there wouldnt be any additional jobs,
and even the current jobs become more unpleasant.
Finally, its true that employees would invest more free time in their personal lives by working four days a week, as
mentioned in the reading. However, the professor claims that they also need to take the risks of reducing their quality of
life. A four-day workweek could decrease employees job stability and harm their chances for advancing their careers.
What is more, workers who work four days a week are more likely to lose their work. They may also lose the chances of
promotion for companies prefer five-day workers to do the management work in order to ensure continuous coverage and
consistent supervision.

Level 2 Level1-Level5
TPO

TPO1-

Do you agree or disagree with the following statement?


At universities and colleges, sports and social activities are just as important as classes and libraries and should receive
equal financial support.
Use specific reasons and examples to support your answer.

2 Agree social activities


1
2
3 Disagree library knowledge

1
2


:
Background informationUniversities: Prepare for the future comprehensive ability; However, academic
knowledge: heart of education
The benefits of social activities & sports: collaboration skills; communication skills
1It is students, rather than schools, who could and should raise most funds for their sports or social activities. +

2With more financial support, the quality of instruction could be guaranteed. +

Sample Answer
Nowadays one rarely attains colleges or universities merely for educational purpose. Students expect to have exciting
experience outside the lecture hall and libraries: clubs, internships, or various forms of sports. The importance of these
activities has been highlighted, even drawn to the same level as that of study itself. Consequently, some student unions
contend that sports and social activities should receive equal financial support as classes and libraries. However, I disagree
with such radical notion.
Admittedly, the role of educational institutions has changed dramatically. Back in the nineteenth century, colleges and
universities generally operated for pure educational purposes. These days, however, in response to the developments of
the society, the functions of colleges have been diversified: expanding students social network, adjusting them to the
ever-changing world, and teaching academic knowledge. We expected graduates to be well-prepared, not only
academically, but socially. Skills for collaboration and communication are as critical as knowledge of physics, algorithm,
chemistry, etc. Meanwhile, one can learn the first from sports and social activities on campus. Consequently, these
activities do equal classes in essentiality.
However, this is not the end of the story. They may receive funds and supports, but not as much as those for classes and
libraries. It is students, rather than schools, who could and should raise most funds for their sports or social activities,
which cultivate students sense of independence and social skills, while, from my own perspective, the most useful amid is
the skill to persuade others to do us a favor, in most cases, provide financial support. Raising fund is a fantastic opportunity
to learn or polish such skill, which will be handy after those very students graduate and start to run funds for their own
projects or campaigns. As a consequence, limited support from school is exactly the momentum for students to improve
themselves.
Furthermore, classes and libraries needs more support than sports and social activities do. While students are able to
tackle their own difficulties, classes and libraries are not. One cannot expect the professors and faculties to scrimp their
supple for the family and use their wages and premiums to purchase new facilities for laboratories while still teaching
lessons of good quality; one cannot expect the lecturers to act as lobbyists to obtain financial support for their classes; one
cannot expect, either, the fate of a new library, which would satisfy the need of more than three thousand students, to be
totally held in the fist of some tight-fist entrepreneurs, who may require the logo of their companies being carved on the
libraries. For the sake of education, the support for classes and libraries is ineligible.
To sum up, sports and social activities may as crucial as classes and libraries, but they are not supposed to receive equal
financial support.


2012/04/22 It is more important for universities to spend money for universities to spend money in improving facilities
than in hiring famous teachers.

2012/09/07 NA Schools should give more money to students social activities or to improve their dining.
2013/10/27 Student activities do as much benefit as academic study.

Level 3 Level1-Level5

TPO 1
TPO1-

TOPIC

1.employees would feel more rested and alertfewer


costly errorsincrease company profits

1.force companies to spend moreadd new workers


training and medical benefitsmore office space and more
computers

2. reduce unemployment rates

2. hiring new workers is costly company ask their


employees to work overtime to make up the difference.
Or company raises expectationno additional jobs

3. better for individual employeesmore free time


improve the quality of their livesspend time with
family, pursuing private interests, enjoy leisure activities

3. shorter week decrease employees job stability and harm


their chances for advancing their careerscompany prefer
five-day employees

Sample Answer
In the reading passage, the writer claims that the advantages of a four-day week. However, the speaker denies the
idea made in the reading that even though there might be some advantages for four-day week but there will be more
disadvantages for it and it is really unrealistic.
First of all, the reading passage contends that the shortened workweek would make employees feel more rested and
alert. In this case, employees would make fewer costly errors in their work and companies would have fewer overworked
and error-prone employees for the same money and companies can hire more new employees to do the same work but
the lecturer argues that a four-day week will force companies to spend more since adding new workers means putting
much more money into providing training and medical benefits. Whats more, more workers mean more office space and
more computers which will cost a lot.
Second, the author states that the primary benefit for the country from the four-day work is reducing unemployment
rates since many full-time employees started working fewer hours and their workload would have to be shifted to others.
The speaker says, however, that hiring new workers is costly and as a result the companies might choose to ask their
employees to work overtime to make up the difference and no additional jobs will be created.
Lastly, in the reading passage, the author believes that the four-day workweek would be better for individual
employees and they would have more free time to improve the quality of their lives like spending time with families,
pursuing private interests and enjoying leisure activities. On the other hand, the professor in the listening argues that
shorter week decrease employees job stability and harm their chances for advancing their careers. Companies prefer
five-day employees in management positions.
In conclusion, four-day workweek is still in consideration and we should see this problem objectively.
.

The reading passage contends that but the lecturer argues that.

According to the professor in the lecture,.This directly contradicts what the reading passage indicates, because.

Level1-Level5
Level 3

TPO1-

Do you agree or disagree with the following statement?


At universities and colleges, sports and social activities are just as important as classes and libraries and should receive
equal financial support.
Use specific reasons and examples to support your answer.

agree/disagree

1agree/disagree
2
Sample

support

support

Sample Answer
Should sports and social activities be just as important as classes and libraries receive equal financial support at
universities and colleges? Although I agree with the speaker's broad statement on the basis that money is supposed to be
put into classes and libraries. Nevertheless, sports and social activities make a complete campus life together with classes
and libraries. So whether sports and social activities are just as important as classes and libraries should receive equal
financial support can be a big problem for students and university.
I concede that education is an essential part of life and educational budgets should be devoted to classes and libraries.
Since the reason why students go to the universities and colleges is that they want to get the knowledge from the class and
read more books from the libraries to enrich their knowledge. If the university and colleges provide the financial support
to the sports and social activities, the financial support for the classes and libraries will decrease inevitably or we can say
there will not be more money for classes and libraries. In this case, the quality of class and libraries will definitely decrease
without enough financial support. Think about this, a solider would think it is his duty to obey the rules without any
hesitation; a surgeon would consider healing patients as his accountability; and a historian would think it is his

responsibility to discover that is behind the historical description. So it is the same with the situation that university and
colleges should put more money into classes and libraries.
However, when we step in the role of the students, playing sports makes a person healthy and energetic to face the
future challenges. As the pressure for the students increase, they may suffer from eye problems or other physical diseases.
Therefore, doing exercises can provide them the opportunities to have a rest and release the pressure. We should stand in
student place to see this problem. For example, when we use base-three number system, two plus two equals 11; a child
learning to write numerals might insist that two and two makes 22. A biologist might joke that two plus two might add up
to a whole lot more than 4 when we're talking about the reproductive life of habits. What's more, social activities can
help foster abilities that students cannot gain during classes or self-studying. They can make more friends with that ability.
Based on the statements above, we can say that sports and social activities are as important as classes and libraries.
So we should provide a better idea to balance these two sides. The university and colleges can provide more money for
sports and social activities when they do not decrease the money for classes and libraries. Nature severs as a good
example of how trial and error can be used for changes. Every now and then genetic mutations occurs errors in gene
reproduction. Most of the time, these mutations have a deleterious effect on the species, and they drop out of the gene
pool. But occasionally, a mutation provides the species with something beneficial, and that change will be passed on to the
future generation.

2014/4/12 Do you agree or disagree: the universities should spend more money in improving facilities (libraries,
computer labs) than hiring famous teachers.
2014/5/24
Do you agree or disagree with the following statement: it is important for government to provide money for
things that are beautiful not just for things that are practical.

Level1-Level5
Level 4

TPO 2
TPO2-

TOPIC

1.

1.

2.

2.

Sample Answer
In the lecture, the professor is skeptical about the idea of the reading passage that teamwork offers a wealth of advantages,
and as a result, it becomes the best way to solve any companys problem. The professor, however, argues that companies
would not largely benefit from teamwork as it is supposed to be based on a six-month observation of a company.
First, in the reading passage, it is pointed out that teamwork provides creative solutions to any problem due to the
extensive knowledge, expertise, and skills that team members possess as a collectivity. On the contrary, the professor
maintains that some team members easily got some free ride, and they did not contribute to the project. Furthermore,
each team member received the recognition for a well-performed job, and the real contributors of the team would not be
particularly rewarded for their insight.
Second, the professor then challenges the readings idea that team members would feel motivated if their voices are heard
and contributions recognized, leading to a more far-reaching influence for the group as well as a greater impact on the
individual. He suggests that it became difficult to reach consensus among team members in a group; thus, the project
could not be easily moved along. Also, the influential member in the group was likely to dominate the decision-making
process, but he/she would not be responsible for the aftermath of any decision that results in failure.

Level 3 Level1-Level5

TPO2-


Always telling the truth is the most important consideration in any relationship between people.

people people
lovers, parents and children, patients and doctors
white lies

Sample Answer
Honesty is one of the most valuable virtues that all cultures cherish. There is no debate that one should be always telling
the truth whenever possible. However, there are some exceptions when white lies strengthen the relationships between
lovers, encourage children when they meet with difficulties, and help patients recover from their diseases.
First and foremost, in order to maintain a close relationship between lovers, it is necessary to tell white lies in some
occasions. For example, a wife does not like the new pair of shoes, as her present from her husband. However, she tries
them on and keeps complimenting on how she looks great by wearing them. The purpose of her behavior of not telling
the truth is that she does not want to frustrate her husband, and she really appreciates the love that she has received
through her husbands present. In other words, white lies between lovers can sometimes strengthen their relationships In
other words, white lies between lovers can sometimes strengthen their relationships and enhance their intimacy by
showing each other their unconditional love in the first place.
Second, there is much evidence to suggest that white lies are acceptable between parents and children under some special
circumstances. Parents intend to encourage their children when they receive low grades in a math examination,
comforting them that they have already worked so hard and what they lack is just a bit of luck. In this case, parents
attempt to avoid hurting their childrens feelings and passion towards learning by hiding the truth, and moreover, they
promise them a bright future with constant encouragement and unwavering support.
Finally, white lies are helpful in building patients confidence when they fight against diseases, such as cancer and AIDS.
Patients might get easily desperate if they know the severity of their illness, and it becomes rather difficult for them to
make a full recovery. Instead, doctors always ensure their patients that with the rapid medical advancement, no disease
would be undefeatable. Accordingly, they should be more optimistic and confident in getting over their diseases.
To conclude, it might be true that the practice of telling the truth is common in every culture. However, it turns out to be
rather arbitrary to conclude that telling the truth is the most important consideration in any relationship between people
with the exceptions shown above in the essay.

2013 7 21
It is impossible to be completely honest with your friends all the time.
2014 12 21
If you are selecting a leader for a student organization, honesty is the most important to consider in deciding whom to vote
for.

Level 4 Level1-Level5

TPO 3
TPO 3-

TOPICwhether the Portrait of an Elderly Woman is a work by Rembrandt

1. inconsistency in the way the woman was dressed

1. the fur collar on the woman was painted over the


original work, to increase the portrait's value

2. the light and shadow were not consistent

2. after the fur collar was removed, it turned out the


woman in the original painting was wearing a light colored
cloth collar which reflected light to illuminate part of her
face

3. the pieces of wood glued together;

3. the wood panel was later enlarged to make it more grand


and valuable; the wood is of the same tree used in other
Rembrandt paintings

Sample Answer -1
While admitting everything the passage said is true about a famous painting, Rembrandt's Portrait of an Elderly Woman in
a White Bonnet, the lecture cites additional evidence to argue against the passage's conclusion that the painting is not
Rembrandt's original work.
First, the passage points out an inconsistency in the way the woman was dressed and argues that's not Rembrandt's style.
The lecture takes its time to explain that after an X-ray examination of the pigments, scientists found out the fur collar on
the woman was painted over the original work 100 years after it was made. People did this probably with a motivation to
increase the portrait's value by making it look like a formal portrait of an aristocratic lady. Therefore, the inconsistency of
dressing style did not exist in Rembrandt's original work, neither did the inconsistency of light and shadow as discussed in
the following paragraph.
The lecture points out after the fur collar was removed, it turned out the woman in the original painting was wearing a
light colored cloth collar which reflected light to illuminate part of her face, leaving part of it in dark. So in the original
painting the light and shadow were consistent, not amateurish as the passage tried to conclude based on incomplete
research.
Last, the passage tells about the pieces of wood glued together on which the portrait was made, and asserts that
Rembrandt was not known for doing so in his art work. The lecture again successfully overturned this argument by
introducing additional evidence, that the wood pieces were glued to the sides and top of the original one-piece wood
panel at a later time, probably to increase the portrait's value yet again. Not only the original painting was made on a
one-piece wood panel, but also the wood came from the same tree as that of Rembrandt's another famous painting, his

self portrait with a hat. The lecture firmly establishes the authenticity of this painting.

Sample Answer -2
The lecture revises the idea presented in the text, that Rembrandt was not the artist who painted the famous painting
Portrait of an Elderly Woman in a White Bonnet.
The inconsistency between the white cap, which identifies the woman as a servant, and the expensive fur collar she wears
dissolves as the Professor explains that the fur collar was apparently painted over the original painting to increase its
worth by displaying an aristocratic woman.
In addition, the assumption that light and shadow in the painting do not fit together is refuted by the fact that in the
original painting, the woman wears a light cloth that illuminated her face. Thus the presentation of light and shadow was
indeed very realistic and accurate, as it is characteristic of Rembrandts paintings.
Finally, the mystery of the panel consisting of patches glued together is also solved in the lecture. Actually, the wood panel
was later enlarged to make it more grand and valuable, but the original painting was painted on a single panel, as
Rembrandt would have done it. Furthermore, the wood is of the same tree used in other Rembrandt paintings, like the
Self-Portrait with a Hat. All this information points to Rembrandt as the painter of the controversial painting.

Level 4 Level1-Level5

TPO3-

It is more important to keep your old friends than it is to make new friends.

1. old friends have longer and closer relationship with us.


2. old friends can provide us with more valuable advice.

Sample Answer
According to an Internet survey, almost 70% of people think that it is more important to keep old friends than it is to make
new friends. I totally disagree with this idea. In my view, the importance of keeping old friends is equal to the importance
of making new friends. I have several reasons for this opinion.
The importance of old friends in reducing stress cannot be exaggerated. Older friends have longer relationships with us;

therefore, they know us better than new friends. When we encounter obstacles, it is our long-term friendsour old
friendswho give us the best suggestions and help us get through our problems. Our new friends sometimes cannot give
adequate advice to us because they do not know our situations well enough. We need time to get to know our friends.
Last week, I faced the dilemma of whether to accept a promising job offer that would take me to another city. All of my
new friends in my current company encouraged me to take the offer because they thought the job had strong future
prospects. However, one of my older friends discouraged me from accepting it, as he knew that my mothers health
condition was poor and that she needed someone to take care of her. I think his suggestion was the best suited to me.
In other situations, old friends simply cannot help, and we need new friends to lend a hand. New friends can mean a lot in
emergencies. In an accident, for instance, an old friend who is far away from you cannot offer any help. In such situations,
new friends often play a crucial role in helping us overcome difficulties. After my cousin was in a car accident, his friends at
his new company gave him a lot of help.Without the help of his new friends, I doubt that he would have recovered as well
as he did.
I often wonder why some people emphasize the importance of having one kind friend, especially an old friend. Maybe
these people are satisfied with the idea that old friends are more important. Ultimately, I think we should develop all kinds
of friendships in order to assure that we can get help whenever we have trouble.

2006/08/12
2007/08/17
age.

Advices from friends who are older than you is more valuable than those you get from friends of the same

2014/08/3 The ability to maintain a small number of friends for a long time is more important to happiness than the
ability to make new friends easily

Level 3 Level1-Level5

TPO 4
TPO 4-

TOPICwhether dinosaurs were endotherms

1. the dinosaur fossils were found in cold Polar Regions


where only endotherms can survive

1. polar region was warmer than it is today, even when cold


whether came, dinosaurs could migrate to warmer area or
just hibernate.

2. the underneath leg position of dinosaurs as a physical


character of endothermy

2. the dinosaurs' underneath leg feature was not for


running, it was to support weight instead, so dinosaurs
could grow into a very large size.

3. Haversian canals in dinosaur's bones which suggests a


rapid growth, another character of endotherms

3. dinosaurs also has growth rings. These growth rings


indicate when it is cold they did not grow or grow slowly.
This periodic pattern of growth shows that they are not
endotherms, because endotherms' growth is not affect by
cold weather.

Sample Answer -1
The topic is about whether dinosaurs were endotherms. The reading article offers three discoveries to assure that
dinosaurs were endotherms. But the listening contents highly doubt about that.
The first proof from the reading is that the dinosaur fossils were found in cold Polar Regions where only endotherms can
survive. But the woman from listening reveals the fact that Polar region was warmer than it is today, so that it was not
difficult for reptiles to live there. And even when cold whether came, dinosaurs could migrate to warmer area or just
hibernate. As a result, the saying of cold Polar is not convincing.
The second evidence is the underneath leg position of dinosaurs as a physical character of endothermy, and the passage
sets running as an example. However, the listening claimes that the dinosaurs' underneath leg feature was not for running,
it was to support weight instead, so dinosaurs could grow into a very large size. That is to say, the leg position theory is not
good enough to prove that dinosaurs were endotherms.
The article's final evidence is Haversian canals in dinosaur's bones which suggests a rapid growth, and it is allegedly
another character of endotherms. However, the listening points out that dinosaurs also has growth rings. These growth
rings indicate when it is cold they did not grow or grow slowly. This periodic pattern of growth shows that they are not
endotherms, because endotherms' growth is not affect by cold weather. In a word, the third evidence in the reading is
again overturned by the reading.

Sample Answer -2
The professor actually contradicts the statements made in the passage. She is of the view that dinosaurs are not
endotherms i.e. they were not able to keep their body temperature at a constant Rate.
The professor contradicts the issue of dinosaurs being endothers based on the availability of fossils being available in thwe
polar regions, she say that the polar regions in those days were not as cold as they are today i.e at least warm enough for
dinosaurs to live. Durin harsh winters she says that there is a possibility of the dinosaurs actually migrating to warmer
regions.
The issue of leg position and movement being used as a reason to clasify the dinosaurs as endotherms does not please the
professor either. She says that dinosaurs had legs under their bodies to support their huge bodies i.e the legs under the
body of the dinosaur were actually to support the huge weight of the dinosaur and not to provide it with a body structure
like endotherms(which is actually suited for running).
The professor acknowledges the presence of haversian canals but also points out that that the fossils show the presence of
growth rings. These rings occur due to the thickening of the bone. The thickening indicates that the dinosaurs werent
actually growing continuously but were experiencing periods of rapid growth and periods of no growth in succesion. This
pattern. she says is characteristic of non endothermic aanimals.
Thus it can be inferred that the professor challenges the passage by giving reasons as to why she thinks that the dinosaur is
not an endotherm.

Level 4 Level1-Level5

TPO4-

In twenty years there will be fewer cars in use than there are today.

1. there will be other types of transportation tools invented in our society.


2. government will enact some laws and regulations to limit the number of cars.
3. people in twenty years will focus more on environmental protection

Sample Answer-1
Currently, there is a widespread belief that the number of cars will continue to increase in next few years. From my

perspective, however, there will be fewer cars in use than there are today in twenty years.
To begin with, there will be other types of transportation invented in our society. With the development of science and
technology, a great variety of transportation tools have been invented and people can take them to go everywhere instead
of cars, which maybe is more convenient to them. For example, in contemporary society, more people choose to go out by
the subway, which is more convenient and most of all, much faster, because people never worry about being caught in a
traffic jam just as cars always do. As a result, I strongly believe that the cars will be obsoleted.
What is more, the Internet has greatly changed peoples lifestyle. Now the Internet has become more and more popular
and people always purchase goods on the Internet, such as eBay, instead of driving to the supermarket. With a simple click
on the computer, what you want will be sent to your house. When it comes to in twenty years, people are able to do more
things at home with the Internet, like dealing with the working, talking to people and managing business and so on. People
will be used to the internet-related operations in every aspect in twenty years, since they feel it's easy and comfortable to
use the Internet, and it makes the life simple and free. Therefore, cars are not indispensable in our daily life.
Last but not least, people in twenty years will focus more on environmental protection, so the usage of cars will be
decreased. As is known to all, more cars on the street will produce more air pollution to the environment where we live. In
order to keep people healthy, and meanwhile, to make the environment clean, there will be fewer cars in use than there
are today. The people living today have already call for a low-carbon life, I believe twenty years later, more people will join
in such a low-carbon activity.
Everything will change in many years in the world, including the transportation of the daily life. In a word, with the
combination of my reasons, I agree with the statement that there will fewer cars in use in twenty years than there are
today.

Sample Answer-1
At first I would say I disagree with the statement according to which the use of cars would decrease in the following twenty
years.
Indeed, the worlds population is expected to grow drastically in the next decades. Population in Western countries is likely
to grow at the same pace it is growing today, but the most important contribution to the global population growth will
certainly come from developing countries. And to my opinion, a growth in the use of cars will accompany this growth in
population. People in developed countries are largely accustomed to using cars, therefore a great change in habits would
be needed to curb the growth in cars use. Moreover, studies show that people live further and further from their work
place, and therefore have to use their car more often. As for developing countries, a rapid economic growth is expected to
go along with population growth in a good deal of countries. These countries are likely to reach the development level of
Western countries within the next couple of decades. Unless these countries can develop their own pattern,which I doubt,
they will use more and more cars.
Nevertheless, some hope remains that the growth in the use of cars can be slowed in the future. People and governments
are becoming more and more aware of environmental issues linked to the use of cars, and some experiments aiming at
reducing it have been conducted in some places (e.g. the toll in the London downtown, or the alternate circulation in Paris
on high pollution rate days). Although these measures certainly have a positive effect on pollution and traffic. I dont think
theyre viable in the long run. To curb environmental problems linked to cars, the only solution is to develop new clean
technologies. This way pollution rates would drop, but we would still use more and more cars. So as for traffic in big cities

in twenty years,Im very pessimistic!

2006/10/08

Level 3 Level1-Level5

TPO 5
TPO5-

TOPIChow Chaco great houses were used

1. residential purpose

1. there should be many fire places left, since there were


more than one hundred of families there, but the reality is
that only few fire places were found.

2. storage use for food supplies

2. excavations show no trace of grains or maize containers.


If the 'great houses' were used to store crops, there should
be at least some spills of crops or remains of containers.

3. ceremonial center because of broken pots found in


deposits

3. the mound contains a lot of other materials besides


broken pots, such as building materials of sand, stones or
construction tools, and those were pretty much just trash
heap of construction materials.

Sample Answer-1
The topic is about what 'great houses' were used for. From the passage, there are three different guesses. However, the
speaker does not think those are convincing, and he illustrates his three arguments in response to other threes in the
article respectively.
First, in order to challenge the residential purpose demonstrated by the reading, the listening identifies that there should
be many fire places left, since there were more than one hundred of families there, but the reality is that only few fire
places were found. In a word, residential use is not plausible.
Furthermore, in the passage storage use for food supplies is considered to be another explanation of Chaco structures. On
the contrary, the lecturer in the listening says this theory is unsupported, because excavations show no trace of grains or
maize containers. If the 'great houses' were used to store crops, there should be at least some spills of crops or remains of
containers. Thus, the houses should not have been used to store foods.
Finally, the listening rebuts the reading's idea of ceremonial center because of broken pots found in deposits, by stating
that the mound contains a lot of other materials besides broken pots, such as building materials of sand, stones or
construction tools, and those were pretty much just trash heap of construction materials. In other words, broken pots are
not enough to prove that 'great houses' were built for ceremonies.

Sample Answer-2
The author of the reading passage proposes three theories as likely explanations of the primary function of Chaco Canyon
houses, giant structures built in the 12th century. The lecturer, however, points to the inaccuracies in each of these
theories.
The lecturer argues that the modest number of fireplaces in these structures is in contradiction with the huge size of these
houses, indicating that these structures could not have been used for residential purposes. The reading, however, draws
comparisons between the Chaco houses and other similar large residential structures in support of the residential
theory.
The second theory, that the houses were used for food storage, is also rejected by the lecturer. He explains that a place
that had been used for storing maize would have many traces of scattered maize, which is not the case in the area of the
Chaco Canyon houses. This proves that the food storage theory is unlikely.
Finally, regarding the third theory, the ceremony theory, the lecturer contends that the presence of broken pots close to
the great houses does not offer sufficient proof that this was a place for ceremonial activities. He argues that there are
other materials such as pieces of construction trash found along with the broken pots, which suggest that these pots were
probably not used for ceremonial purposes but instead were discarded by construction workers upon completion of the
great houses.

Level 3 Level1-Level5
TPO5-

People today spend too much time on personal enjoyment-doing things they like to do-rather than doing things they
should do

//

1. spending much time on personal enjoyment does not indicate that people ignore their accountability which should be
taken.
2. to pursue one's personal enjoyment help to assume the responsibility. People should not be blamed for their time spent
on things they like.

Sample Answer
The above statement poses a hot topic on contradiction of people's personal enjoyments and responsibilities. And the
speaker tends to think people now are dedicating more on personal likeness than on what they should do. I strongly
disagree with this opinion because I think that to pursue one's individual enjoyment does not contradict to assume the
responsibility.
Admittedly, people nowadays increasingly enjoy freedom to pursue personal enjoyment than before. As the development
of democracy, the ideas and behaviors of individual are respected and encouraged to some extent. For instance, students
can choose any major which they favor mostly; graduates can participate in whatever professions they are interested in;
citizens possess absolute freedom on free press and behaviors they like. Briefly, people have much time and freedom to do
their favorite things.
However, spending much time on personal enjoyment does not indicate that people ignore their accountability which
should be taken. Let's take a college student in art for example. Generally, students in art major should spend much time
on reading and appreciating art history, biography of artists and art works. Yet, the student favors more on economics. He
probably continues to earn his degree on art with great interest in economic as well. Or he may pursue a second degree on
economics. No matter what his decision is, he takes his responsibility to become a worthy student of his major.
In addition, to pursue one's personal enjoyment help to assume the responsibility. People should not be blamed for their
time spent on things they like. Obviously, people lived in an time teemed with tense competition and they are often under
a huge pressure. After a whole day job or a period of work, you probably make yourself drunk, or go for shopping crazily, or
bury yourself in a series of insane music, or whatsoever you prefer. Of course you are most likely to become relaxed and
inspired through these favorite experience. Eventually, the great physical and mental recreation is conducive to a better
performance in your profession the next day.
To sum up, it is unreasonable to separate apart what people like to do and what they should do. The pursuit on personal
enjoyment and assumption on accountability are interwoven. They do not contradict to each other. The effort and time on
personal enjoyment does not equal to the ignorance on people's responsibility because of the objective requirement from
each identity. Moreover, personal hobbies are good catalyst for improvement on the original professions.

Level 5 Level1-Level5

TPO 5
TPO5-

TOPIChow Chaco great houses were used

1. residential purpose

1. there should be many fire places left, since there were


more than one hundred of families there, but the reality is
that only few fire places were found.

2. storage use for food supplies

2. excavations show no trace of grains or maize containers.


If the 'great houses' were used to store crops, there should
be at least some spills of crops or remains of containers.

3. ceremonial center because of broken pots found in


deposits

3. the mound contains a lot of other materials besides


broken pots, such as building materials of sand, stones or
construction tools, and those were pretty much just trash
heap of construction materials.

Sample Answer-1
The topic is about what 'great houses' were used for. From the passage, there are three different guesses. However, the
speaker does not think those are convincing, and he illustrates his three arguments in response to other threes in the
article respectively.
First, in order to challenge the residential purpose demonstrated by the reading, the listening identifies that there should
be many fire places left, since there were more than one hundred of families there, but the reality is that only few fire
places were found. In a word, residential use is not plausible.
Furthermore, in the passage storage use for food supplies is considered to be another explanation of Chaco structures. On
the contrary, the lecturer in the listening says this theory is unsupported, because excavations show no trace of grains or
maize containers. If the 'great houses' were used to store crops, there should be at least some spills of crops or remains of
containers. Thus, the houses should not have been used to store foods.
Finally, the listening rebuts the reading's idea of ceremonial center because of broken pots found in deposits, by stating
that the mound contains a lot of other materials besides broken pots, such as building materials of sand, stones or
construction tools, and those were pretty much just trash heap of construction materials. In other words, broken pots are
not enough to prove that 'great houses' were built for ceremonies.

Sample Answer-2
The author of the reading passage proposes three theories as likely explanations of the primary function of Chaco Canyon
houses, giant structures built in the 12th century. The lecturer, however, points to the inaccuracies in each of these
theories.
The lecturer argues that the modest number of fireplaces in these structures is in contradiction with the huge size of these
houses, indicating that these structures could not have been used for residential purposes. The reading, however, draws
comparisons between the Chaco houses and other similar large residential structures in support of the residential
theory.
The second theory, that the houses were used for food storage, is also rejected by the lecturer. He explains that a place
that had been used for storing maize would have many traces of scattered maize, which is not the case in the area of the
Chaco Canyon houses. This proves that the food storage theory is unlikely.
Finally, regarding the third theory, the ceremony theory, the lecturer contends that the presence of broken pots close to
the great houses does not offer sufficient proof that this was a place for ceremonial activities. He argues that there are
other materials such as pieces of construction trash found along with the broken pots, which suggest that these pots were
probably not used for ceremonial purposes but instead were discarded by construction workers upon completion of the
great houses.

Level 3 Level1-Level5
TPO5-

People today spend too much time on personal enjoyment-doing things they like to do-rather than doing things they
should do.

//

3. spending much time on personal enjoyment does not indicate that people ignore their accountability which should be
taken.
4. to pursue one's personal enjoyment help to assume the responsibility. People should not be blamed for their time spent
on things they like.

Sample Answer
The above statement poses a hot topic on contradiction of people's personal enjoyments and responsibilities. And the
speaker tends to think people now are dedicating more on personal likeness than on what they should do. I strongly
disagree with this opinion because I think that to pursue one's individual enjoyment does not contradict to assume the
responsibility.
Admittedly, people nowadays increasingly enjoy freedom to pursue personal enjoyment than before. As the development
of democracy, the ideas and behaviors of individual are respected and encouraged to some extent. For instance, students
can choose any major which they favor mostly; graduates can participate in whatever professions they are interested in;
citizens possess absolute freedom on free press and behaviors they like. Briefly, people have much time and freedom to do
their favorite things.
However, spending much time on personal enjoyment does not indicate that people ignore their accountability which
should be taken. Let's take a college student in art for example. Generally, students in art major should spend much time
on reading and appreciating art history, biography of artists and art works. Yet, the student favors more on economics. He
probably continues to earn his degree on art with great interest in economic as well. Or he may pursue a second degree on
economics. No matter what his decision is, he takes his responsibility to become a worthy student of his major.
In addition, to pursue one's personal enjoyment help to assume the responsibility. People should not be blamed for their
time spent on things they like. Obviously, people lived in an time teemed with tense competition and they are often under
a huge pressure. After a whole day job or a period of work, you probably make yourself drunk, or go for shopping crazily, or
bury yourself in a series of insane music, or whatsoever you prefer. Of course you are most likely to become relaxed and
inspired through these favorite experience. Eventually, the great physical and mental recreation is conducive to a better
performance in your profession the next day.
To sum up, it is unreasonable to separate apart what people like to do and what they should do. The pursuit on personal
enjoyment and assumption on accountability are interwoven. They do not contradict to each other. The effort and time on
personal enjoyment does not equal to the ignorance on people's responsibility because of the objective requirement from
each identity. Moreover, personal hobbies are good catalyst for improvement on the original professions.

Level 5 Level1-Level5

TPO 6
TPO6-

TOPICcommunal online encyclopedias VS traditional encyclopedias

1. the lack of academic credentials from online


encyclopedias leads to errors

1. traditional encyclopedias are much worse than online


encyclopedias. the communal online encyclopedia can be
corrected anytime, however, errors in the printed ones
might be there for decades.

2. easy online abuse of information

2. Read-only format was designed to prevent integrity of


sources from malicious hackers, so that no one can change
facts. the special editors were introduced to monitor
changes and avoid the possibilities of malicious
modifications.

3. an improper impression of what is important and what


is not

3. having enough space, the online encyclopedias can offer


not only academic articles but also variety of articles and
topics which can reveal users' diversified interests. the large
range of information is an advantage rather than a
disadvantage.

Sample Answer-1
The reading compares online encyclopedias with traditional printed encyclopedias, and insists that the new form is not as
valuable as the traditional one. In contrast, the listening holds a different view through revealing that the criticisms are the
result of prejudice against and ignorance about how far online encyclopedias have come.
First, in order to rebut the reading's statement of the lack of academic credentials from online encyclopedias, the woman
says that talking about errors, traditional encyclopedias are much worse than online encyclopedias. Especially, when we
look through a complex reference, rarely can it be error- free. Furthermore, the communal online encyclopedia can be
corrected anytime, however, errors in the printed ones might be there for decades.
Second, the passage shows concerns with easy online abuse of information. But the listening believes that this worry can
be eliminated by two protection methods. Read-only format was designed to prevent integrity of sources from malicious
hackers, so that no one can change facts. Moreover, the special editors were introduced to monitor changes and avoid the
possibilities of malicious modifications.
Finally, to challenge what the reading points out, that communal encyclopedias' profound discussions on all topics create
an improper impression of what is important and what is not, the listening discloses that the reason why the traditional
encyclopedias have to consider the "proportion" of each information is that they do not have enough space. On the
contrary, having enough space, the online encyclopedias can offer not only academic articles but also variety of articles

and topics which can reveal users' diversified interests. In a word, the large range of information is an advantage rather
than a disadvantage.

Sample Answer-2
The lecturer addresses each of the three criticisms of communal online encyclopedias mentioned in the reading passage.
The lecturer admits that communal online encyclopedias, like any reference book, may contain errors, but she claims that
these errors can be corrected much more easily and quickly than those printed in a paper encyclopedia. The reading
passage, in contrast, points to the inaccuracy of information in online encyclopedias, presenting the argument that errors
in these resources are due to lack of professional knowledge among contributors.
The lecturer then gives two strategies that have been proven very effective in protecting online encyclopedias from
malicious alteration. She explains that contents that consist of indisputable facts are stored and presented in a read-only
format so that nobody can make changes to them. In addition, she says, there are specialists who constantly monitor
contents online so that they can quickly remove a suspicious change once it is detected.
The lecturer also challenges the final point in the reading regarding the nature of topics covered in online encyclopedias.
She says that because of virtually unlimited space on the Internet, there is no need to worry about what is important
enough for inclusion in an online encyclopedia. Moreover, the greater variety of topics in online encyclopedias more truly
reflects the genuine interests of the general public, even if some of these topics are less serious or academic than those in
traditional encyclopedias.

Level 5 Level1-Level5
TPO6-
Life today is easier and more comfortable than it was when your grandparents were children.

5. Economic growth enables people not to worry about their material life;
6. Choice of entertainment makes it possible to enjoy leisure time;
7. Social insurance contributes to the easiness of life.

Sample Answer-1
Life is always difficult and the hardship appears in different aspects under different backgrounds. For our grandparents,
people had to strive for raising several children with thin income. Nowadays, we have to keep track with the high pace of
the society.

In the past year, the food was inadequate and the salary was low. In order to cultivate several children, adults had to find
additional work to make up thin payment. The huge pressure from living drove them to eat and wear simply. I was often
told that my grandparents merely had rice fordinner without any vegetables, let alone meat. To reduce the burden of their
families, our grandparents went to work earlier, some of them were even immature. When they received money, they did
nothing but to make an informed arrangement on how to spend it. For example, the daily necessities, the electronic bill,
phone bill and the tuition of younger kids were all needed to be taken into account. Thats why they seldom bought
themselves new clothes and had some entertainment. Furthermore, in most families,the younger children had to wear the
clothes from their brothers and sisters only because they could not afford the new ones. At that time, grandparents had no
choices but to squeeze the expense every day for the preparation for the New Year. It was their effort that resulted in our
relative happier life today.
However, nowadays we also have to face infinite stress from the highly developed society. Admittedly, we are not that
poor as our grandparents, yet the difficulties of our age never fade next to theirs. Actually, the fierce competition provides
us with narrow chance to escape but to work hard, even harder than our grandparents. In spite of the pressure physically,
we are confronted with the mental problems. Children have to bear the constant expectation from their parents and
school and they need to find an efficient way to release their emotions. Otherwise, they will crash in the end. Parents life
is also not easy. To earn a better living, they have to keep energetic both in workplace and at home.Besides, it is essential
for them to grasp more skills and take in more information for the sake of good relationship and opportunities of
promotion. Sometimes, they will find it hard to keep a balance between family and work and it usually results in
incongruous and annoying quarrels. Consequently, their exhausted mind and body break them down, making them look
elder than the real age.
Form the material aspect, people today enjoys a more colorful world. There is no point in worrying the food and clothes,
but more challenging problems emerge and they can easily destroy us, even stronger than the starvation because they are
both mentally and physically. In sum, life is not fair, get used to it. If we persist, we can still light our day.

Sample Answer-2
It seems that people always have a desire to compare: am I prettier than others? Are we living in a better age? Or, as the
question goes, is our life easier and more enjoyable than it was several decades ago? I would say no, as I have witnessed
the struggle and paradox of our generation.
First of all, we are now living in an age of revolution with no previous human experience that could be referred to. Living in
such a fast-changing world would not be easy and comfortable at all; with everything keeps changing, everyone has to
move fast in order to catch up with the majority. However, people were much more stable and care-free when our
grandparents were children; they did not have to learn a second language in order to get a better job, or read a lot in order
to get informed. Yet in our age, these are supposed to be the responsibility of young people.
Another discomfort of our age is that we are experiencing interpersonal alienation. With the development of
communication technology, people in fact become alienated with friends and relatives since they could be so easily
accessed via phone or email. Too often we feel that few of our friends are true friends, without the traditional feeling of
mutual affection which could only be created by longtime apart. But several decades ago, people treasured their friends
and maintained relatively close interpersonal relations, which is much more enjoyable than the estrangements we are
experiencing.
The last factor that has made our age so uncomfortable is the abusive use of technology. Although it brings much

convenience, it essentially changes human we are no longer the master of tools, but instead the slaves of devices. We
rush to metro station in order to catch an early train, sit in front of radioactive computers all day long in order to get our
work done, and stay in air-conditioned rooms all summer without experiencing the natural changes outside thick cement
walls. Several decades ago, people could still live closer to nature and make rational use of modern technology, which to
me is the essence of human living experience.
However, the comparison between different times is itself ridiculous. The criteria could not be easily determined, and
opinions are highly personal. Anyway, we have a longer life span, more advanced medical facilities, and easier traffic than
our grandparent when they were children, and we should treasure the present experience. Imagination of the past might
only be nostalgia if I ask my grandparents the same question, they might as well say that our life is much better than the
past generations. Who knows? As long as the world is still peaceful, life at any time would not become too difficult to
handle.

2007.12.10
2012.11.24 / 2014.03.16

It is easier for people nowadays to become educated than in the past.

Level 2 Level1-Level5

TPO 6
TPO6-

TOPICOnline vs. traditional encyclopedias

1. contributors of online encyclopedias lack academic


credentials, contributions partially informed or
inaccurate;

1. traditional encyclopedias never close to perfectly


accurate;
easy for errors to be corrected in an online encyclopedia.
But with the printed encyclopedia, the errors remain for
decades

2. the communal nature gives unscrupulous users the


opportunity to fabricate, delete, and corrupt information

2. put the crucial facts in a read-only format;


special editors monitor all changes made to the articles and
eliminate those malicious changes

3. focus too frequently, and in too great a depth, on


trivial and popular topics

3. Traditional encyclopedias have limited space, but space is


not an issue for online encyclopedias. There can be a great
variety of articles and topics that accurately reflect the great
diversity of users' interests.

1.

1.

2.

2.

3.

3.

Sample Answer
The professor refutes the viewpoint in the reading passage that online encyclopedias have several limits and thus are
inferior to traditional ones.

First, while the passage argues that online encyclopedias may lack accuracy and completeness, the professor points out
that printed encyclopedias also have errors. According to the professor, no one can find perfect resources without any
mistakes both on and off-line. Besides, it is easy to correct mistakes in online encyclopedias while in printed ones mistakes
will remain for decades.
Second, as mentioned in the reading, hackers have the opportunity to corrupt information in online encyclopedias. In
contrast, the professor states that two measures are taken to deal with this situation. Crucial information is put in a
read-only format and special editors are there to monitor any changes and delete the incorrect ones.
Last, according to the passage, online encyclopedias focus primarily on trivial and popular topics, misleading us about
whats significant and whats not. However, the professor suggests that the limited space of printed encyclopedias cant
reflect the broad interests of people while online ones dont have space issue and they can represent a greater diversity of
user interests. 191 words

Level 5 Level1-Level5
encyclopedia , academic credentials , communal , fabricate , format
contribution
read-only format

TPO6-

Life today is easier and more comfortable than it was when your grandparents were children.

1. Analyzing the question


+

life easier and more comfortable

2. Defining Position
life today is easier and more comfortable Lifeeasier, more comfortable
life ()/
+ 1+ 2+

3. Brainstorming

4. Topic sentences ()
, 2
4-1 First, people today enjoy much better basic living conditions compared with the impoverished families of our
grandparents age.
4-2 Second, the Internet is a revolutionary invention that has vastly improved our life, from the way we communicate to
how we obtain information and learn.

5. Keyboarding ()
5-1 Introduction
Introduction=General Statement+Thesis Statement

5-1-1 It is undeniable that life is changing for the better in many aspects. Todays generation is enjoying an
unparalleled level of wealth and comforts which previous generations could hardly imagine when they were young.
5-1-2 + Overall, despite the more intense competition and pressure, I believe that life today is easier
and more comfortable than it was when my grandparents were children.
5-2 body paragraphs ()
5-2-1 1++ topic sentence 1+details
5-2-2 2++ topic sentence 2+details
2 3
the other side
5-3 Conclusion
+Reworded Thesis Statement+topic sentences

There is no doubt that living in a fast-changing and progressing society means more challenge and stress while life of our
grandparents was relatively simple and less competitive. However, equipped with much higher standards of living and
more knowledge and information, todays generation can adapt to the fast pace and high pressure and create an even
better life.

Sample Answer
It is undeniable that life is changing for the better in many aspects. Todays generation is enjoying an unparalleled level of
wealth and comforts which previous generations could hardly imagine when they were young. Overall, despite the more
intense competition and pressure, I believe that life today is easier and more comfortable than it was when my
grandparents were children.
First, people today enjoy much better basic living conditions compared with the impoverished families of our grandparents
age. With the dynamic economy and fast-advancing technology, feeding a large population becomes a reality. Not only can
we enjoy adequate nutritious foods, but also we have a wide range of foods produced at home and abroad. By contrast,
my grandparents can never forget the memory of going to bed hungry. In addition to food, sanitation and medical facilities
have kept updating so that people do not die from preventable diseases or plague. It is reported that the average life span
has reached around 70 years, which is a remarkable achievement comparing to the last century when many people died
prematurely.

Second, the Internet is a revolutionary invention that has vastly improved our life, from the way we communicate to how
we obtain information and learn. In the past, the only means of getting the message across is mouth-to-mouth and people
rarely knew the world beyond their small village. In the present society, on the other hand, the globe is just a few clicks
away. The most amazing change is that via the Internet, we can make friends, do business and pursue education on a
global scale. The massive online open course is a forceful example that todays generation is enjoying a much easier and
more comfortable life. Students are able to get access to the best educational resources and even get a degree for free,
which is unimaginable for my grandparents generation.
There is no doubt that living in a fast-changing and progressing society means more challenge and stress while life of our
grandparents was relatively simple and less competitive. However, equipped with much higher standards of living and
more knowledge and information, todays generation can adapt to the fast pace and high pressure and create an even
better life. (368 words)

It is undeniable that
unparalleled
intense
compared with
by contrast
impoverished
dynamic
a wide range of
in addition to
sanitation and medical facilities
preventable diseases
It is reported that
average life span
prematurely
revolutionary
massive open online courses
on a global scale
get access to
unimaginable
relatively

(A/D)It is easier to maintain good health today than in the past. (2013/12/05, 2012/05/26)
It is easier to become an educated person than in the past. (2012/11/24)
In the past, people ate food that was better for their health than today. (2011/01/08)

Level 3
Lifeeasier, more comfortable

TPO 7
TPO7-

TOPICwhether wood companies in America will adopt ecocertification

1. American customers would reject advertising about


eco-certified products

1. American consumers do not simply treat every


advertising as the same. Instead, they would distinguish
between advertisement promoted by the company of the
product and the advertisement from independent
certification agencies. As Americans have more confidence
in independent agencies, they would be more willing to pick
up products with independent or some international
certification.

2. high price of eco-certified wood as a backward for


American consumption

2. price matters, only when the competing products' price is


much higher or lower. If there is a slight 5% difference
between prices, consumers would concern more on other
factors rather than on the payment.

3. this strategy is unnecessary because American


consumers are likely to be content with domestically
manufactured products

3. the push of chasing certification is foreign competition.


Americans are so concerned with ecosystem that American
companies may lose those customers, if they pay less
attention on eco-friendly products. As a result, foreign
companies would crowd in with eco-certified wood, and
then domestic wood companies may be under the risk of
losing market share.

Sample Answer-1
Although both the reading passage and the listening discuss eco-certification by the example of United States wood
companies, they hold different views. The passage's position is that in order to attract customers in United States, the
certification for ecologically friendly wood companies is not good enough. But the man from listening takes an opposite
side, and illustrates three particular points to support his idea.
Firstly, the passage states that American customers would reject advertising about eco-certified products. Opposite to that,
the listening reveals that American consumers do not simply treat every advertising as the same. Instead, they would
distinguish between advertisement promoted by the company of the product and the advertisement from independent
certification agencies. As Americans have more confidence in independent agencies, they would be more willing to pick up
products with independent or some international certification. That is to say, eco-certificated wood is more favored in
America.
Secondly, the reading regards high price of eco-certified wood as a backward for American consumption. However, the

listening rebuts that price matters, only when the competing products' price is much higher or lower. If there is a slight 5%
difference between prices, consumers would concern more on other factors rather than on the payment. In other words,
Americans prefer to value environmental preservation related products.
Thirdly, the man from the listening says that the push of chasing certification is not foreign consumers as indicated by the
reading, but foreign competition. Americans are so concerned with ecosystem that American companies may lose those
customers, if they pay less attention on eco-friendly products. As a result, foreign companies would crowd in with
eco-certified wood, and then domestic wood companies may be under the risk of losing market share.

Sample Answer-2
The lecture and the reading passage give contradictory opinions on the topic of eco-certification, a form of accreditation
conferred by an international agency in recognition of a companys eco-friendly practices. The passage explains that it is
not necessary for American wood companies to pursue eco-certification while the lecture provides several
counterarguments to this view.
First, the lecturer argues that the reading passage is too general in its statement that American consumers reject
advertising completely. He asserts that Americans do not trust advertising claims for a product only when these claims are
made by the company that sells the product. When a claim is made by an independent third party such as a wood
certification company, he posits, consumers respond very positively with strong acceptance of the certified product.
The lecturer also refutes the second point in the reading that price-sensitive American consumers are likely to choose
cheap wood products without certification. The professor contends that certified wood is only slightly (less than 5%) more
expensive than uncertified wood, and therefore,he argues, consumers will tend to ignore the price difference and choose
the eco-friendly product.
Finally, receiving eco-certification is, according to the professor, an important strategy used by American wood companies
to ensure that their products can compete against eco-certified wood products imported into the domestic market from
foreign countries. According to the reading, however, this strategy is unnecessary because American consumers are likely
to be content with domestically manufactured products, even if they are not eco-certified.

Level 3 Level1-Level5
TPO7-
It is more important for students to understand ideas and concepts than it is for them to learn facts.

8. learning the facts promotes the efficiency in studying the concepts.


9. studying the facts can be beneficial to deepening the concepts.

Sample Answer-1
Some of my friends are concerned only about learning facts so much so that they know thousands of them. However, I
believe that it is more important to understand ideas and concepts for a variety of reasons.
Students need to understand the big picture before they can begin to concentrate on the details. Merely knowing the
details is not the same as understanding why or how something happened. For example, my history teacher often makes
sure that we understand the broad, historical situation of an event before filling our minds with a variety of facts. Without
this broad knowledge, we could not be able to possess a deeper understanding of certain events.
Many universities are now focusing on teaching students the value of ideas and concepts. Without understanding them, a
students grade will truly suffer. My sister told me that many of her university classes are seminars or classes and students
discuss the ideas and theories behind what they are studying. The students are not expected to memorize lists of facts.
They instead need to know the importance of what they are studying.
Finally, creativity does not come from the mere knowledge of facts. Thinking in terms of broad concepts and ideas is what
helps a person to become more creative. In the 1990s, Apple computer went into a serious decline when its founder Steve
Jobs, was pushed out of the company. His return several years later, however, revitalized the company, and Jobs has been
the driving force behind many of the companys innovative electronic products. Steve Jobss abilities to be creative and to
think in board concept have made his company one of the top ones in the world. His focus on ideas and concepts was
what enabled Apple to become this way.
Ultimately, the ability to learn ideas and concepts is crucial to having a broader understanding of things, to do better in
school, and to develop new ideas of ones own. Knowing facts, while nice, is not as important as knowing the ideas behind
them.

Sample Answer-2
I definitely see the use of understanding various ideas and concepts. However, learning facts is of much greater importance
to people than knowing about ideas and concepts.
Because schools place an emphasis on learning facts, students must do so in order to do well at school. A comprehensive
knowledge of facts will enable students to pass their futures. My chemistry teacher is constantly having us memorize the
characteristics of the different elements. We have to repeat these facts on our tests, or else we will do poorly. Knowing the
concepts in my classes is much less crucial than being able to repeat facts.
Knowing facts is of much greater importance than understanding ideas and concepts in the working world. Possessing
knowledge of lots of facts is therefore one way for people to excel at their jobs. My aunt works at a large company, and she
is constantly attending meetings where she has to talk about different products. She knows everything there is to know
about them, and she can discuss the products without using any notes at these meetings. Her comprehensive knowledge
has impressed her bosses, who recently gave her a promotion for her outstanding work. Clearly, knowing a lot of facts on
the job can be of tremendous help.
A person who has an encyclopedic knowledge of various topics will be respected and admired by many people. Possessing
a lot of factual knowledge can help one impress people, which can work to a persons advantage. Some people refer to my
cousin as a walking encyclopedia. Whenever people want to know the answer to a question on virtually any topic, he
always seems to know the answer. He has won many trivia contests and even runs a successful tutoring business. Learning

facts can help a person lead a very successful life.


In conclusion, people should focus on improving their factual knowledge as it can help them in school, at their jobs, and in
their lives in general. A complete understanding of ideas and concepts is fine, but more emphasis should be placed on
learning facts.

Sample Answer-3
Ideas or concepts or facts, which help students more; which have a significant influence to students to adapt the amazing
developed society? Facts are results of observation, are something cannot changed but can utilize follow the direction of
ideas and concepts. However, on the other hand, ideas and concepts, which cannot see directly, are the creation of mind.
In my opinion, with the development of the technology and society, ideas and concepts are more important for students
due to most of the problems can be solved by using ideas and concepts. The following details can account for my point of
view.
With the development of the fast-paced society, those who have new ideas and use concepts correctly can stand out from
the crowd. Only when individuals bear concepts and ideas in mind can they achieve what they want. On the contrary, just
remembering facts are unable to solve problems directly and efficiently. Take working out maths examination questions for
an example: One cannot remember all details of the questions due to the fact that there are countless questions in maths.
In order to work out the question efficiently, the only way is to keep the maths concepts in mind, utilize them when
encounter difficult questions. If students just remember the facts, they cannot understand and do not know how to do the
relative subjects.
Furthermore, with the technology improve to a higher level, what the society need is a great deal of people who bear
innovation in mind. That is to say, creativities are becoming increasingly important to us. In addition, high technology
makes machine more and more intelligent. And database can hold large numbers of information for us, so we do not have
to remember so many facts. Whereas, elites nowadays are those who have lots of new and advance ideas in mind, creating
and inventing things that benefits our daily life. Thus, ideas play a more important role in students' character.
Facts are tools, while ideas and concepts are essence of creativeness, which make human differ from machines and the
wisdom stand out of the common. Thus, it is more important for students to understand ideas and conception instead of
just wasting their brain to record facts.

2007.02.04
2007/12/15

Level 5 Level1-Level5

TPO 7
TPO7-

TOPIC

1.

1.

2.

2.

3.

3.

Sample Answer
The professor in the lecture explains why the US wood companies should gain the ecocertification. There are following
three reasons.
First, it is necessary for US wood companies to pursue the ecocertifications. This is because American consumers do not
hold the same attitude towards all advertising. They do not value advertisements claimed by producers themselves, while
they have confidence in commercials made by independent agencies with international reputations for trustworthiness.
Therefore, the ecologically minded Americans may be in favor of the ecocertification made by independent agencies. This
contradicts the point in the reading that American consumers do not value advertisement at all because of too much
advertising.
Second, the price may not the only factor interfering consumers choice. The price would decide peoples choice only when
the price of one competing product is much higher or lower than the other, but the price of ecocertified wood only
increases less than 5 percent. Furthermore, Americans are becoming more aware of the significance of environmental
protection. Thus, getting the ecocertifications is still a must for US wood companies. This is how the lecture casts doubts
on the point in the reading that American consumers would not choose ecocertified wood because of the higher price.
Finally, foreign competition makes sense for American wood companies to obtain the ecocertification. Although home
market is the main target for most American wood companies as the passage indicates, ecocertifications are also
necessary since foreign competitors may begin crowding in American market if their local counterparts do not have
ecocertified products that could attract the American consumers.

Level3 Level1-Level5
TPO7-
Do you agree or disagree with the following statement? It is more important for students to understand ideas
and concepts than it is for them to learn facts. Use specific reasons and examples to support your answer.

TPO

Sample Answer
A fact is something that we have noticed in daily life, such as the sun rising in the east and setting in the west. However,
what we do not notice is the ideas or the concepts that could explain the reasons for daily facts. In fact, the two items are
so interrelated that we even could not emphasize one at the expense of the other. Therefore, I believe that it bears the
same importance for students both to understand ideas and concepts and to learn facts.
To begin with, a good understanding of ideas and concepts can contribute to the efficient learning of facts for students.
The primary reason for this is that the ideas or theories are the essential summaries of various facts with which students
can have deep comprehension towards the facts they encountered in the daily life. For instance, when I was in primary
school, I was curious about the fact that people on the bus keep moving forward when the bus stops and I wonder why
this would happen. When I was in secondary school, I learned the First Law of Motion by Newton, known as the law of
inertia. My puzzle in primary school was perfectly solved by the theory of Newton. After that, I could quickly understand
almost all the facts related to the law of inertia, such as the safety belt in cars and shaking the thermometer before using it.
Clearly, understanding the notions does help students to learn the facts.
Furthermore, facts are the source of ideas and concepts. If there is no fact to support the ideas, then the so called notions
or theories are just the castle in the air. In fact, all the scientific theories are based on the vast observation of facts. The
discovery of evolution by Darwin could serve a good example to testify this point. He once travelled with the fleet as a
doctor and visited a lot of places around the globe. He studied plenty of fossils found in the place where he set foot on,
and then he combined all the facts he found. After a long and arduous research, here came the great theory of evolution
that later had a groundbreaking influence on both on science and society. The survival of the fittest explains that the
connection of different species and how the creatures develop throughout time. It is obvious that a concept or an idea
does come from the study of fact.
Interestingly, based on what I have discussed above, here comes the conclusion that facts and ideas are paradoxically
connected, as an idea comes from the facts and the idea may have a positive role in explaining nearly all the facts related

to the idea. Therefore, I do not agree with the statement in the topic, and I think that facts and ideas are both important
for students to know about.

Level5 Level1-Level5

TPO 8
TPO8-

Chevalier de Seingalt
TOPICSeingalt However, . than it really was.

If the Chevalier had really been very rich, he would not


have needed to borrow money.

The accuracy of the conversations between himself and


Voltaire in the memoir was doubted.

Critics have also questioned the memoirs account of the


Chevaliers escape from a notorious prison in Venice,
Italy.

1. Loan poor; Money: parties & gambling in Switzerland;


Sell the property to get money;
Ran out of cash borrow money.

2. After conversation: write down everything that he


could remember;
Keep the notes for years;
Referred to the notes when writing was confirmed;

Others: have more powerful friends but fail to bribe their


way to freedom, bribery
Venetian government documents soon after escape: the
ceiling of his old prison room was repaired;

Sample Answer
The lecture completely refutes the reading which raises doubts about the accuracy of the memoir written by the Chevalier
de Seingalt. In the lecture, the professor believes that the Chevaliers memoir is a reliable historical source.
To begin with, the reading presents an example that the Chevalier had to borrow large amounts of money from others, so
he must have fabricated stories of his wealthy life in Switzerland. However, the professor counters that argument by
asserting that the Chevalier spent a large sum of money on parties and gambling. Besides, not all the assets are cash, some
of which have to be sold first in order to get money. So when the Chevalier ran out of cash, he had to borrow some before
his property was converted into actual money. That doesnt mean that he was poor.
Besides, in contrast to the reading, which doubts the accuracy of the conversations between the Chevalier and the famous
writer Voltaire, the lecturer insists that the Chevalier wrote down everything he could remember each night immediately
after the conversation with Voltaire. There is evidence that the Chevalier kept those notes for many years and referred to
them when writing his memoir. Witnesses also confirmed that he regularly consulted notes when composing the memoir.

Finally, the critics in the reading express that the Chevalier might actually bribed his way out of a prison in Venice instead
of the exciting escape. However, the professor challenges the skepticism by stating that other prisoners in that prison, who
had more powerful friends, failed to bribe their way to freedom. Therefore, bribery seems unlikely. There exist some old
Venetian government documents which indicate that the ceiling of the old prison room was repaired soon after the
escape.

Level 3 Level1-Level5

TPO8-

Do you agree or disagree with the following statement?


Television advertising directed towards young children (aged two to five) should not be allowed.
Use specific reasons and examples to support your answer.

2 AgreeNave
susceptible
1
2
3 Disagree
1
2

Describe the realityAdvertisers: mastered every weakness and loophole in human nature; advertisements:
a display of the newest psychological discoveries; Disagree with the statement.
1Young children can hardly refine the massive information. + the filtering-and-blocking skill


2Young children lack the ability to identify the differences between the fiction and fact, and therefore they are more
likely to take advertisements as they appear to be. +

Sample Answer
As long as one does not pursue the life of a recluse, television advertising is inevitable in ones life. They may be too
common to be noticed, but their influence never deserves ignoring. In their effort to compete for customers, advertisers
have mastered every weakness and loophole in human nature. Hence, advertisements gradually became a display of the
newest psychological discoveries, instead of a presentation of products, in tremendous scale. Eventually, we surrender and
give up our money in exchange for something that we might never be in need of. In regard for their considerable power, I
totally agree with the sociologists who insist that television advertising directed toward young children aged two to five,
who more susceptible, should not be allowed.
First of all, young children can hardly refine the massive information. People gradually obtain the skill to filter and block
information as they grow up. We, as adults, may sometimes stare at the screen but take nothing from the advertisement in
mind, although this may not contribute to the blocking skill but simply sleepiness. However, unfortunately, according to a
recent study of psychological group from University of Iowa, the filtering-and-blocking skill is quite undeveloped in young
children. In other word, the psychological tricks embedded in advertising would exert more significant influence in them,
breeding stronger temptation to purchase certain product, which may prove to be useless afterward. Morally, it is unjust to
take advantage of the susceptibility of younger children.
Besides, young children lack the ability to identify the differences between the fiction and fact, and therefore they are
more likely to take advertisements as they appear to be. In most cases, advertising does not aim at illustrating the quality
or other virtues of certain goods, but to left an impression of that good in viewers mind. For adults, advertisers could use
movie stars or singers to strengthen the impression, and for children, they may use images from animations or even some
magical scene. This does not look evil at the first glance, as it simply response to young childrens interests and imagination.
However, researchers in University of Iowa found that young children tend to take images in such advertisements as real.
From the study, over seventy percent of children responded to a toy superman suit enthusiastically as a result of believing
that the suit could really let them fly, as was depicted in the advertising. The incapacity to discern imagination from reality
negatively influences young childrens judgment when selecting goods by focusing childrens attention on things that do
not promise to be. Do not take this as defaming young childrens intelligence: even a smart five-year-old young may
believe in Santa Claus.
The manipulating power of advertisements on young children is unjust. As it takes advantage on childrens susceptibility to
psychological suggestions and incapability to distinguish the false from the real, television advertising directed at children
younger than five should be banned.

2012/12/02 Movies and television have more negative effect than positive ones on young people's behavior.
2013/12/21 Advertisement is not honest than it is in the past; and can not help us to decide what product do we buy.

Level 3 Level1-Level5

TPO 8
TPO8-

TOPICChevaliers memoir ()

1 claimed to be very rich, but borrowed considerable


sums of money from a Swiss merchant.

1. He had wealth but it was the property that had to be sold


first to get money. It usually took a few days, during which
he had to borrow, to convert his assets into actual money.

2.the memoir cannot capture the conversations between


him and Voltaire accurately, because it was written many
years after the conversations occurred.

2. each night immediately after conversing with Voltaire, he


wrote down everything he could remember. Chevalier kept
his notes of these conversations for many years and
referred to them when writing the memoir.

3. more likely that the Chevaliers jailers were bribed to


free him, because Chevalier had politically
well-connected friends

3. Other prisoners had even more powerful friends, and


none of them were ever able to bribe their way to freedom,
government documents indicate that soon after the
Chevalier escaped from the prison, the ceiling of his old
prison room had to be repaired.

TOPICChevalier

1. Chevalier

1. Chevalier

2. Chevalier

2. Chevalier

3. Chevalier

3.
Chevalier

Sample Answer
While the passage holds that Chevaliers memoir may have distorted or exaggerated many events to make his life seem
more thrilling, the professor maintains that the memoir is a reliable historical source.

The first doubt in the reading is that Chevalier borrowed money from a Swiss merchant despite his claim of being wealthy.
The professor, however, explains that borrowing money doesnt mean Chevalier was poor. He spent huge money on parties
and gambling, and he had to wait for days when he converted his asset into real money, so it is reasonable for him to
borrow money.
In addition, according to the passage, the accuracy of Chevaliers conversation with Voltaire is also not reliable, because
the memoir was written many years after the conversation occurred. In contrast, the professor points out that after the
conversations, Chevalier wrote down the content every night, which served as the reference when he wrote the memoir.
Last, contrary to the doubt about/the reading questions Chevaliers escape from an infamous prison in Venice based on
the assumption that he could bribe the jailers; the professor suggests that there were other more powerful prisoners and
none of them could bribe themselves free. Also, the Venetian record shows that the ceiling was repaired after Chevaliers
escape, which fits his description of escape through the roof.

level 3 Level1-Level5
bribe, convert asset/property into actual money

TPO8-

TV advertising directed towards young children should not be allowed.

1. Analyzing the question


+

children

2. Defining Position

+ 1+ 2+

3. Brainstorming

TPO 14
passage 1 Children and advertising
young children

4. Topic sentences ()

4-1 First of all, misleading techniques, such as exaggeration, fantasy and celebrity, are commonly employed by advertisers
to draw and the attention of consumers. Unfortunately, children generally cannot tell the difference between fancy ads
and the actual products.
4-2 Besides, TV advertising targeted at young children has a subtle influence on their consuming psychology.

5. Keyboarding ()
5-1 Introduction
Introduction=General Statement+Thesis Statement

5-1-1 TV advertising is widely used as a means for companies or manufacturers to promote their products and
services.
5-1-2 + However, advertisements directly aimed at children are controversial as they may have a
negative influence on this vulnerable group. Considering the tempting nature of advertisements and childrens cognitive
development, I agree that advertisements targeted at young children should be banned.
5-2 body paragraphs ()
5-2-1 1++ topic sentence 1+details
5-2-2 2++ topic sentence 2+details
2 3
the other side
5-3 Conclusion
+Reworded Thesis Statement+topic sentences

Of course, as parents are able to analyze the information and make judgment on their own, TV advertising can provide
useful information to help them make right decisions in buying children-related products. However, considering its harmful
effects on childrens consuming habits and mentality, TV advertising should be forbidden among this age group so as to
minimize the negative influences on children.

Smple Answer
TV advertising is widely used as a means for companies or manufacturers to promote their products and services. However,
advertisements directly aimed at children are controversial as they may have a negative influence on this vulnerable group.
Considering the tempting nature of advertisements and childrens cognitive development, I agree that advertisements
targeted at young children should be banned.
First of all, misleading techniques, such as exaggeration, fantasy and celebrity, are commonly employed by advertisers to
draw and the attention of consumers. Unfortunately, children generally cannot tell the difference between fancy ads and
the actual products. As a result, they are easily tempted to buy this or that product, which may lead to wasteful consuming
habits or harmful effects on health. Research has shown that theres a direct association between commercials for junk
foods and mounting levels of obesity among children. Excessive consumption of the unhealthy food is the direct result of

TV advertising.
Besides, TV advertising targeted at young children has a subtle influence on their consuming psychology. It even distorts
their real needs by advocating whats in and whats not. As children are easily swayed by advertisements, they may
blindly follow the so-called mainstream lifestyle promoted by TV advertisements. For example, because of the intense
advertising campaign, it has become a trend for children to throw their birthday parties at McDonald. In time, children
may feel that McDonald and its coke and hamburgers are part of their birthday, even if they have the better option of
having nutritious food at home. In the long run, these children may grow up with fast food as their lifetime diet. Afraid of
being left out, an increasing number of children make their buying decisions on whether the products are advertised and
thus popular with their peers, rather than on their real needs. Peer pressure resulted from TV advertising has a long-lasting
influence on childrens consuming habit and psychology.
Of course, as parents are able to analyze the information and make judgment on their own, TV advertising can provide
useful information to help them make right decisions in buying children-related products. However, considering its harmful
effects on childrens consuming habits and mentality, TV advertising should be forbidden among this age group so as to
minimize the negative influences on children. (377 words)

In order to be successful, businesses must put more money in advertising. (2014/04/19)


Today advertising is less honest than it was in the past, for that reason it is less useful for people to choose what they want
to buy. (2013/12/21)
Most advertisements make products seem better than they really are.TPO17

Level 3 Level1-Level5

TPO 9
TPO9-

TOPIC Fuel-cell engines replace them.

: way too optimistic; not the solution

Fuel-cell engines utilize easily available, renewable


resources.

Not that easily available;


useable in the common form;
pure liquid state (highly artificial substance) store &
produce: difficult;
Eg: minus 253 degrees Celsius, elaborate cooling
technology.

Fuel cells will solve many of the worlds pollution


problems.

2. solve the pollution associated with cars;


Produce pure H2 a lot of pollution;
Purification process energy burning coal or oil
pollution;
The factories that generated the hydrogen pollute.

Fuel-cell engines will soon be economically competitive.

manufacture the fuel-cell engine: expensive;

components: platinum chemical reaction electricity;


Replacement: unsuccessful.

Sample Answer
Given the three advantages that fuel-cell engines have over the internal-combustion engines, the reading material claims
that the hydrogen-based fuel-cell engine is the most promising alternative source of energy for cars. The lecture, however,
believes that the reading is way too optimistic in the assessment of hydrogen-based fuel-cell engines and hydrogen cannot
solve those problems.
To begin with, even though the hydrogen needed for fuel-cell engines is a renewable resource and cannot be easily
depleted, as mentioned in the reading, the professor counters that hydrogen could not be available that easily, for it is not
directly useable in the common form. Hydrogen needs be obtained in a pure liquid state before being used in fuel-cell
engines. Pure liquid hydrogen is a highly artificial substance, which is difficult to produce and store. For instance, it must be
kept at minus 253 degrees Celsius. This requires elaborate cooling technology. Therefore, hydrogen is not that easily
available.
Besides, in contrast to the reading, which expresses that hydrogen-based fuel cells will solve pollution problems, the
lecturer insists that the process of producing pure hydrogen creates a lot of pollution. The purification process to get pure

hydrogen from water or natural gas requires a lot of energy obtained by burning coal or oil. This burning process creates
lots of pollution. Thus, although the cars will not pollute, the factories that generated the hydrogen will pollute.
Finally, the reading states that fuel-cell engines will be economically competitive for its lower cost of operation, whereas
the professor in the lecture argues that manufacturing the fuel-cell engine is extremely expensive, for the components
fuel-cell engines of are made of platinum, a rare and expensive metal. The hydrogen cannot undergo the chemical reaction
that produces the electricity without the platinum components. What is worse, There have not been any cheaper material
to replace platinum so far.

Level 3 Level1-Level5

TPO9-

Do you agree or disagree with the following statement?


Technology has made children less creative than they were in the past.
Use specific reasons and examples to support your answer.

2009 05 04

5 Agree
3 Photoshop; Overture
4

6 Disagree
3
4


:
Conflicting ViewsBoth the positive and negative influence on childrens creativity exists. However, the
development of technology still enhances the creative minds of children.
technology reluctant to think diligently e.g.: the invention of calculators
1Computers and the internet inspire the intelligence of children. +Photoshop, overture, 3D printer

2The advancement of technology also encourages childrens to concentrate on more productive pursuit, leading to
the improvement of creativity. +

Sample Answer
Admiration is always followed by fear or disgust; the case of technology is the same. While some critics may cast dubious
eyes on the influence of technological achievements on children, I embrace the breakthroughs, which also lead to the
improvement of childrens creativity.
True, technology has ensured a more convenient life for children, who might be reluctant to think diligently. For instance,
fewer and fewer kids demand the skill to do the number operations in mind these days, given the easy access to
calculators. The ability to find creative way to solve mathematic questions is diminishing in our children. However, that can
hardly lead to the conclusion that technology makes children less creative. Instead, technology simply eliminates certain
necessities, the disappearance of which should not be considered as drawbacks but advancement. Calculators, freeing us
from perplexing calculation, are akin to matches, liberating us from hitting flints to get fire.
Besides, computers and the internet inspire the intelligence of children, encouraging them to think more creatively.
Children nowadays are exposed to a large amount of technological devices and software. That allows children to gain a
wide range of knowledge they are interested in. For example, Photoshop provides platforms for creative graphic design;
Overture eases the process of composing a piece of music; a 3D printer makes it possible to design almost anything in
ones mind. These great works are essentially helpful to arousing their curiosity and prepares them with key knowledge
necessary for any creation. In contrast, children in the past rarely had chances to acquire the cutting-edge skills. Those
wonderful gadgets were not available in the past, when young students could only reach limited resources in public
libraries or schools. In this sense, technology has provided the possibility for children to be more creative.
Furthermore, the advancement of technology also encourages childrens to concentrate on more productive pursuit,
leading to the improvement of creativity. In the past, as described in Dickens work, Oliver Twist, most children were driven
to work place at an early age. If they are lucky enough to survive, they were more likely to attempt stealing to keep
themselves from starving to death than indulge in creative work of any sort. Nowadays, however, technology not only
provides better education opportunity, but various ways to access to it, which promote childrens creativity tremendously.
There was a Ted talk delivered by a fifteen-year-old teenager, who chose not to attain school but found study group in his
community and conducted self-taught lessons in totally original methods. He called his project hacking knowledge, which
highlights the innovative way in which each one can approach information. He and his group acquired knowledge from the
internet, conducted experiments by themselves, and got a chance for internship at Apple before high school. Had not
technology made easy access to information and facilities possible, such creative self-taught project could have never
come true.
To sum up, technology does not make children less creative than their counterparts in the past. Instead, it is the leading

power in the process of human evolution.

2009/05/04
2011/01/21 NA The rapid development of society today is a positive trend.
the development of technology has made children less creative.

Level 4 Level1-Level5

TPO 9
TPO9-

TOPIC

1.

1.

2.

2.

3.

3.

Sample Answer
The lecture explains why the new fuel cell engines cannot replace the traditional internal combustion engines. There are
following three reasons.
First, the author supports the theory of replacement because hydrogen is easily to obtain and cannot easily be depleted. In
contrast, the speaker in the listening believes that despite the easy access of hydrogen, the pure liquid hydrogen needed in
the fuel cell engine is highly artificial and difficult to obtain and store. For example, liquid hydrogen has to be kept in very
low temperature, -253 degrees. Therefore, the difficulty in producing and storing liquid hydrogen makes it harder to
replace the traditional engines.
According to the passage, another reason for the replacement is the environmental friendliness that fuel cell engines
posses. However, the professor challenges this idea by saying that the hydrogen-powered cars do not pollute the
environment, since the only byproduct is water, but the factories that produce liquid hydrogen would pollute. To obtain
the liquid hydrogen, a great amount of fossil fuels, such as coal and oil, would be burned, releasing a lot of carbon dioxide.
This poses greater risk to the environment.
Finally, the author points out that the replacement is possible because of its competitive price. However, the speaker
argues against this point by stating that there is a very expensive component made of rare and expensive metal in the fuel
cell engines and no cheaper alternatives have been found successfully so far. Therefore, the cost of the new engine may
not be competitive to the traditional one.

Level3 Level1-Level5

TPO9-
Do you agree or disagree with the following statement? Technology has made children less creative than they
were in the past. Use specific reasons and examples to support your answer.

2

3

Sample Answer
There is no doubt modern technology has greatly transformed peoples life. The best transformation could be reflected by
the rising creativity of children, since technological development provide more practical skills and more chances to expose
to different kinds of knowledge. Therefore, I believe modern technology boosts the creativity for children nowadays than
their counterparts in the past.
First, children nowadays are more creative because of the practical skills gained from the manufacturing technology. With
more practical skills, such as being more capable of hand-making, children can be more able to apply what they have in
their imaginative minds into practice, and this may inspire more creative work. The well-developed manufacturing can just
offer more chances for children to improve the hand-making ability. For instance, nowadays, factories have been able to
produce a great variety of precise models for children, like model planes, model boats. They are so similar to the real ones
that even all components can be separated by hand. My neighbor, Tom, is a primary school student as well as a fan of
model planes, and he is very adept at designing the new-modeled planes. Last year, he designed a model plane named
Hawk, and won the first prize as the most creative designing in the model plane competition. But , when I was little, I
even did not know what the model plane was.
Second, more exposure to all sorts of knowledge may contribute more to childrens creativity today. Thanks to the
booming of the information technology, such as the Internet and computers, children have more opportunities to learn
about physics, chemistry, and even astronomy, which are the basic subjects for scientific creations. A survey from the
National Department of Patent can serve as the good example to support the idea. The authority has collected the number
of patents contributed by children under 16 years old and compared those figures of 20 year ago. Here comes the
conclusion: the number of under-16- year-old patent holders has increased by 30 percents and being more knowledgeable
is regarded as the most important factor for this change. From this report, we can see that children nowadays surely are

more creative than those in the past.


It has to be admitted that the convenience brought about by the technological devises makes children lazier than before,
and the laziness may harm the creativity. For example, the all kinds of family appliances can be operated by a remote
control or even a cell phone. However, in my opinion, the so-called laziness is the force of creativity, as if there were no
lazy people, cars or planes may not be invented and people would still travel long distance by riding horse or on foot.
To sum up, due to the more knowledge and more practical skills by technology, children may be more creative than those
in the past.

2014/12/6 CN
Do you agree or disagree with the statement: Being creative, rather than planning carefully more often results in the best
solution of a problem.

Level 4 Level1-Level5

TPO 10
TPO10-

1. first, second(in addition, moreover, furthermore, etc), third(finally, last, etc)


2. but, yet, however, in fact, it is true, nonetheless, nevertheless, though, even though, etc.
3. because, therefore, thus, consequently, as, for, lead to, attribute to, contribute to, etc.
4. TPO22 cellulose , let me explain, I will explain, etc.
5. only, best, most, it isthat, etc.
6. tpo5-maize, tpo10-orcas
7. more than, less than, rather than, compare, contrast,
8. so, well, generally, etc.

TOPICThe causes of sea otter decline


(pollution hypothesis)

(predation hypothesis)

1. There were known sources of it along the Alaskan 1.No dead otter found -- first no dead
coast, such as oil rigs and other sources of industrial
chemical pollution. Water samples from the area
revealed the existence of increased levels of chemicals.

bodies of sea otters were found, which is not in accordance


with the infection from pollution.No dead otter consistent
with predation on the other hand more
consistent with the predator hypothesis. eaten
immediatelybecause sea otters could
have been eaten immediately by their predators.

2. Other sea mammals such as seals and sea lions along 2.Orcas hunt whales -- although although
the Alaskan coast were also declining, which indicates
that whatever had endangered the otters was affecting
other sea mammals as well, since it usually affects the
entire ecosystem rather than a single species.
Orcas

orcas may prefer to hunt whales. Whales disappeared by


humanbecause of whales have
disappeared because of human hunters. Orcas change
diet, hunt smaller mammals that means
orcas have had to change their diet, probably hunting
smaller mammals like sea otters. Orcas otter decline -so
So it probably is the orcas that caused the
decline of sea otters.

3. The pollution hypothesis could also explain the 3. uneven pattern better explained by predation --third
uneven pattern of otter decline: at some Alaskan
locations the otter populations declined greatly, while at
others they remained stable.

uneven pattern of otter decline is better


explained by the orcas predation theory.location access to
Orcas or notit depends
on whether the location is accessible to orcas or not.Orcas
access easily, otter decrease greatly--
Locations that orcas can access easily, the number of
sea otters has declined greatly. Orcas large, shallow and

rocky is inaccessible--However because


orcas are so large, they cannot access shallow or
rocky locations. shallow and rocky, otter not decline--
shallow and rocky locations are the places sea
otter have not declined.

Sample Answer(271words, )
The lecture casts doubt on the pollution hypothesis in the reading, stating that following investigations reveal that
predation is the most likely cause of sea otter decline. The speaker supports her stand with the following three points.
First, the reading passage shows that pollution hypothesis is supported by evidence, such as oil rigs and other industrial
chemical pollution wastes along the coast. However, the professor figures out that no dead bodies of sea otters were
found washed on shore, which is more consistent with the predator hypothesis because the sea otters could have been
eaten immediately by their predators and cannot be washed away along the shore.
Second, the reading material suggests that only environmental factors could have an overall effect on the entire ecosystem,
causing the widespread decline of other sea mammals, such as sea otters, etc. However, the professor asserts that whales
have virtually disappeared because of human activities, so the orcas began to change their diet from hunting whales to
feeding on other sea mammals for survival, thus triggering the overall decline of sea otters, seals and sea lions.
Finally, the reading passage claims that the pollution hypothesis can explain the uneven pattern of sea otter decline.
Nevertheless, the professor contends that the orca predation theory provides a better explanation to this phenomenon. It
is because the survival of sea otters depends on whether the place they live is accessible to orcas or not. The number of
sea otters keeps constant in shallow and rocky locations because these locations are evidently inaccessible to orcas, and
the number of sea otters declines greatly where orcas can access with ease.

Level 4 Level1-Level5
whales, orcas, sea otters

TPO10-

Do you agree or disagree with the following statement?


Playing computer games is a waste of time. Children should not be allowed to play them.


()

Thesis statement
Topic Sentence
N

TPO10 playing computer games


wasting time children
Thesis Statement: keep children away from computer games is neither wise nor sensible.
1. Computer games serve to develop players intelligence.
2. Mutual respect and understanding between parents and children is crucial.
3. Concession: Childrens indulgence in computer games leads to poor academic performance.
4. Conclusion: Parents should be inspiring, tolerant and supportive to children.

Sample Answer(466words, )
A great number of people are willing to vote for the fact that playing computer games is detrimental to childrens study,
physical health and personalities, and hence, they should be prohibited from playing computer games. As for me, the claim
that students should be kept away from computer games is groundless, for they fail to take positive effects into
consideration. In fact, keeping children away from computer games is neither wise nor sensible.
To begin with, computer games serve to develop players intelligence. In the past decades, people were regarded as
illiterates for the reason that they could not manage to read or write. With the emergence of information sweeping,
however, people who are unable to run computers are also labeled as illiterates. Parents always assume that playing
computer games is bound to deprive the children of their time to interact with the real world. Few would deny, absolutely,
that computer games and online games have become so prevalent that students who are pessimistic to them will feel
constrained communicating with their peers. Parents who do not want their children to be considered as eccentrics should
give a second thought when considering introducing a restriction to their children. Furthermore, playing computer games
does not only involve negative consequences. Still, there are a lot of computer games targeting at enhancing intelligence.
Obviously, playing computer games fosters skills for children to tackle with assorted problems or tough situations.
Additionally, parents should strive to keep up with the impressive strategies when dealing the relationship with their
children, namely largely mutual respect and understanding in terms of study and entertainment. Needless to say, children
have been exposed to various situations and they have increasingly become aware of the significance of being respected
even adored. Parents should not totally ban their children from playing games although the merits are far outweighed by
the demerits. Restriction is, to a large extent, never a good idea because it inevitably triggers rebellion. The most
impressive approach is that parents are supposed to cope with the issue of playing computer games based on the principle

that encompasses mutual respect and understanding. In order to maintain a harmonious relationship and solve problems
wisely, parents should learn to figure out how their children think and what they desire for, instead of imposing a lot of
baseless restrictions upon playing computer games.
It is true that engaging in computer games does bring about some negative effects if children focus overwhelmingly even
get indulged in them, causing their poor academic performance and failing to correctly interact with the real world.
Accordingly, parents should embrace supervision towards childrens time spending on computer games but not restricted
them to play games. For parents, being encouraging, tolerant, and supervised to their childrens occupation upon
computer games could make an all rounded individual.

()
Should children spend most time playing and studying, or they should be required to help the family with household
chores, like cooking and cleaning? 2013/12/18

Level3 Level1-Level5

TPO 10
TPO10-

TOPIC

-(Sea Otter)
1.

-
1.

2. 2.

3. 3. Orca
Orca

Orca

Sample Answer
The population of sea otter off Alaskan coast started to decline rapidly a few years ago. Two competing hypothesizes are
raised by experts: pollution and predation. The passage is convinced by pollution hypothesis, while the lecture explains
how the predation theory is more persuasive.
Though the pollution theory was supported in the passage by polluted water samples, which is owing to oil rigs and
chemical pollution resources. The lecture, however, points out a fact that no dead sea otters were found washed off the
Alaskan beaches. Therefore, they would not die of pollution but predators who consume their bodies after killing them.
Second, pollution theory is the only explanation to the declining population to other sea mammals such as seals and sea
lions as well as sea otters, according to the passage. However, the lecture discusses that orcas had to change their diet to
smaller mammals like sea otters due to the great reduction of sea otters amount, which, inevitably, causes the population
decline of sea otters.
Moreover, the phenomenon of uneven pattern of otter decline was explained two ways. The passage believes ocean
currents or other environmental factors causes uneven pollutions along the coast. However, the lecture argues that the
population of otters depends on whether their living places are accessible to orcas or not. Shallow and rocky locations,
where orcas are too big to enter, are the exact places otter population remains still. This supports the very hypothesis of
orca predation.

Level4 Level1-Level5

TPO 10-
Do you agree or disagree with the following statement? Playing computer games is a waste of time. Children
should not be allowed to play them. Use specific reasons and examples to support your answer.

-
1
2
3

Sample Answer
Computers are the origin of sin. numerous parents in China once said. For them, computers, despite the advantage of
making convenient communication, are more likely to exert adverse effect on children since computers are simply game
creators in their eyes. Thus, it is obvious that those parents will claim that spending time on computers, especially
computer games, is a waste of time. Nevertheless, as for me, the benefits of computer games outweigh the drawbacks.
One noticeable thing about computer games is that they provide children a chance to relax their mind and body. Different
from children in the past, todays youth suffer a lot from tedious homework, numerous tasks and social activities. As is
once reported in China Daily, a world renowned magazine on common topics and current affairs in China, 80% students
spend almost eight hours a day attending classes and finishing homework assignment, and the time increases as their age
and academic year. Under such pressure, it is undeniably that some funny games, like Angry Bird, help release their
pressure and allow a space for those poor kids to get rid of all the burden and worries. While exposing to the computer
games, childrens mind will be refreshed and they may be better at study later. On account that spending a few hours on
games brings about higher efficiency and more focus on work, it is worthwhile to give them freedom to play.
In addition, while some parents indicate that computer games are a cause of underdeveloped communication skills in
children, I disapprove of such a statement. For one thing, children who do not spend all of their after-school hours playing
computer games may not spend at least some of that time talking with people. For another, some games, such as Anatonio,
a foreign famous game, actually help build a virtual network online by drawing players all over the world to chat freely
through the chatting box on the screen. In this way, children who play the game get a chance to know foreign culture,
make foreign friends and even extend their outlook. Moreover, playing computer games could help to improve childrens
intelligence. Many games available today require players to have knowledge and deep thought so that they can
successfully accomplish each task step by step. Therefore, under such practice, computer games play an indispensable role
in childrens personal development by enhancing their ability to solve problems in the future and increasing their
communication skills.
Admittedly, the worry of the majority of parents is not limited to the prohibition of communication skills, but its negative

influence on academic performance. While students are too indulged in the games, they may even hold averse attitudes to
study. Several cases of school dropouts attribute to addiction to computer games. However, only if parents give enough
guidance and supervision to children by limiting the hours children spend on games can the disadvantages be minimized.
To sum up, despite the demerits of playing computer games, I still acclaim the advantages it provides to children, especially
in terms of helping relieving the pressure and improving certain necessary skills.

Do you agree or disagree with the following statement? Parents should spend time with their children helping them with
schoolwork rather than playing games and sports. Use specific reasons and examples to support your answer.
2013.11.9B
Do you agree or disagree with the following statement? Spending a lot of time watching sports on TV and following their
favorite teams will have negative impact on ones life. Use specific examples to support your answer.2014.5.11
Do you agree or disagree with the following statement? It is better to relax by watching a movie or reading a book than
doing physical exercise. Use specific examples to support your answer.2014.5.17

Level 2 Level1-Level5

TPO 10
TPO 10-

TOPICwhy sea otters decline

1. known sources of pollution--oil rigs and other


industrial chemical pollution

1. no one can really find any dead sea otters eaten


immediately by predators

2. other sea mammals are also declining;


The orca could have the same effect, but orcas prefer to
hunt much larger prey like whales.

2. Whales have disappeared because of human hunters.


Because only smaller sea mammals are now available, orcas
have had to change their diet to hunt these.

3. uneven pattern of otter decline because of the uneven


concentrations of pollutants.

3. In locations that orcas can access easily, the number of


sea otters has declined greatly.
In shallow or rocky locations, sea otter populations have not
declined, because orcas are large, and can't access these
locations.

Sample Answer
While the passage attributes the decline in the sea otter population to environmental pollution, the professor holds that
predation is a more convincing cause.
The first reason proposed in the reading passage is that there were known sources of pollution such as oil rigs and
industrial chemical pollution. However, the professor states that no dead otters can be found washed off Alaskan beaches.
This fact shows that environmental infection is not the cause of the decline in the sea otter population. Instead, otters
were more likely to be eaten by predators.
Second, the decrease in the populations of other sea mammals like seals is cited by the passage as support for the
pollution theory, because only pollution and orcas can affect the entire ecosystem, but orcas prefer larger preys like whales.
However, the professor maintains that whales are less available as a result of human hunting and orcas are forced to eat
smaller mammals like sea otters in order to survive.
Last, the professor points out that the pattern of otter decline further supports predation. As opposed to the different
concentration of pollutants caused by ocean currents in the passage, the professor argues that the number of otters in
shallow and rocky areas remains stable as the orca is large and cant access these areas, while those easily accessible areas
saw rapid decline. (223 words)
attribute A to B A B

level 4 Level1-Level5
otter, orcacurrents, concentration, uneven
shallow, rocky
predator orca

TPO10-
Playing computer games is a waste of time; children should not be allowed to play them

1. Analyzing the question


+

2. Defining Position
validate or invalidate the condition
Playing computer games is a waste of time

++ 1+ 2+

+ 1+ 2+

3. Brainstorming

4. Topic sentences ()
+/
4-1 There is no denying that if children are not self-disciplined enough, computer games can be addictive and
time-wasting.
4-2 The first major benefit is that, contrary to the stereotype, playing computer games is also a stimulating and interesting
way to learn.
4-3 Another way that computer games benefit children is that children can socialize and cooperate with other players.

5. Keyboarding ()
5-1 Introduction
Introduction=General Statement+Thesis Statement

5-1-1 Computer games are getting increasingly popular with both children and adults.
5-1-2 + Although some parents or teachers, worrying that children may waste too much time playing
these games, argue that children should not be allowed to play, I believe that children should be encouraged and guided to

play computer games as it is an important way for them to learn and socialize.
5-2 body paragraphs ()
5-2-1 1++ topic sentence 1+details
5-2-2 2++ topic sentence 2+details
2 3
the other side
5-3 Conclusion
+Reworded Thesis Statement+topic sentences

To sum up, playing computer games is not necessarily a waste of time. On the contrary, it has many positive influences on
children as it offers them a way of relaxation and socialization. Therefore, it is unnecessary and unreasonable not to allow
children to play computer games.

Sample Answer
Computer games are getting increasingly popular with both children and adults. Although some parents or teachers,
worrying that children may waste too much time playing these games, argue that children should not be allowed to play, I
believe that children should be encouraged and guided to play computer games as it is an important way for them to learn
and socialize.
There is no denying that if children are not self-disciplined enough, computer games can be addictive and time-wasting.
Playing some easy-to-difficult level games, for example, takes up a large amount of time and energy which can be better
spent on study or rest. However, as long as they are guided by parents or teachers and carefully choose the types of games
and set a time limit, computer games are beneficial to children in many ways.
The first major benefit is that, contrary to the stereotype, playing computer games is also a stimulating and interesting way
to learn. Children can gain knowledge and skills from playing those interactive and engaging computer games. As we know,
many games are designed to develop childrens intellectual abilities, such as Spelling Bees. Those involving history or math
help children to learn in a vivid and easy way. Compared with textbooks, these instructional and educational games make
learning process more enjoyable and enlightening. Of course, some computer games, with interesting stories, background
music and animation, are designed just to help players relax and entertain. Playing games enables children to take a
break away from their heavy school work and get refreshed when returning to study.
Another way that computer games benefit children is that children can socialize and cooperate with other players.
Building relationship and trust with other players is the first step to accomplishing a task. Children need to share roles and
coordinate and solve problems together. This is challenging because most of their communication happens online and
misunderstanding or disagreements can occur frequently. Teamwork and good communication skills, vital in those games
like Counter Strike, are also important qualities in childrens future study and life. It is common that skilled players of
computer games are also good at dealing with their personal relationships and ready for real-life challenges.
To sum up, playing computer games is not necessarily a waste of time. On the contrary, it has many positive influences on
children as it offers them a way of relaxation and socialization. Therefore, it is unnecessary and unreasonable not to allow
children to play computer games. (412 words)

socialize v.
There is no denying that

self-disciplined adj.
addictive adj.
as long as conj.
set a time limit
be beneficial to benefit n. v.
stereotype n.
stimulating adj.
interactive adj.
engaging adj.
intellectual abilities
vivid adj.
instructional adj.
enlightening adj.
animation n.
be designed to
enable sb. to do
refreshed adj.
accomplish v.
vital adj.
not necessarily

If parents are too busy to accompany their kids, do you think the kids should spend more time playing games or sports or
they should do more work related to school? (2013/11/09)
(A/D)Games are as important for adults as they are for children.

Level 3 Level1-Level5

TPO 11
TPO11-

TOPIC

The reading public is missing out on important benefits.

Intellectually stimulating: have to be literature;


Science writing, history, political analysis etc.: high quality,
creative & well-written, stimulate the imagination;
reading literature reading a good book.

Diverting time previously spent in reading literature to


trivial forms of entertainment has lowered the level of
culture in general.

The writing as well as the reading of literature is likely to


decline because of the poor standards of todays readers.

2. Culturally valuable material that isnt written: music &


movies;
Many forms of expression;
More directly to the contemporary concerns.

dont be too quick to blame the readers;


authors faults;

modern literature: difficult to understand.

Sample Answer
The lecture completely refutes the reading which presents unfortunate effects caused by the facts that people spend less
time reading literature.
To begin with, the author in the reading material claims that by reading less literature, the reading public is missing out on
important benefits brought by literature, whereas the professor counters that argument by asserting that a book doesnt
have to be literature to be intellectually stimulating. Other kinds of books, for example, science writing, history and
political analysis, can be just as creative and well-written as literature. They can also stimulate the imagination. So we
cannot be certain that someone who is not reading literature is not reading a good book.
Besides, in contrast to the reading, which expresses that diverting time previously spent in reading literature to trivial
forms of entertainment has lowered the level of culture in general, the lecturer insists that spending less time with books
in general does not necessarily mean the decline of culture for there are other valuable materials that are not written,
such as music and movies. Listening to a brilliant song or watching a good movie is not a waste of time. Also, these
non-literary activities do not lower cultural standards. In todays culture, there are many forms of expression besides
novels and poems, some of which speak even more directly to contemporary concerns than literature does.

Finally, the reading states that the poor standards of todays readers lead to the decline of the writing as well as the
reading of literature to some extent; on the contrary, the lecturer states that this decline is supposed to be attributed to
the authors. Some modern literature is intended to be difficult to understand. Hence, even earlier generation of readers
would hardly have read a lot of todays literature.

Level 2 Level1-Level5
TPO11-

Some people say that the Internet provides people with a lot of valuable information. Others think access to much
information creates problems.
Which view do you agree with? Use specific reasons and examples to support your answer.

7
8 Agree
5

6
9 Disagree
5
6

:
Conflicting ViewsBoth the positive and negative influence on childrens creativity exists. However, the
development of technology still enhances the creative minds of children.
technology reluctant to think diligently e.g.: the invention of calculators
1Not all the information is trustworthy. +
2Internet also spreads radical ideas and serving as a manipulating tool. +

Sample Answer
The twentieth century witnesses the birth of two great inventions: the mobile phone and the internet. The later
completely changes our relationship with intelligence, providing us with massive information which could hardly be
perceived. Every second, billions of bits of information are flying to every corner of the world through optical fibers,
connecting everyone with the rest of the world. One who has access to the internet is connected to the rest of the world
recorded by digital technology. Even though some embrace internet, believing it provides people with valuable information,
while others, including me, show concerns about the problems that the rapidly-spread information creates.
True, the internet contains more information than any single library in the world, which is beyond doubt as nowadays even
libraries have been digitalized so that readers can approach to books as long as the internet is accessible. Indeed, internet
is the place where you can find useful knowledge. From Wikipidia one can learn from the construction of the wing of a fly
to the medieval view point toward the universe. Searching through Taskrabbit, people looking for an errand to earn money
or someone to run an errand for them would never be disappointed. Meanwhile, the Ted allows tons of new ideas to flood
in and the only problem is which one to click open first. Yes, the internet contributes to the spread of valuable information.
However, accessing to too much information is indeed a problem, especially when not all the information is trustworthy.
Internet is the right place to find valuable information, but it is also the den and media for false science and all kinds of
wickedness. Some of them are harmless sensation which cause not big crisis but a laughter, while some may mislead one
who is not good at sieving and discerning information. When it comes to the structure of the solar system, students who
search it on the internet may be overwhelmed with hundreds of illustrations of the solar system, some of which depicted
the orbits of planets as ellipse. However, this is far from the truth, as their orbits are approximately circles. As Thomas
Cardinal Worlsey said, be very careful what you put into that head, because you will never, ever get it out. The confusing
status caused by too much information may look unimportant at the first glance, but the power of accumulating mistakes
and misconceptions is not one to be ignored.
Besides, people might fall victim to bad or incorrect information and act on it. Since internet is also the medium for really
bad information, spreading radical ideas and serving as a manipulating tool. For instance, terrorists can publicize their
ideas and so called projects through the internet and the easy access to internet make the majority susceptible to their
inhumane notion. These days, may suicide bombers are recruited through the internet and the number increase by a
staggering ten percent the year when the optical fiber becomes prevailed. Without critical thinking, one might gradually
lose the ability to distinguish right from wrong.
Internet, like most technology, is a double edged sword. It may serve for our good by providing valuable information, or
hurt us when we fail to prevent the danger hidden in excess intelligence.

2015/04/12 Internet is as important as other service such as building roads that government should make Internet
access to all the citizens at no cost.

2010/09/26 Technology designed to make our life simpler actually makes our life more complicated.

Level 2 Level1-Level5

TPO

TPO 11
TPO11-

TOPICThe influence of decreasing trend on book reading for young people

1.By reading less literature, the reading public is missing


out on important benefits.

1.Literatureintellectually stimulating
other subject with high quality and creative and
well-written

2. Diverting time previously spent in reading literature to


trivial forms of entertainment has lowered the level of
culture in general.

2. not the culture decline.


Music and movies are not written but people are not
wasting their time on them.

3. The writing as well as the reading of literature is likely


to decline because of the poor standards of todays
readers.

3.A lot of modern literature is intended to be difficult to


understand.

Sample Answer
In the reading passage, the writer claims that the decreasing trend on book reading for young people has unfortunate
effects for the reading public, for culture in general, and for the future of literature itself. However, the speaker denies the
idea made in the reading that reading less literature doesnt mean the bad effects for the culture.
First of all, the reading passage contends that by reading less literature, the reading public is missing out on important
benefits because nothing else provides the intellectual stimulation that literature does but the lecturer argues that it
doesnt have to be literature to be intellectually stimulating and some books with other subjects also have high quality and
are creative and well-written as a novel or a play. Not just the literature can stimulate the imagination.
Second, the author states that diverting time previously spent in reading literature to trivial forms of entertainment
has lowered the level of culture in general. The speaker says, however, that there are so many other valuable materials
that arent written like music and movies. We cant say that we waste time listening to a brilliant song or watching a good
movie. Therefore, the non-literature doesnt lowered the level of culture.
Lastly, in the reading passage, the author believes that the writing as well as the reading of literature is likely to
decline because of the poor standards of todays readers. On the other hand, the professor in the listening argues that a
lot of modern literature is intended to be difficult to understand so we cant just blame the readers.
In conclusion, we cant say that reading less literature can lead unfortunate effects for the culture.

The reading passage contends that but the lecturer argues that.

According to the professor in the lecture,.This directly contradicts what the reading passage indicates, because.

Level1-Level5
Level 3

TPO11-

Some people say that the Internet provides people with a lot of valuable information. Others think access to much
information creates problems. Which view do you agree with?
Use specific reasons and examples to support your answer.

agree/disagree

1agree/disagree
2
Sample

support

support

Sample Answer
When it comes to internet nowadays, different people have different opinions. Some people prefer to say that the
Internet provides people with a lot of valuable information. However, some other people would like to say that access to
much information creates problems.
I concede that Internet can provide people so much information and all you should do is to sit in front of the
computer and click your mouse. With the Internet, people can get the fresh news and whats happening all around the
world at any time and any place. It compares with the time when we dont have Internet and at that time the way people
can get the fresh new is very limited like watching the TV news or reading the newspaper. All of these have the time lag
and we cant get the news when it comes out. Furthermore, with the internet people can connect with others easily.
People can chat with others on the chat software, email and other social networking services and share what they see or
what they think with others. Its a good way for people to be connected since not everyone will meet in the daily life.
In addition, with the development of the Internet and computer, people whose major are computers are easier to find a
well-paid job by themselves. But several years ago, without the funding from family to buy a computer people cant study
computer in the university.
However, some conflicts come out. Access to much information creates problems. With internet, people waste too
much time on it even their rest time. Furthermore, much information on the internet will distract people from their work

or study since people always want to get the fresh new information at the first time and they cant help to refreshing their
social networking services. Because of the much information, we cant guarantee the quality of the information. Some
rumor will spread all around the society and cause panic that make people feel less sense of safety for the government or
other people. Whats more, the rumor spreading around is especially harmful for young people since their outlook on life
has not developed they dont have the ability of judgment for the good information and rumor. In this case, it will
influence their growing. Nowadays, so many people abuse other people on the internet for no reason and if young people
also do the same thing it will be harmful for them.
Based on the statement above, as the internet brings people the fresh information and convenience it also comes out
so many problems. So when we use the internet for our daily life, we should also notice the way it used.

2011/6/25 Some people prefer to buy technological devices as soon as they are available to the public, while other people
prefer to wait. Which do you prefer?
2014/11/29 Some people like to keep a record of their own experience by uploading pictures and other information to
social-networking sites. Other people prefer not to create such records. Which do you prefer and why.

Level1-Level5
Level 4

TPO 11
TPO11-

TOPIC

1.

1.

2.

2.

3.

3.

Sample Answer
The reading passage uses three reasons to argue that a decline in literature reading brings along seriously negative effects.
However, the professor in the lecture doesnt think the reasons hold water.
First, the reading says that a sharp decline in reading literature means less opportunities to exercise intelligence and
imagination. However, the professor believes that books in science, history and politics are also written in high quality and
serve the same purpose to stimulate imagination and develop intelligence.
Second, contrary to the statement in the reading that the cultural level lowers due to less literature reading, the professor
says that culture has changed; other forms of materials like movies and music speak more than literature and has actually
promoted cultural prosperity.
Finally, the professor claims that the writer unfairly blames the taste of today's readers for the decline in literature reading.
According to the professor, it is actually because some of nowadays literature are written in an excessively complicated way.
Even readers from ealier periods cannot tolerate the convoluted style either.

The reading passage uses three reasons to argue that ... However, the professor in the lecture doesnt think the reasons
hold water.
First, the reading says that ... However, the professor believes that
Second, contrary to the statement in the reading that ... the professor says that
Finally, the professor claims that the writer unfairly , According to the professor,

Level 2

TPO11-

Do you agree or disagree with the following statement?


Some people say that the Internet provides people with a lot of valuable information. Others think access to so much
information creates problems. Which view do you agree with?

Use specific reasons and examples to support your answer.

access to Internet information has brought as many annoying problems


1
internet is full of unorganized -or even false- information
2
Too much information will distract us
3
there are many unhealthy information on the internet

Sample Answer
[345 words]
The Internet, with its marvelous storage and wide bombastic spread of information, has raised controversy. For computer
buffs this is definitely a great incentive. Others, however, will drown in an overload of information. In my view, access to
Internet information has brought as many annoying problems as any possible benefits. It is really hard to imagine a
technology that has more negative impact on modern life than the internet.
To begin with, internet is full of unorganized -or even false- information. We often get lost in oceans of data that contains
no useful information. According to research, people are now spending much more time searching online while the data
found is fragmentary, unreliable or even mutually contradictory. We have to go through a time-consuming process to sort
out useful information from the vast useless or false data we receive searching the internet.
Whats more? Too much information will distract us. We often waste time unconsciously by surfing the internet aimlessly.
It is common that we listen to music, click the pop-up news or chat with friends online while working or studying. This
slows us down dramatically, and we lose concentration easily.
In addition, the internet is filled with violence and obscenity. I believe this is morally repugnant. Internet users, especially
youngsters, play violent role games in online game settings. Killing monsters or other role players mercilessly. In the real
world this can lead to serious trouble. Although they cannot do the similar things, they may be influenced by such violence.
For example, researches have shown that youngsters who are frequently exposed to violent movies are more inclined to
hurt others physically. Also, numerous news sources reported rape, because similar themes where found on the internet.
To sum up, the internet has exerts negative impact on our lives. For one thing, it wastes our time with its enormous
ill-organized data, preventing us from being concentrated and efficient. For another thing, adult contents are not always
filtered, which have a bad influence on young people. Thus, when having a blast surfing the Internet, we must be heedful
of possible hurricanes.


CN 2014/11/9
Do you agree or disagree with the statement: When making major purchase (for example, car or laptop), our decisions can
be influenced by different sources of information. Explain how each of the following sources of information can influence
your decision.
(1) recommendations from friends or colleagues
(2) information from media (for example, TV, magazines, newspapers)
(3) recommendations from sales person in the store
CN 2010/5/22
Do you agree or disagree with the following statement? Government should put more money on Internet access rather
than pubic transportation.

Level1-Level5
Level 4

TPO 12
TPO12-
Jane Austen
TOPIC Jane Austen

In 1882, Austens family gave permission to use the


portrait as an illustration in an edition of her letters.
1882 Jane Austen

The face in the portrait clearly resembles the one in


Cassandras sketch, which we know depicts Austen.
Cassandra Jane Austen

Humphrey was active in the late 1780s and early 1790s,


exactly the period when Jane Austen was the age of the
girl in the painting. Humphrey
Jane Austen

: not convincing
When authorized, JA: dead for 70 years;
never actually seen her themselves;
know for certain.
JA
2. A relative of Austens;
Austens family: large;
Many female cousins: teenagers & have children who were
teenagers;
Some resembled JA;
Experts: Marianne Kempian, JAs niece.
JA
Only style;
Other evidence a later date;
Stamp the blank canvas was sold by William Legg;
WL sell canvases in London when JA was a teenager;
WL began to sell when JA was 27.
JA was older.
JA

Sample Answer
The lecture completely refutes the reading which presents unfortunate effects caused by the facts that people spend less
time reading literature. In the lecture, the professor argues that it's not convincing to regard the subject of the sketch as
Jane Austen.
To begin with, even though the family has approved to publish the portrait, as proposed in the reading, the professor
counters that argument by asserting that it was 70 years after Jane Austen's death. Since the family members had never
actually seen her themselves, they were unable to assert the fact.
In addition, in contrast to the belief in the reading that the face in the portrait resembles the one in Cassandra's sketch,
which depicts Austen, the professor claims that the face might be one Jane's relatives of the similar age. The Austen family
was pretty large, so there are possibilities that some of the teenage girls in the relevant period, who were Austens
relatives, resembled Jane Austen.

Finally, the author attributes the portrait to Humphrey because of the style, which links to the exact time when Jane
Austen was a teenager; on the contrary, the lecturer argues that the stamp on the back of picture suggests that the canvas
was sold by William Legg, who didnt start selling canvases until Austen was 27 years old. Hence, Jane Austen was older
than the girl in the portrait at that time.

Level 4 Level1-Level5
TPO

TPO12-

Do you agree or disagree with the following statement?


It is better to have broad knowledge of many academic subjects than to specialize in one specific subject.
Use specific reasons and examples to support your answer.

10

11

Agree
7
8

12

Disagree
7
8

:
Background informationstudents need to choose one major so that they can be the experts in that field.
1As ones time and energy is highly restricted, it would be less efficient if he distributes these finite resources into
several fields of study. +
2People with broad knowledge could hardly outcompete experts in any specific subject they learn. This makes them
less desirable for the employers. +

Sample Answer
An LAC, Liberal Art College, always considers the breadth of knowledge important, but it still requires students to select
certain major in their sophomore. As for most universities, students are doomed to stick to one major so that they can be
the experts in that field. We can perceive from these cases that educational institutions generally highlight the merit of
specialization.
First of all, as ones time and energy is highly restricted, it would be less efficient if he distributes these finite resources into
several fields of study. To allow knowledge to fully exert its power, we need depth. Even if one chooses to learn many
academic subjects, his work will end up fruitless if he simply dabbles in each field. Such consequence is very likely
especially when a student approaches his academic study by less efficient method. Consequently, studying many academic
subjects is highly possible the pathway to nothingness. On the contrary, when one puts all the potential to one subject,
which is the case of specialization, it would be generally easier to study efficiently and gain great depth of knowledge. This
point could be supported by a contrast between college students with single major and their counterparts with double
major. According to a statistic study done by University of Ohio, students majoring in one subjects generally do a better job
than that of students having double major. Dividing ones resources into two could render such an obvious result, let alone
into several.
Besides, people with broad knowledge could hardly outcompete experts in any specific subject they learn. This makes
them less desirable for the employers. Specialization is exactly the result of fierce competition. While the majority shares
average skills in all sorts of fields, one exceptionally skillful in one field will definitely boom in it. Armed with specialty in a
specific field, employees can concentrate more on this task, leading to a high quality and efficiency. It is the philosophy
behind the emphasis of specialization these days. Those expecting to fit in various kind of occupation will end up finding
no good jobs as long as experts in these fields still exist. As an old ballad of England said, the carpenters are well-off, the
mansions are well-off, but those knowing only how to plane and how to carve are doomed to be starving.
To sum up, in order to pursue the efficiency and expertise, to specialize in one subject is better than to obtain a broad
knowledge of many kind of academic subjects.

2006/12/08

Level 3 Level1-Level5

TPO 12
TPO12-

TOPIC

1.

1.

2.

2.

3. 3.

Sample Answer
In terms of whether the painting depicts Jane Austen, the reading and the lecturer hold contradictory opinions. The
reading believes that several evidences prove it is indeed a picture of Jane Austen, however, the lecturer contends that
none of the evidence is convincing.
First, the reading states that the subject of the painting is Jane Austen because Austen\'s family recognized Jane Austen in
the picture and allowed it to be published as her portrait. By contrast, the lecturer argues that when the picture was first
published, Jane Austen has been dead for 70 years. Even her families who claimed to recognize the portrait did not see her
themselves, so they could have mixed up.
Second, contrary to the idea in the reading that the face in the portrait looks like Jane Austen in Cassandra's sketch, the
lecturer suggests the girl in this portrait could be one of Austen\'s relatives. Austen had a large extended family and
actually the girl could be one of a distant niece of Austen.
Finally, the lecturer refutes the opinion in the reading that the girl was Austen because the portrait's style was linked to a
painter who was active when Jane Austen was young. According to the lecturer, the canvas used in the portrait was sold by
a businessman who did not come to London until Austen was 27 years old, so Austen at a young age could not have been
painted on such a type of canvas.

In terms of whether , the reading and the lecturer hold contradictory opinions. The reading believes that , however,
the lecturer contends that none of the evidence is convincing.
First, the reading states that . By contrast, the lecturer argues that .
Second, contrary to the idea in the reading that , the lecturer suggests
Finally, the lecturer refutes the opinion in the reading that... According to the lecturer,

Level 4

TPO12-

Do you agree or disagree with the following statement?


It's better to have a broad knowledge of many academic subjects than to specialize in one specific subject.

Use specific reasons and examples to support your answer.

it's better to have broad knowledge of a variety of subjects

a wide range of knowledge enables you to become well-rounded and open-minded people

many subjects are so closely related to each other

3
a focus of field still involves several subjects

Sample Answer
[578 words]
There is a heated debate about whether to learn a lot of subjects, and become a generalist, or to focus in one subject and
become a specialist. Some people argue that specialization enables one to be an expert in a certain field. This helps finding
a better job. Those who advocate having a broad knowledge, on the other hand, argue that society needs talented people
with background of diverse subjects. From my perspective, it's better to have broad knowledge of a variety of subjects.
Here is my argumentation for becoming a generalist:
First and foremost, a wide range of knowledge enables you to become well-rounded and open-minded people. In terms of
open-mindedness, it is necessary to be receptive to brilliant ideas and experience of other fields. If we narrow ourselves
too tightly in only one subject, having no desire to know anything outside our major field of expertise, we will fail to see
merits in others' standing. And thus cannot learn from them. In terms of well-roundedness, varied knowledge helps us
perceive beauty of many other facades of the academic world. Otherwise, we may jump into and sit on a deep well that
we dig by ourselves, staring at the sky of the size of a well and unaware of the outside world.
In addition, many subjects are so closely related to each other; it's simply impossible to learn about one and ignore the
other. I can't imagine a physicist with no understanding of mathematics, or a mathematician who never applied math in
other fields of life. Likewise, subjects as history and philosophy have a far-reaching function for all people, not only
historians or philosophers. For example, in order to thoroughly understand a literature work, one needs to analyze it in its
historic context and associate it with mainstream outlooks at that time. In fact: many today's major academic
breakthrough take place in interdisciplinary areas.
Although learning many different subjects has many merits, we should not go to extremes. In other words, in the process
of pursuing diverse knowledge, there is no reason to refuse to focus on a specific area. After all, we should avoid becoming

a "Jack of all trades and a master of none. However, a focus of field still involves several subjects. A student with a major
in economics, for instance, would typically be required to take courses such as microeconomics and macroeconomics
among others. Meanwhile, courses such as industrial economics, environmental economics and labor economics, may be
offered on an optional basis. These seemingly unnecessary courses actually allows students to develop a broad
understanding of their study, so they're able to prepare themselves even better for future employment and further
research.
In the final analysis, considering the inextricability among different subjects and obvious benefits of employing a wide
range of knowledge, we can safely draw the conclusion that it is better to learn a variety of subjects.

CN 2013/10/20
it is always better to purchase in large stores than in several small shops that specialize in different items
CN 2006/12/8
It is better to learn broad subjects or specialize a special subject.

Level1-Level5Level 4

TPO 13
TPO13-

TOPICcommercial fossil trade

1. the public will have less chance to appreciate these


precious fossils due to this commercial fossil trade

1. this trading trend could give people greater exposure to


those fossils. even low level public institutions, such as
public schools and libraries, could buy fossils and display
them when they are available for purchase.

2. the scientific community will miss out important


scientific discoveries because of these fossil trades

2. fossils involved must be identified by scientists before


entering the market. even though fossils are destined to a
private collector, scientists should be the first person to
examine them.

3. the commercial fossil trade may ruin important


scientific discoveries associated with fossils

3. even though some evidence may be destroyed, more


fossils can be found. And its a better result for scientist to
have more fossils found rather to have many fossils go
undiscovered.

Sample Answer -1
The lecture refutes the reading passages idea that selling fossils in the commercial market can cause negative
consequences. The professor argues that the benefits of commercial fossil trade outweigh the disadvantages.
First, the lecture disagrees with the readings point that the public will have less chance to appreciate these precious fossils
due to this commercial fossil trade. Actually, this trading trend could give people greater exposure to those fossils. The
professor states that even low level public institutions, such as public schools and libraries, could buy fossils and display
them when they are available for purchase.
Second, the lecture challenges the texts statement that the scientific community will miss out important scientific
discoveries because of these fossil trades. The truth is that fossils involved must be identified by scientists before entering
the market. The professor stresses that even though fossils are destined to a private collector, scientists should be the first
person to examine them.
Third, the listening passage challenges the final drawback of fossil trade mentioned in the reading part. According to the
reading, the commercial fossil trade may ruin important scientific discoveries associated with fossils. On the contrary, the
professor points that even though some evidence may be destroyed, more fossils can be found. And its a better result for
scientist to have more fossils found rather to have many fossils go undiscovered.

Sample Answer -2
The lecturer rejects the ideas presented in the reading passage about the problems involved with the booming business of
commercial fossil trading. In her opinion, the benefits of this new development outweigh its negative consequences.
The lecturer does not agree with the first point made in the reading-that private collectors will keep their fossil collection
away from the public. She contends that the commercial trading of fossils actually makes them available to a wider public,
because everyone with a budget, such as private schools and libraries, can purchase them for study and exhibition.
The lecturer goes on to refute the claim in the reading that fossil trading business prevents scientists and public museums
from benefiting from fossil finds, as these parties cannot compete with wealthy private buyers to acquire important fossils
for research purpose. On the contrary, she argues, scientists themselves are the first to evaluate any important fossil
before it can be sold in the commercial market at a price, so the academic community does not miss any opportunity to
study privately traded fossils.
Additionally, the lecturer challenges the final downside of fossil trading mentioned in the reading. The lecturer reminds us
that the damages, if any, caused by private collectors in their field operations are more than offset by the effort they have
made to increase the number of fossils available to the public that would otherwise remain undiscovered.

Level 3 Level1-Level5

TPO13-
The extended family (grandparents, cousins, aunts, and uncles) is less important now than it was in the past.

//

1. we no longer live with the extended family members;


2. we do not need family members as much as we did in the past due to advanced technology;
3. Everyone is so busy with his own stuff and has no time for extended family members.

Sample Answer-1
In the past, people made great use of their extended families to help improve their lives. However, this is not the case
anymore as extended families are clearly less important now than they used to be.
These days, it is much easier for a person to live alone and not rely upon any family member for help. In the past, extended
family member were crucial to peoples survival since most of them lived on farms and needed much physical labor to
cultivate the land. But this is not the case nowadays. For example, my uncle lives alone in a big city and works for a bank.

Most of his work involves dealing with figures, reports, and records. He gets paid enough to make a living. He does not
require any of his family members assistance to survive. This is tremendous change from times in the past.
Furthermore, while people often relied upon extended family members to take care of various household chores in the
past, this is simply not the case anymore. Instead, people can employ various services to take care of all their needs. In the
past, a grandmother or aunt might have helped to take care of the children in a family. However, parents now merely send
their children to daycare centers. This is what my parents did for us when my brother and I were growing up. Duties and
chores once done by extended family members are being replaced by people in paid positions.
Finally, most extended family members live too far away from one another to be of any use both in general and in times of
need. A person who is not around cannot have any influence on the other members of his or her family. While my
grandparents are wonderful, loving people, they live several hours away from us. We typically only see them on holidays or
occasions like weddings or funerals. They have virtually no influence on me or anyone in my family. While this was not the
case in the past, it is definitely true for many people today.
In conclusion, because people can survive living alone, can employ others to do important chores, and do not live close to
many extended family members. It is clear that extended family members are not nearly as important as they once were.
Additionally, I find it likely that this trend will continue and that extended family members influences will continue to
decrease.

Sample Answer-2
Numerous claims have been made that the extended family is not as important now as it was in the past. I, however,
disagree with this contention and instead believe that it is just as important to people nowadays.
For one, in my country, extended family members still form a tightknit social group that is tremendously important.
Many people do not make crucial decisions in their lives until consulting with various members of their extended families.
For example, my extended family often gets together several times a year, especially on holidays. During these times, each
family member talks to the others about what is going on in his or her life. The grandparents, aunts, uncles, and even
cousins then offer their opinions and give advice on what they think each person should do. Assistance like this has helped
extended families retain their importance.
In addition, many extended family members live with other family members, playing important roles at home. Rather than
merely taking up space, these extended family members become vital family members. My grandmother lives with my
family. She provides invaluable help to everyone. She helps my mother with the cooking, especially on days when she is
tired from her job. My grandmother also helps clean the house and raise my younger sister. Without her, our home would
be chaotic.
Last of all, extended family members have extensive connections, so they can provide crucial help to others in times of
need. When an extended family is big, it provides ample opportunity for its members to take advantage of his network. In
my country, personal connections are incredibly important in all facets of society. One of my cousins just got hired because
my father knew his employer and recommended for him for his job. Without that personal connection, my cousin would
probably still be seeking employment in what is currently a tight job market. Thanks to extensive networking, extended
family members can prove their value time and time again.
I feel that extended families are still of great importance these days thanks to their ability to provide good advice, to help
take care of other family members, and to provide a network for others. In many countries, like my own, people can use

their extensive families to get ahead in their lives.

2006/06/09
2007/09/22

Level 4 Level1-Level5

TPO 14
TPO14-

TOPICsalvage logging

1. salvage logging could clear dead trees and give


necessary spaces for new trees

1. the removal of dead trees will not create a better


condition for the new trees. the trees natural
decomposition process will enrich the soil and provide a
more suitable environment for new trees

2. rotten wood will be harmful for the forest; dead trees


could hinder other healthy trees growth by providing
insects such as the spruce bark beetle with living
environment

2.the spruce bark beetle has lived in Alaska forest for really
long time without causing any damage. And this dead wood
could be used by other birds and insects that are important
contributors to a healthy forest.

3. salvage logging will gain great economic benefits

3. the economic influence is small and not long- lasting.


high-cost vehicles like helicopters have to be employed if
people want to recover these lumbers; also, wood recovery
jobs will be given to skilled outsiders rather than the local
people.

Sample Answer -1
The lecture casts doubt on the reading passages idea that salvage logging benefits not only the forest but also the
economy. In fact, this method could lead to a longer term environmental damage and leave the economic benefits
questionable.
First, the statement of the reading passage that salvage logging could clear dead trees and give necessary spaces for new
trees is negated by the lecture. Actually, the move mentioned in the text will not create a better condition for the new
trees.The professor points out the trees natural decomposition process will enrich the soil and provide a more suitable
environment for new trees , but the rapid sweep of dead trees will leave the soil in a meager condition.
Second, the point of the reading part that rotten wood will be harmful for the forest is flatly ridiculed by the lecture.
According to the reading passage, dead trees could hinder other healthy trees growth by providing insects such as the
spruce bark beetle with living environment. But the professor claims that this insect has lived in Alaska forest for really
long time without causing any damage. And this dead wood could be used by other birds and insects that are important
contributors to a healthy forest.
Finally, the conclusion of the reading material that salvage logging will gain great economic benefits is rejected by the
speech. In fact, the economic influence is small and not long- lasting. The speaker presents two reasons: high-cost vehicles
like helicopters have to be employed if people want to recover these lumbers; also, wood recovery jobs will be given to
skilled outsiders rather than the local people.

Sample Answer -2
The lecturer and the reading passage hold completely different views toward the practice of salvage logging, which is the
removal of dead trees from a forest after a fire or a storm.
First, the lecturer states that removing dead trees is not good for the health of a forest because it deprives it of the
nutrients necessary for future tree growth, which dead trees release into the soil as they decompose. In contrast, the
reading passage states that removing old trees provides more space in which new generations of fresh trees can grow
more healthily.
Second, the lecturer argues that some insects and birds that inhabit dead trees are beneficial for tree growth, so salvage
logging may cause unwanted damage to the forest in the long run. The reading, however, points out that some of these
insects are harmful to trees; thus, because clearing the forest of dead trees also destroys the habitat of these harmful
insects, it ensures the healthy recovery of a forest after a fire.
Finally, with regard to economic impact, the lecturer argues against salvage logging because the dead trees can only be
salvaged at a very high cost, and the employment associated with salvage logging is often temporary and taken by
non-native residents. On the other hand, the reading argues that this practice provides many industries with the wood
necessary to sustain their production and offer employment opportunities to local people.

Level 3 Level1-Level5

TPO14-
People benefit more from traveling in their own country than from traveling to foreign countries.

//

1. Traveling in a foreign country can broaden people's horizons.


2. Traveling in a foreign country provides opportunity to make friends with people with different backgrounds.
3. Traveling in a foreign country stimulate people to set new goals.

Sample Answer
Traveling can expand one's outlook, amplify one's experience and make many new friends. It could be a superb lesson in
one's life. If I possess adequate time and money to plan my tour, I would prefer to tour foreign countries.
First and foremost, foreign life can serve various life experiences and lifestyles which are hardly acquired in domestic life.
In the identical social and environmental circumstances, people generally have relatively familiar and similar views and
experiences. The distinction is, if any, slight. If people do not go abroad, they can rarely obtain refreshing ideas. For
example, some people who come from highly industrialized countries with high welfare, tend to take the free health care
and education for granted, but if they have a foreign tour in impoverished countries or regions where the food is almost far

from enough, not to mention schooling, they can be taught a precious lesson: cherish one's own life and help someone
who needs help.
Secondly, oversea traveling can offer broader choices. As far as I know, some countries are as small as one common city of
the United States. In such countries, if people refuse to journey abroad then they have to repeat his footprints time and
time again. on the other hand, some countries are located on some lonely islands without forests and all the fish comes
from the sea, if these residents want to ride a horse in the plain or taste freshwater fish, they have to travel abroad.
In addition, the exotic landscapes and customs are more curious and attractive than familiar ones. Generally speaking,
people are curious, they prefer to acquire purely refreshing experiences, rather than relatively familiar or even well-known
ones. The foreign lifestyle, food and scenery are always far different from one's own domestic experiences, which is the
key factor in satisfying people's curiosity more deeply.
In conclusion, foreign tour can afford more various life experiences, more options, and satisfy people's curiosity. These
advantages appeal to us. Needless to say, these are the contributing factors that attract more and more people to travel
abroad.

2008/10/17
2015/06/13

Spending money on traveling and vacation is better than saving money for future

Level 3 Level1-Level5

TPO 15
TPO15-

TOPICstop the spread of cane toad

1. a national fence should be built

1. young toads and toad eggs are found in rivers and


streams that would carry them to the other side, because
no matter where the fence is built, there will always be
waterways flowing from one side to another. As a result, the
population will continue to grow.

2. Volunteers could trap and destroy cane toads

2. untrained volunteers tend to inadvertently destroy


endangered frogs since it is hard to distinguish them when
they are young.

3. develop a disease-causing virus

3. reptiles and amphibians infected by the virus would be


transported to Americas where the local cane toad is a vital
part of the ecosystem. Once the virus attacks them, the
local ecosystem would suffer.

Sample Answer -1
The reading and listening materials have a conflict of opinions about how to control the population of cane toads. The
writer puts forward three solutions to solve the problem, which are contradicted by the following lecture.
Firstly, the author suggests that a national fence should be built. However, the lecture opposes this suggestion by saying
that young toads and toad eggs are found in rivers and streams that would carry them to the other side, because no
matter where the fence is built, there will always be waterways flowing from one side to another. As a result, the
population will continue to grow.
Secondly, the passage proposes that volunteers trap and destroy cane toads. However, the speaker views this issue from
an opposite angle. According to her, untrained volunteers tend to inadvertently destroy endangered frogs since it is hard to
distinguish them when they are young.
The third measure proposed by the writer is to develop a disease-causing virus. By contrast, in accordance with the
professor, this view does not hold water. It is because reptiles and amphibians infected by the virus would be transported
to Americas where the local cane toad is a vital part of the ecosystem. Once the virus attacks them, the local ecosystem
would suffer.

Sample Answer -2
The lecturer argues against the three measures mentioned in the reading passage to reduce the population of the cane
toad, a species introduced to the Australian continent.
The lecturer argues that the first measure, a national fence, would not prevent the flow of streams or rivers and, therefore,
would allow young toads or toad eggs to travel to the other side of the fence. The reading passage, however, argues that
such a fence would effectively cut off the route that animals use to establish colonies and expand in population.
Regarding the second measure, recruiting a large group of volunteers, the lecturer explains that volunteers often have
difficulty distinguishing between cane toads and native frogs, an endangered species; therefore, they might kill members
of both species. The reading passage gives the opposite view: Organizing a large group of volunteers to join an
extermination campaign would speed the destruction of cane toads.
Finally, the lecturer objects to the third measureusing an infectious virus. She points out that a virus intended to
eliminate Australias cane toad population could be transmitted through animal transportation to other continents where
cane toads are an essential part of the ecosystem. This is in direct contradiction with the claim in the reading passage that
an infectious virus could be developed to stop the reproduction of cane toads without harming other species.

Level 3 Level1-Level5

TPO15-
In order to become financially responsible adults, children should learn to manage their own money at young
age.

1. learning to manage money makes a child know how precious the money is.
2. learning to manage money teaches children how to increase their fortune.
3. learning how to manage money can also help children to manage other aspects of their daily life.

Sample Answer
Currently, there is a widespread belief that it is unnecessary to let their young children learn how to manage money.
Nevertheless, I cannot agree with this statement. As far as I am concerned, it is crucial for children to learn how to manage
their money, though maybe only small amount of, in order to be a financially responsible adult.
First and foremost, learning to manage money makes a child know how precious the money is. Therefore, they wont
exploit their money on things of no use when they grow up. Taking myself as an example, when I was young, I used to

waste money on snacks and toys which is of no use. Then one day, my mother gave me 10 dollars and told me that this
was all the money available in this month. I reluctantly managed my money and felt that it was really hard to get enough
money. Therefore, I never waste money any longer as I grow up.
Secondly, learning to manage money teaches children how to increase their fortune. As I have said in the last statement,
money is precious and it is hard to get enough money to use. Therefore, the best way to get adequate money is to increase
our fortune by ourselves. We manage our budget and put our money into things that can increase our fortune for
instance, a child may use a part of his pocket money to buy some cards and sell the cards to his friends to gain some extra
money. Children who are familiar with how to increase their fortune by managing their money are more likely to gain
financial success in the future.
In addition, learning how to manage money can also help children to manage other aspects of their daily life. As a child is
used to managing his pocket money, he tends to put this practice into managing other aspects in life, such as time. People
who do good time management usually have a balanced life. They work and study happily and enjoy their life. For example,
my brother, Tom, is good at allotting time in his life, because when he was a child, he was free to decide how to make use
of his money and then cultivated a superior ability in arrangement.
To conclude, I strongly recommend children be taught how to manage their money when they are young. This practice can
provide them with a greater likelihood to succeed and a well-balanced life when they become adults. It will benefit not
only themselves but also their whole life.

2009/05/09
2010/01/15

Level 3 Level1-Level5

TPO 15
TPO15-
Three measure to stop the spread of the can toad in Australia
TOPICthree measures proposed to stop the spread of the can toad in Australia

1.Build a national fence

1.young cane toad and eggs in the river can easily go


through the fence in the rivers or stream. As long as a few of
them can reproduce on the other side, the fence is
ineffective.

2.Volunteers capture and destroy cane toads

2.Some untrained volunteers, who cant tell the difference


between cane toads and native toads, some of which are
endangered, especially when they are young, may destroy
many Australias native frogs.

3.Reserachers develop a disease-causing virus to control


the cane toad populations

3. Virus may be transported to other continents by


researchers or pet collectors, where cane toads are a native
species. Therefore, the destruction of cane toads in those
areas will cause an ecological disaster. And if they are
eliminated, the whole ecosystem will suffer.

Sample Answer
The article proposed three measures to stop the spread of cane toads in Australia. However, the professor argued that
those measures would either be unsuccessful, or worse still, cause unwanted environmental damage.
First, a national fence to prevent cane toads from colonizing other parts of Australia. The professor pointed our that rivers
and streams where young toads and toad eggs are found could flow from one side to the other, carrying the young toads
and their eggs with them. As long as a few get through the fence, they will establish population on the other side,
rendering the fence ineffective.
Second, sending volunteers to trap and destroy can toads. The professor said that untrained volunteers may not tell the
difference between the cane toad and native frogs, especially when they are young. Therefore, they would inadvertently
destroy many of Australias native frogs as well, some of which endangered species.
Third, Using a virus to control cane toad population. The professor argues that this virus on Australian animals may be
transported to other continents by researchers or pet collectors, and consequently, cause terrible damage to cane toads in
their original habitat in Central and South America, where they are a vital part of the ecosystem. The consequence would
be a huge ecological disaster.

Level3 Level1-Level5

TPO15-
Do you agree or disagree with the following statement?
In order to become financially responsible adults, children should learn to manage their own money at a young age.

/
/ financially
responsible

Thesis statement: statement.


1
2
3
4
2 3

Thesis Statement: statement.


1
1
2
2
3
3
However
statement

financially responsible

1
2
3

Sample Answer
People, especially adults, are expected to be financially responsible, that is, to be able to save money and earn a living.
What about young childrenShould we trust money to their tender hands? From my perspective, the freedom to mange
money at a young age will cultivate financially responsible adults.
To begin with, getting in touch with money at a young age will prompt children to save money. Traditionally, children are so
used to turning to their parents for anything that they generally have no idea about saving money. I used to see a child
showed off a brand-new apple watch, a luxury worth half of my salary and shrugged my mom got it for me. Its not
expensive at all! However, when children were asked to manage their own money, they start to realize how difficult it is to
collect even a small amount. My 8-year-old cousin used to order a large pizza without even blinking an eye. However, after
his mother gave him a piggy bank and a daily pocket money of 5 yuan, he found he even had to wait for over a week for a
small slice of pizza. From then on he never wastes even a single penny. Obviously, an early exposure to money cultivates in
children a sense of money saving.
Furthermore, children managing their own money are more willing to make money. Once they realize the value of each
penny and start to save, they will also consider how to add more money to their coffer; consequently, they will figure out
ways to make money. Still take my cousin as an example, he made an agreement with his mother during summer holiday:
he would walk the dog twice a week, and in return, his mother would pay him 10 yuan each time. Gradually his business
branched out and he started to walk dogs for his busy neighbors as well. At the end of the holiday, he earned a total of 500
yuan!
In conclusion, learning to manage money will urge a child to save and make moneyboth valuable qualities that will
pave way for a financially responsible adult in the future.

Level 3 Level1-Level5

TPO 16
TPO16-

TOPICthe development of archaeology in Britain

1. new constructions led to the loss of artifacts

1. Before the new construction starts, examinations have to


be taken. And preservation plans would be worked out if
the site is of archaeological interest.

2. the financial support from the government is


inadequate.

2. archaeology works will be paid by the construction


company, not the government. this fund will cover a far
more greater range of archaeology works.

3. a decline in archaeology-related job opportunities

3. new rules provide quite a lot paid work for


archaeologists. the job opportunities in archaeology has
been the highest ever.

Sample Answer -1
According to the lecturer, all three problems upon the science of archaeology mentioned in the passage has been
improved by the new rules and guidelines.
First, the lecturer disagrees with the passage's opinion that new constructions led to the loss of artifacts. Before the new
construction starts, examinations have to be taken on the archaeological value of the cite. And preservation plans would
be worked out by builders together with local government and archaeologists, if the site is of archaeological interest. Then
construction would be carried out around the site or after proper excavation. Therefore the passages assumption on
careless destruction of archaeology sites is false.
Second, the lecturer revises the inadequate financial support which is discussed in the passage. One important part of the
new rule is that archaeology works will be paid by the construction company, not the government. The construction
company will pay for the examination of the site, as well as all other preservation works under the plan. And this fund will
cover a far more greater range of archaeology works. So the new rule will largely turn around the financially deprived
situation of archaeology works by providing a steady stream of fund.
Third, the lecturer thinks that new rules provide quite a lot paid work for archaeologists, not like what is stated in the
passage. Archaeologists are hired in many works, from examining the site to excavation work, artifact protection, and
finally processing the data. Moreover, the job opportunities in archaeology has been the highest ever. The new rules
largely expanded the originally shrinking job pool in archaeology work.

Sample Answer -2
The professor explained three aspects of the new guidelines adopted in the United Kingdom, each of which has
successfully addressed the problems mentioned in the reading passage concerning the preservation of artifacts, the
funding of archaeological research, and career opportunities in archaeology.
First, the professor pointed out that according to the new guidelines, any construction project must undergo an evaluation
process in order for the archaeological value of the site to be ascertained. If a site is determined to contain precious
objects, a panel of experts is convened to draw up a preservation plan. This measure has proven very effective in
addressing the first problem mentioned in the reading careless and irresponsible construction projects being pursued
without any regard for archaeological preservation.
Second, the lecturer explained that the new guidelines shifted the responsibility for funding archaeological work, from
initial examination to future preservation, from the government to construction companies. As a result of this shift, the
second problem highlighted in the reading concerning insufficient government funds for archaeological research has been
significantly addressed.
The professors last point concerned the final problem raised in the reading material a decline in archaeology-related job
opportunities. She argued that thanks to the new guidelines, many skillful, full-time archaeological experts and
professionals are now needed to handle work at various stages of archaeological investigation, research, and preservation.

Level 3 Level1-Level5

TPO16-
The best way to travel is in a group led by a tour guide.

//

I disagree
4. Admittedly, traveling in a group is a good way. However, to consider this as the best way is a piece of overemphasis.
5. Traveling on one's own may be more flexible and more funny.
6. Traveling on one's own may cost less.
I agree
1Being led by a tour guide in group can offer us convenience during our vacation.
2following a group also guarantee a deeper understanding of the culture in your destination.
3being in a group with a tour guide will make you safe during the whole vacation.

Sample Answer
Currently, there is a widespread belief that it is a better choice to travel on one's own. Nevertheless, I cannot agree with
this statement. I would prefer be led by a tour guide while I am traveling around. There are several benefits of being led by
someone who is acquainted with the place you go and of being in a group.
Being led by a tour guide in group can offer us convenience during our vacation. Usually we are not familiar with the
landscape, the direction and the customs of our destination. Sometimes if we travel abroad, we may even know nothing
about the language and thus cannot turn to anyone for help. I still remember when I traveled to Italy; I knew no Italian at
all and found it hard to stay in this foreign country, let alone enjoy myself! Finally I found a travel agency and then followed
the group. The tour guide offered me a lot of convenience during my stay in Italy and finally I went back home safe and
sound.
In addition, following a group also guarantee a deeper understanding of the culture in your destination. Almost all the tour
guides have received formal training in the local history and culture. Actually every time we stay in a group with a tour
guide, he will always tell you the customs and the detailed history of a site of interests. Therefore, by staying with the
group and listening to the tour guide, we will learn more about our destination without much effort.
Moreover, being in a group with a tour guide will make you safe during the whole vacation. Actually this point of view is
the most important in my point of view. The tour guide is familiar with the country or the city we visit and thus we can
hardly get lost. Staying in group makes us less likely to be focused by some bad people. All of us want a safe tour. As a
result, the best way to guarantee safety is to stay in group with a tour guide.
All in all, thought we might lose some freedom by staying in a group with a tour guide, this way of traveling still can be
well-rounded because it offers us convenience, deeper understanding and safety. Under this circumstance, I want to state
that the best way of traveling is to be with a group and to be guided by a person who is familiar with our destination.

2006/11/18
2008/03/02

Level 3 Level1-Level5

TPO 16
TPO16-
problems and solutions regarding British archaeology
TOPICProblems and solutions regarding British archaeology

1. Many valuable artifacts were lost to construction


projects.

1. New guidelines state that before any construction


project, the site has to be examined for its archaeological
interest or value. If the site is of archaeological interest, a
plan for preserving the artifacts should be made by builders,
archeologists and local government officials.

2.Many archaeologists lack financial support for their


research.

2.Any archaeological work done on the construction site will


be paid by the construction company rather than the
government. The company should pay for the initial
examination fee, as well as all the work done under the
preservation plan.

3.A career in archeology Is difficult to find.

3. New guideline provides a lot of paid work for


archaeologists. They are hired at all stage of the process. In
fact the increased job opportunities in archaeology has
boosted the number of professional archaeologist in Britain
to the highest in history.

Sample Answer
The article points out three difficulties facing British archaeology. However, the professor argues that with the
adoption of new rules and guidelines in 1990, the situation has been improved.
First, many valuable artifacts were destroyed in construction projects. The professor argues that according to the new
guideline, all construction sites must be examined beforehand to determine its archaeological values. Once a site is of
archaeological interest, a preservation plan would be jointly made by builders, local governmental officials and
archeologists, either by circumventing the archaeological site or by excavating artifacts first.
Second, Many archaeologists lack financial support for their research. The professor points out that the guidelines
stipulate that construction company, instead of the government, should pay for the archaeological work done, including
the initial examination fee, as well as the work done under the preservation plan. This new policy allows a greater range of
sites for archaeological research.
Third, Archaeologists find it difficult to find a career. The professor said the problem could also be solved as a result of
the new guidelines, which provides a lot of paid work for archaeologists. In fact they are hired at all stage of the
archaeological process from initial examination to preservation plan to the final date processing and report writing, which
boosts the number of professional archaeologists to a record high in British history.

Level4 Level1-Level5

TPO16-
Do you agree or disagree with the following statement? The best way to travel is in a group led by a tour guide.

group tour

1
2.
3.

1.
2.
3.

Sample Answer
Are you tired of constant study, exhausted from endless work, or just bored to death my monotonous life? Travelling can
totally refresh and reboot you. However, when you are mentally prepared for a relaxing tour, you are faced with a practical
issue: should I travel along, travel with friends or family, or just join a tour group? As far as Im concerned, group tour led
by a guide is the best choice.
To begin with, group tour is the most convenient way of travel. With the solicitous help and meticulous arrangement by
the tour guide, you no longer need worry about trivial business such as booking hotel rooms or ordering tickets before
hand, nor would you worry about language difference and inadequate knowledge about that place. Pack up your things,
carry cash or credit cards, and you are ready to go! I still remembered the trip to France I had last year. At first I was so
worried about my poor French, yet my kind-hearted tour guide was such a blessing! She stood by me every step of the way,
helping me communicate with locals and introducing me to magnificent architecture and brilliant culture in Paris. I skated
through my days in Paris with great joy.
Second, Group tour is the most economical travel choice. We may all have enjoyed the luxury of a one-week travel, yet
also lamented the large amount of money paid along the way. However, group tour can strike a perfect balance between
tourist delight and a limited budget. Travel agencies are capable enough to negotiate bargains with airlines, hotels and
restaurants and come up with the most money-saving plans for us. Moreover, they could land us discounts for tickets of

tourist scenes and performances that would usually cost us quite a fortune. Take my mother journey to Korea for example,
she originally planned to travel with just my aunt, but after calculating all the cost; she decided to join a group tour that
cost her only half of her original budget.
To sum up, group tour is the wisest way to enjoy a carefree and money-saving journey.

Level2 Level1-Level5

TPO 17
TPO17-

TOPIC

1.

1.

2.

2.

3.

3.

Sample Answer
The writer of reading passage is worried that the number of birds in US will decline as a result of human intervene.
However, the lecturer argues that this is not the case.
First, the reading indicates that birds' natural habitats (forests, wetlands, and grasslands) will give way to human
settlements (homes, malls and offices), so the size of the bird population will also decrease. In contrast, the lecturer claims
that this only causes some types of birds to disappear; for some others, like seagull, pigeons and hawks, they have increase
their populations in human environment.
Second, the lecturer refutes the argument in the reading that bird populations in rural areas will decline because bird
habitats are turned into agricultural land. According to the lecturer, less land is converted to agricultural use every year
owing to new crops that are more productive in per unit of land. So there is no need to increase agricultural land, and birds
will continue to live in these wilderness areas.
Finally, contrary to the statement in the reading that an increasing use of pesticides will harm birds, the lecturer argues
that more and more pest-resistant crops are being planted. Therefore, the use of pesticides will actually decline and birds
will enjoy a better living environment.

The writer of reading passage is worried that ... However, the lecturer argues that this is not the case.
First, the reading indicates that ... In contrast, the lecturer claims that
Second, the lecturer refutes the argument in the reading that ... According to the lecturer,
Finally, contrary to the statement in the reading that , the lecturer argues that

Level 3

TPO17-

Do you agree or disagree with the following statement?


Most advertisements make products seem much better then they really are.
Use specific reasons and examples to support your answer.

products in most advertisements appear much better than they really are

Sample Answer
[483 words]
There is a heated debate over the honesty of many advertisements. Naturally advertisers argue that they present the best
aspects of products and shrug off the responsibility of providing a realistic presentation. However, products in most
advertisements appear much better than they really are. This is unfair to consumers.
To begin with, advertisers use light and special effects to make products look brighter and more colorful. For example food,
such as fruit, dishes and snacks are presented in advertisements in such a way that they show up to be more fresh,
appealing and delicious. But if we buy them with the hope of getting the products as good-looking as in ads, we will end
up in disappointment. This is because advertisers pick the best items they have for promoting and edit or modify the
image to perfect it. The products available to purchase, however, are plain and less appetizing.
Also, many products are displayed in a way to give a false sense of size. As explained in the principle of perspective, things
near the eye appear larger than the same things in distance. Similarly, when the camera is closer to a product, the product
looks bigger. Contrary to what an ad wants us to believe, a large doughnut can be tiny, a suitcase not capacious, and
luxurious rooms incommodious. Thus, instead of believing everything as it is depicted in ads, the best way is to see it for
real, in person.
In addition, spokespeople in advertisements often falsely brag out the efficacy of a product. According to the annual
report on the advertising industry published in the New York Times in 2014, in nearly 90% of all commercial
advertisements around the world, spokespeople overrate the products they speak for. If a girl who was born thin speaks
for weight-controlling medicine. Or a blonde, with naturally soft hair, says it is a special shampoo that makes her hair
smooth. Or a bunch of teens yell that a particular beverage builds up their happiness, while they may be just as happy
without drinking the beverage. Then consumers would be unwise to swallow the bait of the advertisements.
Finally, advertisers might mislead the customers by employing celebrities to endorse their products or services.
Consumers often trust that these celebrity endorsements mean the items are of superior quality even if there is no proof.
For example, if Michael Jordan is endorsing a certain brand of basketball sneakers, people may feel those sneakers are the
best available ones even if Michael Jordan does not say this at all. In fact, Michael Jordan may not actually wear the
sneakers. However, people trust the celebrity endorsements and do not do any research on the products.
As a final analysis, considering advertisements modify the appearance of products, present them out of proportion and
exaggerate their usefulness and functions, we can safely draw the conclusion that most advertisements reflect products
deceivingly better.


CN 2007/12/8
Advertisements are a waste of company's money and energy because consumers know what they want to buy.
CN 2014/4/19 2013.1.25NA
For any business to be successful, it must spend a lot of money on advertising.

Level2 Level1-Level5

TPO 18
TPO18-

Torreya

taxifoha 3 3

TOPIC

1. 1

1. 1 Florida

2. 2, 2. 2
migrated migration

2
3. 3

3. 3

Sample Answer
The lecture believes that the three ways about protecting the Torreya trees in the reading are not reasonable. The reasons
will be listed as follows.
First, according to the lecture, the effort to save the Torreya by reestablishing the microclimate in the same location is
unsuccessful because of what has happened to the climate in the large area. Global warming and the drain of wetlands all
contribute to the warmer and dryer climate in the Florida. Therefore, wherever the Torreya were established in the local
area, it would be useless because of the change of the big climate. This is different from the point in the reading that
restoring the habitat for Torreya locally can be a good way to address the decline because of the favorable climate.
Second, the lecturer believes that assisted migration may not be a proper solution either. She cites an example that people
once helped another kind of trees to move to another place to protect the species, while this kind of trees spread so
quickly that they killed other trees in the new environment, some of which had even already endangered or faced
extinction. So the human assistance may even pose more harmful effects to the new place. Again this contradicts the point
in the reading that finding a new location to restore tree population can be effective.
Finally, Contrary to the point in the reading that setting up a research center could be a better shelter for this endangered
species, the speaker says that trees in research center may be less resistant to diseases. In order to resist the diseases, the
trees population needs to be relatively large and genetically diverse. Unlike the wild area, research centers are relatively
small, and the genes of trees are not diverse enough to help the trees to survive the diseases. Therefore, tree population
may suffer from the illnesses in the long term.

Level 3 Level1-Level5
TPO18-

Do you agree or disagree with the following statement?Students are more influenced by their teachers than by
their friends. Use specific reasons and examples to support your answer.

influence influence

Sample Answer
It is not uncommon to see that young students are easily influenced by the people around them, such as by their teachers
or by their friends, since they have not been mentally developed. If you ask me who would have more influence on
students, teachers or friend, my answer is that friends do more, simply because friends are people who students spend
more time with, and they are easier to communicate with each other. My reasons will be further illustrated as follows.
First, friends influence more on students because of more time spent together. More specifically, teachers spend time of
lecturing or tutoring with their students, which actually occupies a small part of students time. On the other hand, it is
their friends, like their roommates, who spend more of their spare time together. Take myself for example; I was a boarder
when I was in my secondary school, and Tom was and is my best friend of mine. At that time, we did most of things
together and influenced each other a lot. In weekdays, together we attended our favorite courses on American history and
physics in the morning; together we had same passion on football and played it in the late afternoons; together we went
to the library to read our favorite novelSherlock Homes, and discussed it on the way back to the dormitory. Clearly,
because of the influence from friends, we have developed similar academic interest and after-class activities.
Second, another reason that friends influence more is that friends are of the same age and easier to communicate with
each other. Generation gap is usually broader between students and teachers, and teachers always have some sort of
authority. Because of these two reasons, teachers often cannot match friends in influencing students interest in some
subjects. For instance, I was not a good learner on math when I was in my secondary school, because my math teacher was
an old lady, whose way of teaching was not a piece of cake for me. Worse, I had lost the interest in learning math. However,
it was my roommate, Tom, who always discussed math problems with me and inspired my interest in math again. He
always used the ways that I could understand to explain some difficult concepts for me. Without his influence, I would not
be interested in math any more.
It has to be admitted that teachers do have influence on students in a few things, such as ways of learning and the
knowledge of the subjects they teach. However, because of the disadvantages in time and age difference, the teachers
influence may possibly be diminished compared with that of friends. Therefore, I can come to the conclusion that friends
have more influence on students than teachers do.


2015.6.27 ()
Do you agree or disagree with the statement: It is better to make friends who are intelligent than with a good sense of
humor.
2015.5.30()
Do you agree or disagree with the statement: When you have a friend for a long time, whether you will continue the
friendship even he or she does something you do not like.
2014/11/8 /
Getting advice from friends who are older than you is more valuable than getting advice from friends of your own age.
2014/9/21
Moving to a new place or a new city is not a good idea because of the loss of old friends.
2014/3/1
It is often not a good idea to move to a new city or a new country because you will lose old friends.
2013/11/2
Do you agree or disagree: it is better for people to have friends who are intelligent than to have friends who have a good
sense of humor.
2013/7/21
It is impossible to be completely honest with your friends.

Level3 Level1-Level5

TPO 19
TP19-


Reading:
Buzzing-the advisers hire people-buzzers-who personally promote product to people they know or meet. Many critics like
to see it banned.
Listening: the passage described buzzing leads out a lot and gives the wrong impression.

TOPICBuzzing

1. Consumers should know whether a person praising a


product is being paid to praise the product.

1. Buzzing is that firms hire people who use the product and
find the product is really good.

2. Since buzzers pretend they are just private individuals,


consumers listen to their endorsement less critically than
they should.

2. People do not trust all hear from buzzers.

3. Buzzing effects social relationships harmfully.

3. Buzzing can strength social relationships.

Sample Answer
The reading material introduced three reasons to explain why buzzing should be banned. However, the speaker says that
the theory stated in the article was entirely different and somewhat inaccurate when compared to what happened for real.
First, the speaker asserted that firms hire people who use the product and think the product is good. Therefore, buzzing is
not the ordinary advertisement like the actors are paid to read what the company want they read. Buzzers believe the
product is good and express their real opinion. For speakers personal experience, he uses the phone service and does
think that it is great. Thats why the company hires him.
Second, the passage states that buzzers pretend to be private individuals. Thus, the customers listen to their
recommendation less critically than usual. Nevertheless, the speaker believed that consumers would not trust all hear
from buzzers. The people he talked to ask information about the product such as service, price and how long he used it. If
he cannot answer all the questions, the consumers neither trust his words nor buy the product.
Third, the article talks about buzzing effect social relationships harmfully. The speaker totally refutes this idea. In his
opinion, buzzing can bring sincerity and trust. Companies recruit people who really think the products are good as the
buzzers. Persons who try the products buzzers promoted to them are going to have the good experience. Therefore,
buzzing strengthen social relationships.
Because of the above reasons, it is highly questionable that buzzing should be banned. In fact, it could bring positive
consequence for companies, customers, buzzers and social relationships.

Level2 Level1-Level5
OG 150-225 words

TPO19-

Do you agree or disagree with the following statement? In order to be well-informed, a person must get information from
many different news resources. Use specific reasons and examples to support your answer.

Agree: in order to be well-formed, a person must get information from many different news resources.
1. Different news resources enable people get latest information
2. Receive information from different perspective
3. Distinguish reliable news
Disagree: a person does not need to get information from many different news resources
1. Many resources only provide segment information
2. It is time-consuming to read all the news from different resources

Sample Answer
Media is everywhere in these days. Trying to obtain a variety of information from robust and reliable sources becomes
more critical. Therefore, I believe that people need to acquire information from various news resources in order to be
well-formed. The following are the reasons to explain the significance to gain knowledge from different news resources.
First, diverse news accesses enable people to get the latest information. Previously, people usually obtain information from
newspaper and TV. My family subscribed three kinds of the newspaper. And my parents used to watch the news report on
TV at night. However, these two channels do not provide updated information. They usually summarize events happened
in yesterday and earlier today. Nowadays, people can get the latest news from websites and mobile phone applications.
They updated news very quickly. When the earthquake happened in China, people knew this information several minutes
later online.
Moreover, it is crucial to receiving information from the different perspectives. Various sources of information help people
understand things comprehensively. For instance, last week, I had to prepare a presentation about the influences of the
declined stock price in the Chinese market. The Chinese newspaper mainly reported the impacts on citizens. On the other
hand, the Western presses such as The Wall Street Journal and the New York Times stressed the effects on the European
and American stock markets. The reports in eastern Asian countries such as Japan and Korea predicted that the decrease
of Chinese stock price would cause the fluctuations in Asian stock market. I need to collect these three sources of
information in my presentation because they all have different perspectives, and each of them is quite indispensable.
Last but not the least, adequate news supplies help us distinguish which information is reliable and which is not. There are
numerous news presses in recent years. Some are well-known and established; some are newborn and quite small. It is
usual that some journalists report news biased and even hidden the truth for their sake. Therefore, obtaining information
from different news sources is very useful to distinguish reliable information. Also, sometimes the reports are not very
precise. Once, I looked for the statistics of the changes of the currency rates. The journals only gave me the rough numbers.
Then, I searched on the internet and got more precise number round to four decimal places. One news source may not
provide us adequate information. Therefore, we need collect information from various resources.
In a nutshell, I do agree that persons need to get information from many different news supplies in order to be

well-formed. Various news sources provide the latest information, different perspective and trustworthy supplies to us.

2014/02/22 CN
People who develop many different skills are more successful than people who focus on one skill only.
2014/12/19 NA
Independent Writing Task:
Do you agree or disagree with the following statement:
You can learn more through reading or watching news presented by people with different views with you than reading or
watching news presented by people with similar views with you.
Use specific reasons and examples to support your answer.

Level4 Level1-Level5

TPO 19

TPO19-

TOPICbuzzing ()

1. Consumers should know whether a person praising a


product is being paid. With buzzing what you hear is just
paid advertising

1. Buzzers tell the truth about the products theyre buzzing.


Buzzers are people who use the products and who really
think the product is good.

2. Consumers listen to buzzers endorsements less


critically than they should.

2. Consumers dont believe whatever they hear from the


buzzer. They ask questions and wont buy the product until
they get good answers.

3. Buzzing is likely to have harmful effects on social


relationships.

3. What you get from a buzzer is not only sincere but is


likely to be about a good product. People who try buzzed
products are going to end up being more trustful and open
up to people.

Sample Answer
While the passage holds that buzzing has negative effects and should be banned, the buzzer Bill/speaker in the listening
material claims that the article gives a wrong impression of buzzing.
To begin with, Bill casts doubt on the idea that consumers are misled by the buzzers with incorrect information from paid
advertising. He points out that buzzers are chosen from the people who indeed used the products and felt good. Therefore,
buzzers are just paid for telling truth to other people.
Second, according to the passage, while consumers are critical of commercial advertisements, they are more likely to trust
the opinions of buzzers who pretended as private individuals. Bill refutes this point by stating that people do not always
believe what the buzzers say, instead, they will keep asking the details of products and wont make a purchase until they
get good answers.
Last, Bill thinks the third idea that buzzing can destroy social relationships by spreading dishonesty is also unconvincing. He
points out that the opinions of buzzers are sincere and trustful. As a result, people who buy buzzed products will have a
good experience and buzzing can actually enhance the trust in social relationships among people. (198 words)

level 3 Level1-Level5

professor

TPO19-

In order to be well-informed, a person must get information from many different news resources.

1. Analyzing the question


+

2. Defining Position
neccessity and importance + 1+
2+

3. Brainstorming

4. Topic sentences ()

4-1 First, it is necessary to read news from as many sources as possible because only a variety of news supplies can give us
a comprehensive, close-to-truth report of what is happening in the world.

4-2 Another reason for the importance of getting information from different kinds of sources is that the news supplied by
different reporters or journalists may unavoidably reflects their own experiences and viewpoints. Whats worse, some
news reporters or medias may deliberately fabricate or distort facts for their own interest.

5. Keyboarding ()
5-1 Introduction
Introduction=General Statement+Thesis Statement

5-1-1
With the explosive growth of news media, it becomes much easier for people to obtain information today than in the past.
We can choose from a wide range of news sources such as printed media, the TV and radio, and the Internet.
5-1-2 + However, in order to obtain accurate and updated news, a person needs to get information
provided by a variety of news sources instead of a single one for the following reasons.
5-2 body paragraphs ()
5-2-1 1++ topic sentence 1+details

5-2-2 2++ topic sentence 2+details


2 3
the other side
5-3 Conclusion
+Reworded Thesis Statement+topic sentences

Sample Answer
With the explosive growth of news media, it becomes much easier for people to obtain information today than in the past.
We can choose from a wide range of news sources such as printed media, the TV and radio, and the Internet. However, in
order to obtain accurate and updated news, a person needs to get information provided by a variety of news sources
instead of a single one for the following reasons.
First, it is necessary to read news from as many sources as possible because only a variety of news supplies can give us a
comprehensive, close-to-truth report of what is happening in the world. The fast-changing and advancing society makes it
difficult for any news source to give an all-round picture of events happening around the globe. Some news sources
specialize in the latest trends while others are committed to do in-depth investigations and reports. For example, when the
central bank lowers the interests of saving and borrowing money, the financial reporters may write on how it affects the
real economy and stock market while the local TV station may focus on the effects on the publics daily life and purchasing
power. The perspectives of the reports vary greatly from one news source to another and offer us different interpretations.
Only in this way can we avoid forming a narrow or biased understanding of the news items.
Another reason for the importance of getting information from different kinds of sources is that the news supplied by
different reporters or journalists may unavoidably reflects their own experiences and viewpoints. Whats worse, some
news reporters or medias may deliberately fabricate or distort facts for their own interest. It is common that some reports,
opinions and commentaries may give partial, misleading or inaccurate information to gain publicity or make profits. When
work injuries happen on a construction site, for instance, the media may be bribed by the construction company to cover
the truth and do not report the actual casualties. As a result, a person pursuing the objective and truthful reports needs to
choose and discriminate between different sources.
To conclude, as the society is flooded with all kinds of information and the accuracy of their sources cannot always be
verified by individuals, a person must get access to a wide range of news sources in order to get objective and complete
information. (393 words)

2013/10/11, 2012/06/30

Level 4 Level1-Level5
well-informed

TPO 20
TPO20-

TOPIClet it burn policy let it burnlet it burn

1. damage to trees and other vegetation one third of


Yellowstones land had been scorched

1. the plants in Yellowstone became more diverse because


the fire created an opportunity for some other species
smaller plants took over open and shaded space;
seeds of some plants germinated when they were exposed
to high level of heat

2. park wildlife was affected deer and elk fled; other


smaller animals were unable to escape

2. smaller plants that grew after the fire provided habitats


for animals like rabbits and hares the thriving of rabbits
and hares contributed to the increase of predator
population food chains became stronger

3. fires affected tourism of Yellowstone tourist season


was cut short and many tourists chose to stay away

3. Yellowstone fire would affect tourism only if it happened


every year but the 1988 Yellowstone fire was a rare
combination of factors: low rainfall strong winds
accumulation of dry undergrowth this combination
has not occurred since 1988 visitors came back year each
year after the 1988 fire

Sample Answer
question: Summarize the points made in the lecture, being sure to explain how they
challenge the specific point made in the reading passage.

response

1

2
1
2
TPO ---
---

1
The reading proposes let it burn policy should be replaced with another policy of extinguishing the fire as soon as it
starts because of three kinds of damages. However, all the points are questioned by the listening, claiming that the fire can
actually bring three corresponding benefits.
First, the reading states the fire caused devastating damage to the vegetation of Yellowstone Park because nearly one third
of the land has been scorched after the fire. By contrast, the lecture argues the fire provided an opportunity for new plants
to grow. Smaller plants that needed open and shaded space could thrive, and certain kinds of plants would not germinate
unless they were exposed to very high level of heat. Thus, the fire made the vegetation species more diverse by adding
new plants that will not grow otherwise.
Second, the reading claims fires also affected the animals in the park: elk and deer fled and smaller animals were unable to
escape from the fire. By casting doubt on the second idea of the reading, the professor argues the wildlife population not
only recovered but also thrived. Rabbits and hares could find ideal habitats in the smaller plants that grew after the fire.
The increasing number of rabbits and hares would invite predators. In this way, the food chain became even stronger.
Finally, the reading presents its worry about the negative influence of fire on Yellowstone tourism. While according to the
lecture, this is not the case. The professor in the listening explains the occurrence of 1988 Yellowstone fire was an unusual
combination of many factors: low rainfall, strong winds and accumulation of dry undergrowth. This kind of fire has never
happened again after 1988, and visitors came back every year after that. Therefore, the worry of tourism in the reading is
unnecessary.

2
The reading and the listening contradict each other on the issue of whether let it burn policy is good. The reading cites
three damages of this policy from critics. The damage includes devastation of vegetation, the fleeing and death of animals,
and the loss of tourism. However, the listening states let it burn policy is fundamentally a good one because it is only
destructive but also creative.
First, the professor argues the fire provided an opportunity for new plants to grow. Smaller plants that needed open and
shaded space could thrive, and certain kinds of plants would not germinate unless they were exposed to very high level of
heat. Thus, the fire made the vegetation species more diverse by adding new plants that will not grow otherwise. This
challenges the first idea in the reading that the fire caused devastating damage to the vegetation of Yellowstone Park
because nearly one third of the land has been scorched after the fire.
Second, the professor continues the wildlife population not only recovered but also thrived. Rabbits and hares could find
ideal habitats in the smaller plants that grew after the fire, and the increasing number of rabbits and hares would invite
predators. In this way, the food chain became even stronger. This is opposite with the second idea in the reading that fires
also affected the animals in the park as elk and deer fled and smaller animals were unable to escape from the fire.
Finally, the lecturer claims the occurrence of 1988 Yellowstone fire was an unusual combination of many factors: low
rainfall, strong winds and accumulation of dry undergrowth. This kind of fire has never happened again after 1988, and
visitors came back every year after that. Therefore, the worry of tourism in the third paragraph of reading is unnecessary.
This denies the third idea in the reading that tour time was cut short and visitors would stay away from Yellowstone after
the fire.

TPOTPO 20 TPO 14

1
The reading /talks about a phenomenon (/problem)/tends to believe However, all the points in the reading are
questioned by the listening, claiming that
First, the reading claims By contrast, the lecturer argues
Second, the reading states While according to the lecture, this is not the case. The lecturer presents that
.
Finally, the reading holds that By casting doubt on the third idea of the reading, the professor in the listening
points out

1
The listening and the reading contradict each other. The professor in the listening thinks/claims/ argues/ believes/
holds/ states/ insists/ maintains/ (that). However, the reading notes (that) .
First, the lecturer states (that) .
This / contradicts / disagrees with/ opposes/ is opposite with / challenges/ conflicts with/ the first idea in the
reading (that) .
Second, the professor in the listening holds (that).
This challenges the second idea in the reading (that).
Third, the professor believes (that) .
This conflicts with the third idea in the reading (that) .

Level4 Level1-Level5
LEVEL 4
TPO20 LEVEL 4 TPO 20
seeds of some plants
open and shaded space seeds of some plants wont germinate unless they are exposed to very
high level of heat . germinate
Yellowstone fire low rainfall strong winds
accumulation of dry undergrowth. 20 25

TPO20-

Do you agree or disagree with the following statement? Successful people try new things and take risks rather than only
doing what they already know how to do well.

try new things and take risksdoing what they already


know how to do well
successful people commercial, political,
entertainment, sports, arts, literature successful people try new
things and take risksdoing what they already know how to do well

OG4 565
No matter which position you take, it is important to support your opinion with details and examples.

try new things and take risks try new things,


takes risks
try new things and take
risks doing what they already know how to do welldoing what they already know how to do well
try new things,
take risks Take
risks,

Successful people
successful people,
Emma Watson. Emma Watson
HeforShe
Emma Watson

Amazon Jeff Bezos


2300%Jeff
Bezos Amazon, Jeff Bezos
15
Michael Jordan 1994
Michael Jordan
1995 NBA

paraphrase

topic sentence(
) topic sentence

agree
disagree

Sample Answer
Agree
Some traditionally deem that people should stay in their familiar fields thus they can be proficient and productive.
However, the truth is, success always belongs to people who have curiosity about new things and have the courage to take
risks.
Initially, new fields broaden peoples horizon and tap their potential. Many successful people enter new fields even though
they have gained some achievements in their original specialties. Thats because they have the discernment to realize that
if they stay in the familiar niche, they will slack off in the comfort zone. To be eminent, they have to push themselves to
pursue challenging dreams despite its difficulty. For instance, Emma Watson, an English actress who rose to prominence by
portraying Hermione Granger in Harry Potter series, uplifted her personal value and career success by stepping onto
political arena. This beautiful actress engaged herself not only in movie shooting, but also in philanthropic and political
causes. Appointed as a UN Women Goodwill Ambassador in 2014, she visited Uruguay where she gave a speech
highlighting the need for womens political participation, and went to Bangladesh and Zambia to promote education for
girls. In September that year, Watson delivered an admittedly nervous speech at UN headquarter, which was aimed to
launch the campaign Heforshe. Today, Emma Watsons contribution to gender equality around the world earns her
admiration from both her fans and those who believe in gender equality. This movie star obtained a wider accomplishment
because she did not confine herself in movie career.
Besides, successful people are willing to take risks as their intuitive feelings encourage them to make gut decisions. Those
energetic ones believe everyone has individual gift; a gift that frowns on dogma and offers you audacity to embrace
originality. Jeff Bezos, the founder of Amazon.com, would never be so successful if he chose to lead a life of ease. Before
starting his business, Bezos worked in a hedge fund company where he enjoyed high salary and reputation for his
profession. When he came across a fact that web usage was growing at 2300 percent every year, he began to scheme an

online book store a store that did not exist in physical world but could bring revolutionary change to traditional selling
market. Yet this idea was too risky because no one before had thought about this innovative selling channel and it seemed
there was no need for a man with an already satisfactory job to take the shot. Ultimately, Jeff decided to give it a try, as
quoting from him I would be haunted by a decision to not try at all. Under his guidance, Amazon became the largest
online retailer and Bozoss personal wealth now ranks number 15 on Forbes list of billionaires. Successful people take pride
in their pursuit for adventure.
In conclusion, rather than huddling up in comfort zone, productive people choose to go out and explore in new areas.

Disagree
The society is loaded with chicken soup, depicting an image to young people if one wants to be successful, he should have
the audacity to try new things and take risk. Does this really make sense? In reality, those who are committed in an area
for years have more possibility to succeed.
The first reason for only doing what you already know is you can be professional while trying new areas will expose
yourself in an awkward and amateur manner. One of the best examples is Michael Jordan. Michael Jordan, honored as the
greatest basketball player of all time, reached the climax of his professional career in basketball and slipped into the abyss
of his career when he tried to play baseball. In 1994, Michael Jordan transferred himself from basketball to baseball,
signing a minor league baseball contract with Chicago White Sox after being a NBA player for 2 decades. Although he
trained himself very hard in his new career field by starting training two hours ahead of his team members, his
performance in baseball competition was almost a failure. Only after one year, Jordan announced his return to NBA and
quickly proved his power by winning regular seasons and All-Star Game MVP awards. The experience demonstrated
Michael Jordan was born for basketball. It is crystal clear that those who are aware of what they excel in and continue to
improve themselves in that specific area are more likely to attain success.
Whats more, placing too much value in taking risks is too aggressive as one might fall down on a less safe path. People
know every detail of what they should do in a familiar area, but they are prone to make mistakes in a new situation. Shi
Yuzhu, a famous businessman in China, tasted the bitterness of taking risks in investment in real estate. After succeeding in
his familiar industry, healthcare product research and selling, he decided to expand his business by constructing a 70-store
edifice, which his fund could not afford. Aggressive Mr. Shi decided to take a gamble on the building by using the profit
from his healthcare products. This action gambled away all his money, letting an unfinished edifice standing there. Later he
warned other businesspeople that it was wiser to stay in ones familiar area because profession brought victory, and
touching strange industry was never a good idea.
All in all, although chicken soup always encourages people to be brave enough to embrace new things and take risks, the
chicken soup saying only offers you an imaginary success. Truth success requires pragmatism. When individuals are coping
with matters they already know, they are able to figure out the best solution and achieve the greatest accomplishment.

Some people say that we should always follow our dreams even if the dreams are ambitious; others think we should focus
on achieving realistic goals. Which do you prefer? 2015/4/18

Level4 Level1-Level5
Level 4
successful peopletry new things and take risksdoing what they already
know how to do well
Level 4.

TPO 21
TPO21-

TOPIC

1. Genetically modified trees are hardier and more likely


to survive than unmodified trees do.

1. Genetically modified trees may be resistant to


specially-designed conditions, but that does not necessarily
ensure the survival of a tree population.

2. Genetically modified trees can bring farmers great

2. Genetically modified trees have hidden costs.

economic benefits.

3. Planting genetically modified trees can save wild trees


from overexploitation.

3. Genetically modified trees will bring damages to local


trees.

Sample Answer
The reading passage claims that planting genetically modified trees yields many benefits. The speaker in the listening,
however, disagrees with the author by stating that genetically modified trees bring serious problems and costs and then
refutes the three benefits mentioned in the reading one by one.
First, the speaker maintains that although some genetically modified trees are more resistant in given conditions, this does
not necessarily ensure the survival of that species. Natural trees are genetically diverse. Accordingly, when facing adverse
conditions like poor climates or insects, some individual natural trees can survive even if other trees of the same species
die. Genetically modified trees that are genetically uniform, on the other hand, may fail to survive when facing unexpected
conditions. Any climate changes or invasion of new pests can wipe out a genetically modified tree population. The
speakers statement, therefore, is a sharp contrast to the point made in the reading maintaining that genetically modified
trees are hardier.
Additionally, the speaker mentions that genetically modified trees bring hidden costs. For one thing, the genetically tree
providers will price their tree seeds much higher. For another, once the trees grow up, farmers cannot collect the tree
seeds for free and have to pay for the genetically modified trees provider every single time they plant by law. What the
speaker believes, undoubtedly, contradicts the point made in the reading maintaining genetically modified trees have great
investment returns.
Finally, the speaker indicates that genetically modified trees will bring more damage to native trees. Genetically modified
trees grow aggressively. Besides, they are planted among natural trees. Accordingly, genetically trees will compete for
resources like sunlight, soil, nutrients, and water, making it harder for natural trees to grow. The speakers statement
challenges the point made in the reading saying that genetically modified trees can save natural trees from
overexploitation.
305

Level 3 Level1-Level5
/
TPO21
TPO1TPO2
VIP 3

there are some serious problems and costs associated with planting genetically modified trees
the speaker believes that the reading is not convincing.

there are some serious problems and costs associated with


planting genetically modified trees costs causes

hidden cost hiddencosts


causes
serious problems and costs
c costs

fair

first costs causes

costs causes
150-225 OG 199
150~225
275 good
/
believe, state, claim, maintain, illustrate, indicate, suggest

oppose, contradictrefute
however, in contrast, is a sharp contrast to
if they are exposed to some environmental challenges that they havent been designed for,
have not been designed unexpectedun-programed unpredictable

TPO21-
.

Do you agree or disagree with the following statement? For success in a future job, the ability to relate well to
people is more important than studying hard in school. Use specific reasons and examples to support your answer.

Agree 1
TS1: For doing a good future job in teaching, the ability to relate well to students is more important than studying
pedagogics hard in school.
TS2: For doing a good future job in sales promotion, the ability to relate well to customers is more important than studying
marketing skills hard in school.
TS3: For doing a good future job in interpreter, the ability to relate well to foreign guests is more important than studying
translating skills hard in school.

Agree 2
TS1: For fresh graduates to do successful future jobs, the ability to relate well to people is more important than studying
hard in school.
TS2: For job hopers to do successful future jobs, the ability to relate well to people is more important than studying hard in
school.
TS3: For housewives coming back for career and seeking successful future jobs, the ability to relate well to people is more
important than studying hard in school.
Ps
Disagree 1
TS1: For doing a good future job in teaching, the ability to relate well to students is less important than studying
pedagogics hard in school.
TS2: For doing a good future job in sales promotion, the ability to relate well to customers is less important than studying
marketing skills hard in school.
TS3: For doing a good future job in interpreter, the ability to relate well to foreign guests is less important than studying
translating skills hard in school.
Disagree 2
TS1: For fresh graduates to do successful future jobs, the ability to relate well to people is less important than studying
hard in school.
TS2: For job hopers to do successful future jobs, the ability to relate well to people is less important than studying hard in
school.
TS3: For housewives coming back for career and seeking successful future jobs, the ability to relate well to people is less
important than studying hard in school.
Ps:

Sample Answer
Many people have a stereotyped belief that for doing a successful future job, getting straight As in transcripts is far more
important than great social skills or interpersonal relationships. People holding this view are convinced that high energy,
extreme effort, perseverance, and persistence in study can guarantee high achievements. From my perspective, however,
relating well to people plays a more crucial part in deciding success in any future jobs than does studying hard in school.
For doing a good future job in teaching, the ability to relate well to students is more important than studying pedagogics
hard in school. Teachers always fail to teach students in the way they planned since students needs and situations vary
from one student to another. For this reason, getting along well with students and modifying their teaching approaches is
more important than studying teaching approaches hard in school. My elder sister Lucia, a third-year student majoring in
English, for instance, got stuck in her first job as an intern English teacher in a high school. Feeling disappointed, Lucia
determined to learn more about teaching approaches during her last year in college. Thereafter, during all her courses, she
learned hard about the skills for making step-by-step video tutorials about English pronunciation. Lucias painstaking effort
in pedagogics, however, did not work. Having found studying hard the doctrines did not make any difference, Lucia tried
plan Brelating well to her students during her internship. She talked to some student representatives trying to figure out
what those post-95 students really liked. The students then told her that playing English movies like Frozen and Hobbits
may arouse their interest in English studying. Afterwards, Lucia abandoned the doctrinaire teaching skills and used movies
to improve students spoken English. The ability to relate well to students rather than the so-called maxims of teaching
helped her classes appealing.
Aside from in the field of teaching, for doing a good future job in sales promotion, the ability to relate well to customers is
more important than studying marketing skills hard in school. Some salesmen are indeed good learners of marketing rules;
however, they do not sell their products well for not knowing customers expectations. If they relate well to customers and
therefore cater to the potential buyers needs, they will do a better job. My cousin taking a part-time selling job during the
summer break in a dairy company, for instance, failed to sell the dairy products at the beginning. He first registered for a
marketing course to learn some marketing tricks like buy one, get one. Three weeks arduous learning, however, did not
make sense when my cousin tried to apply marketing-related norms to selling yoghurt along with low fat pure milk. Having
realized the sneaky marketing tricks were useless, my cousin tried plan Brelating well to customers especially women
aged forty to fifty who were willing to talk to buzzers. The ensuing days witnessed how my cousin made friends with
consumers and got their real feedback on yoghurt and pure milk. The buyers told my cousin that people crave for some
new flavors for yoghurt. After getting closer to customers and knowing their real expectations, my cousin did a better job
in selling the dairy products.
Admittedly, academic performance is essential. The lack of communication with other people, however, plays a more
significant role in ensuring a good future job.
561

2015.01.24 SAT essay11 2014 SAT


SAT SAT CB ETS
SAT essay native speaker SAT
with the development

ofsome people believe while others believehigh energyextreme effort


perseverance

persistence SAT prompt

SAT

30 2
body 2 body
moviesHobbits
Frozenproducts
low fat pure milk

like food like Lays Chips, M&Mcelebrities


like Justin Bieber and Taylor Swift.
admittedly

for instance : My elder sister Lucia, a third-year student majoring in English, for instance, got stuck in her
first job as an English teacher in a high school.
in contrast : Three weeks learning, however, did not make sense when my cousin tried to apply this skill
to selling yoghurt along with low fat pure milk.
step-by-step:
vary from oneto another:
painstaking effort:
pedagogics:
does not make any difference: ()
figure out:
maxims:
norms:

2011.10.14
Agree or disagree, the ability of adapting to new environment is more important than excellent knowledge for job.
2014.06.29 2010.10.09
Agree or disagree, to remain happy and optimistic when you fail is more important than achieving success.
2010.10.9ML
2014.4.27
Agree or disagree, your job has more effect on your happiness than your social life does?
2013.01.27

Agree or disagree, your job has more effect on your happiness than your living environment does

SAT
(2005.12) Are leaders necessarily people who are most capable of leadership? prompt SAT
leadership relating well to peopleSAT prompt genuine achievement
studying hard in school
(Prompt: We like to think that if someone has the right stuff, he or she will naturally rise to the top. But it isnt true. In
that same way that acting talent does not guarantee stardom, the capacity for leadership doesnt guarantee that one will
run a corporation or a government. In fact, at least in our time, genuine achievement is not highly valued, and those who
are skilled at achieving greatness are not necessarily those who are ready to lead.)

Level 4 Level1-Level5

1.

2.

3.
4

TPO 21
TPO21-

TOPIC
-

1.
1.

2. 2.

3. 3.

Sample Answer
The lecture argues against the view in the passage that, genetically modified trees bring a number of benefits. It presents
some potential problems caused by genetic modification.
First, the lecture claims that genetically modified trees may only be resistant to one certain condition. Therefore they
cannot guarantee survival when threatening condition comes along. On the other hand, these trees are not genetically
diverse, which means when environment challenge they have not been designed arrives, they shall all die. This directly
contradicts the first statement made in the passage.
Second, the lecture points out that genetically modified trees do not provide many economic benefits, which is against
what the passage says. The fact is, genetically modified trees are more expensive than traditional trees. Whats more, by
law, famers cannot collect the seeds and plant the new tree for free. Instead, they have to pay every time they plant.
Finally, the lecture argues that, genetically modified trees can only do harm to the local trees and fail to satisfy the demand
for wood to the native trees, as the passage refers. For the reason that genetically modified trees are usually planted
among natural trees, their aggressive growth would outcompete native trees for resources, sunlight, soil, nutrients and

water. And eventually kill the native trees.

Level 3 Level1-Level5

TPO 21-
Do you agree or disagree with the following statement? For success in a future job, the ability to relate well to
people is more important than studying hard in school. Use specific reasons and examples to support your answer.

-
1
2

Sample Answer
No one is an isolated island. As an old saying goes. As social begins, no one in the society can achieve success without the
help of others. This drives us to attach much importance to cooperation and communication. In other words, isolation is
the strongest enemy in ones career. Then, regarding which one is more significant in paving ways for people to achieve
professional success, views among people differ widely. As for me, neither can respectively contributes to success. Thus,
building a good relationship with others is equally of great necessity as studying hard at school.
For one thing, it is widely acknowledged that knowledge is the large component in factors of professional achievement.
The essence of what we have learnt in school, no matter the knowledge itself or the skills accumulated in the process of
learning, can help us accomplish tasks in career with efficiency and effectiveness. That is why each student, starting from
age five, is required to attend school and study diverse subjects in many fields and no one is able to shun the most basic
task of studying. Numerous elites prove the significance of studying hard at school. Without accumulated knowledge,
Madam Curie would have hardly discovered radium. Surely, apart from knowledge, it is also her bravery in face of failure
and persistence that give rise to her eventual accomplishment. Nevertheless, those mind and spirit are not born with, but

are increasingly cultivated in ways of working hard.


For another, modern world values communication and teamwork, which is also the key to ones professional success. We
may never be real successful individuals unless we master the art of interaction and cooperation. With further division of
labor, toadys work provides more chances for us to work within a group, which means individual work is paid less
attention to. As a result, even if one throws his time into oceans of knowledge and turns himself to be a Wikipedia, he
still need communication skills to accomplish projects. During the course of a group task, each individual has to work
together to attain the common goal. Thus, if one knows nothing about collaboration or have no idea of relating well with
others, one may either refuse to accept others opinions or disturb others and make working process inefficient. Then the
result may be a disaster. Only by resolving conflicts and working harmoniously can the group tackle problems in the way
and make eventual success.
In a word, both studying hard in school and getting a good relationship with others are fundamental for us. Therefore, as
students, what we are supposed to do is to make full use of time building knowledge in every field and participating in
school activities to enhance our communication skills. Only by achieving the two at the same time may students have a
promising future career.

Do you agree or disagree with the statement? An effective leader should try to make others feel that they are part of the
decision making process.(2013.1.12)
Do you agree or disagree with the statement? Student activities do as much benefit as academic study.(2013.10.27)
Do you agree or disagree with the following statement? All school teachers should be required to take courses every five
years to update their knowledge.(2013.12.1)
Do you agree or disagree with the following statement? People who cannot accept criticism will not succeed working in
teams. Use specific examples to support your answer. (2014.6.21)
Do you agree or disagree with the following statement? The ability to maintain a small number of friends for a long time is
more important to happiness than the ability to make new friends easily.(2014.8.30)

Level 2 Level1-Level5

TPO 21
TPO 21-

TOPICgenetically modified (GM)trees()

1. GM trees are designed to be hardier than nature trees;


more likely to survive than their unmodified
counterparts

1. non modified trees population is genetically diverse, but


GM trees are genetically much more uniform. So if the
climate changes, the GM trees will likely to be completely
wiped out.

2. a number of economic benefits: grow faster, give


greater yields

2. hidden costs associated with GM trees: farmers pay more


for its seeds, pay every time they plant.

3. prevent overexploitation of wild trees, satisfy the


demand for wood and save the endangered native trees

3. cause even more damage to the local wild trees: grow


more aggressively than natural trees do, outcompete the
native trees for resources and eventually crowding out the
natural trees.

Sample Answer
While the reading passage lists three benefits of genetically modified trees, the professor holds a different opinion.
To begin with, the reading states that genetically modified trees are hardier and more pest-resistant than natural ones, as
is shown by the example of papaya. However, the professor disagrees by arguing that these qualities cannot necessarily
ensure trees survival, because unlike natural trees, genetically modified trees are largely uniform in genes. Consequently,
their lack of genetic diversity makes it hard for them to survive various adverse conditions such as pest or climate change.
Second, the professor questions the idea in the reading that genetically modified trees bring economic benefits as they
grow faster and yield more. She points out that there are hidden costs associated with genetically modified trees because
farmers will be charged more to buy seeds and they have to pay every time they plant genetically modified trees.
Last, according to the passage, genetically modified trees can prevent overexploitation of wild trees and save endangered
native trees. However, the professor refutes this idea by suggesting that genetically modified trees may do more damage
to local trees as they grow more aggressively and eventually outcompete and crowd out native trees. (199 words)

Level 4 Level1-Level5
hardier , uniform , be wiped out outcompete , endangered
hidden costs

TPO 21-

For success in a future job, the ability to relate well to people is more important than studying hard in school.

+
1. Analyzing the question
+

2. Defining Position

3. Brainstorming

4. Topic sentences ()
4-1 On one hand, social networking is a key factor when it comes to professional performance and advancement.
4-2 On the other hand, work expertise is the basic requirement for a persons work competence. Only by studying hard can
students equip themselves with the knowledge, abilities and skills needed in the workplace.
5. Keyboarding ()
5-1 Introduction
Introduction=General Statement+Thesis Statement

5-1-1
There are many factors determining ones professional success, like networking and job expertise.
5-1-2 +
While it is claimed that the ability to relate well to people is of greater importance than hardworking, I would argue that
both qualities are equally vital.
5-2 body paragraphs ()
5-2-1 1++ topic sentence 1+details
5-2-2 2++ topic sentence 2+details
2 3
the other side
5-3 Conclusion
+Reworded Thesis Statement+topic sentences

Sample Answer
There are many factors determining ones professional success, like networking and job expertise. While it is claimed that
the ability to relate well to people is of greater importance than hardworking, I would argue that both qualities are equally
vital.
On one hand, social networking is a key factor when it comes to professional performance and advancement, as teamwork
and cooperation are increasingly emphasized in todays workplace. Most projects or tasks, which are beyond the capability
of any single individual, require the efforts and wisdom of a whole group. When accomplishing these group projects, a
person will enjoy a tremendous advantage if he can clearly get his ideas and thoughts across. For example, when a
company plans to design a new product, the designer needs to get first-hand customer feedback from the marketing
department, latest technological advances or breakthroughs from the research and development, and budget plans from
the finance. In such cases, the ability to relate well ensures the smooth cooperation and high work efficiency. Besides,
employees with good networking abilities are more likely to be promoted to higher positions or be given a pay rise,
because their excellence is recognized and appreciated more easily through clear communication.
On the other hand, work expertise is the basic requirement for a persons work competence. Only by studying hard can
students equip themselves with the knowledge, abilities and skills needed in the workplace. Without a solid foundation in
the related field, one can hardly get a job interview, let alone achieve further professional success. Studying hard in school
offers students the opportunity to master a subject systematically under the guidance of experienced teaching staff.
Medical students, for example, generally study for five years at a college level and then do clinical practices and internships
in hospitals for at least two years before they can graduate. The rigid academic requirements and training are the
cornerstones of a qualified medical major student.
To sum up, both the ability to relate well to others and studying hard at school are essential in attaining future success in
the workplace. With the two abilities, one can excel at his job with efficiency and professionalism.

2012/06/15 The most important characteristic for a politician or a leader is good communication skills.

Level 5 Level1-Level5
relate well to people,

TPO 22
TPO22-

first, second(in addition, moreover, furthermore, etc), third(finally, last, etc)


but, yet, however, in fact, it is true, nonetheless, nevertheless, though, even though, etc.
because, therefore, thus, consequently, as, for, lead to, attribute to, contribute to, etc.
TPO22 cellulose , let me explain, I will explain, etc.
only, best, most, it isthat, etc.
tpo5-maize, tpo10-orcas

more than, less than, rather than, compare, contrast,


so, well, generally, etc.

TOPICEthanol VS Gasoline

1. Ethanol fuel may not solve the global warming mainly


caused by burning gasoline in that burning ethanol also
generates carbon dioxide into the atmosphere.

1. E Ethanol,E not GW -- first


The increased use of Ethanol will not add to global
warming. EburnedCO2 -- its true
Ethanol releases carbon dioxide into the atmosphere
when burned. E from plants, corn -- but
Ethanol is often made from plants such as corn. Plants
absorb CO2 -- let me explain Plants absorb
carbon dioxide from the air its nutrition through
photosynthesis. Growing plants counteract release CO2 -well The process of growing the plants
counteracts the release of carbon dioxide.

2. In making significant amount of ethanol, a large


number of plants will be consumed. Therefore, animals
will lose a substantial source of forage.

2. Production of E not reduce animal food -- second


Production of Ethanol does not necessarily
reduce the sources of food for animals. Produce E using
C--because Thats because we can produce
Ethanol using cellulose. C, from Plant cell walls, not eaten
by animal -- Cellulose is the
main component of plants cell walls, and most celluloses
are not eaten by animals. animal feed not reduce -- so
The amount of animal feed will not be
reduced.

3. Ethanol fuel will never be able to compete with


gasoline on price. The price of Ethanol is
approximately the same with gasoline with the
help of tax subsidies from government for
Ethanol.

3. E will compete with G on price--third


Ethanol will be able to compete with gasoline on price.
Subsidies make E cheaper--its true Its
true that government subsidies will make Ethanol cheaper
than it would normally be. subsidies not needed--but

This support of subsidies will not always be


needed. customers increase, production increase--Once
Once enough people start to buy Ethanol,
Ethanol producers will increase the production of Ethanol.
production increase, price drop-- generally
Generally, increased production of products leads to a drop
in its price.production 3 times, price drop 40%--
If Ethanol production could be three
times greater, the cost of producing a unit of Ethanol will
drop by forty percent.

Sample Answer(364words)
The reading passage and the lecture both involve the topic whether ethanol fuel is a good replacement for gasoline. The
reading argues that ethanol fuel fails to be a better alternative over gasoline, but the speaker holds the opposite and
illustrates three points to side with his idea.
First, the reading passage indicates that the ethanol fuel may not help to solve the global warming mainly brought about
by burning gasoline for the reason that burning ethanol also releases carbon dioxide into the atmosphere. Conversely, the
professor contends that by using ethanol fuel rather than gasoline, the amount of carbon dioxide will be reduced because
ethanol is made from plants like corn, which can remove carbon dioxide from the air as part of its nutrition via
photosynthesis. In a word, the professor does not go along with the idea made in the reading.
Second, the reading claims that a large number of plants will be consumed in making large amount of ethanol. Accordingly,
animals will lose a substantial source of feed. This point is completely argued against by the lecturer who asserts that
ethanol can be produced from part of plant, namely cellulose, which is not edible to animals. Therefore, it is available for
ethanol producers to make use of corn without utilizing foods for animals. Evidently, the second point of the reading
material is refuted by the speakers idea based on the details mentioned above.
Finally, the professor suggests that its true that subsidies from government make the price of Ethanol cheaper, but this
support will not be needed. The speaker figures out that once more customers start purchasing ethanol, the ethanol
producers will increase their production, making the ethanol product cheaper and easily affordable. To better illustrate his
stand, the professor cites a study anticipating that if the amount of ethanol product goes up to two times bigger than the
current level, the average cost of this alternative fuel will decrease 40% of the original price per unit of product. Obviously,
all the argument discussed above contradicts the point of the reading passage that the price of ethanol fuel will confront a
surge once the government ceases the help of tax subsidies.

Level3 Level1-Level5
() cellulose

TPO 22-


Do you agree or disagree with the following statement? Teachers should not make their social or political views known to
students in the classroom. Use specific reasons and examples to support your answer.

()

Thesis statement
Topic Sentence
N

TPO22 social or political social


political
Thesis Statement: Teachers should feel optimistic and unconstrained to tell students their viewpoints about social or
political events.
Topic sentences:
1. It is meaningful for teachers to show their subjective aspects upon social or political events to students.
2. Students should not be labeled as passive receptacles for acquiring knowledge.
3. Concession: Social or political events in class does trigger some negative effects.
4. Conclusion: Teachers have obligation to make their students run parallel with this changing world by sharing their
opinions or views upon social or political events out of their textbook-oriented teaching.

Sample Answer(516words)
Few would deny that the primary purpose of teacher is to impart knowledge to his or her students, and teachers always
need to wisely collect the knowledge or information they involve in class. A great number of people hold the opinion that
teachers should vigorously abide by rules and regulations and be confined to the contents in the textbooks while other
individuals suppose that presenting their own social or political opinions to their students is also termed as a necessity. As
for me, I am inclined to the fact that teachers should feel optimistic and unconstrained to tell their students what they land
thoughts upon the social or political events.
To begin with, it is true that teachers opinions about social or political events might be labeled, to some extent, subjective,
but this is the justification and significance of why they are supposed to show their students their own opinions.
Additionally, this is a process to enable their students to form critical capabilities. Through telling their students their views

on social or political events, students would learn how to figure out which viewpoint is moral and meaningful, which is a
very crucial character for their future. Take a political event as an example. If the government wants to build a factory in a
single place, but there are divergent views to this proposal. As a teacher, he or she should not only tell his or her own
opinion about this event, but also guide their students how to differentiate which opinion is more beneficial to both the
people and local authority.
In addition, students should not be regarded as passive receptacles for acquiring knowledge. Needless to say, students
need more than theoretical knowledge in textbooks, and teachers sharing with students their own views about social or
political events would be a precious chance to help students put theoretical knowledge into practice. If teachers just
encompass knowledge that is overwhelmingly limited to their textbooks, their students would fail to be cultivated
creativity and imagination, which is detrimental to expand their outlook and utilize what they have accumulated from their
books. The election of CEOs could vividly illustrate this point. It is a simple thing by telling students what an election is
about and what kinds of merits and demerits it embraces, but they would have a vague impression about the concept.
However, students are able to grasp it immediately if their teachers combine the concept with their own CEO election
process.
Admittedly, involving social or political events in class does trigger some negative effects, including some students poor
academic performance due to their indulgence in these events and probably some bad mood or emotion toward the
government or authority. However, I intend to conclude that teachers should not only instill the theoretical knowledge to
students but also share their opinions about real events stemmed from society. This is because all of us live in a practical
realm, and teachers have obligation to make their students run parallel with this changing world by sharing their opinions
or views upon social or political events out of their textbook-oriented teaching.

Do you agree or disagree with the following statement? People should know about events happening around the world
even if they have little influence on their daily lives. Use specific examples to support your answer. (2014/10/18)

Level4 Level1-Level5

TPO 23
TPO23-

TOPICThe decline of yellow cedar population

1. the cedar bark beetle is a main cause of the trees


decline

1.the barks and leaves of healthy yellow cedars contain a


chemical that is poisonous to the insects feeding on them.
Thus, the cedar bark beetle can never possibly attack trees
before they are sick or dead.

2. brown bears are blamed for the overall population


decline of the yellow cedar

2. There is decline both on the mainland of North America


and islands along the coast. However, there is no brown bear
on the islands at all.

3. freezing climate is responsible for the general decline


of the species

3. many more trees die at lower elevations, where its


warmer than that at higher elevations.

Sample Answer-1
While the text attributes yellow cedars decline to insect parasites, brown bears and freezing climate, the lecture proves
this statement to be inaccurate. According to the lecture, the proposed reasons in the text only accounts for the poor
health of some individual plants instead of the decline of the whole species throughout North America.
First, the lecture counters the argument in the text that the cedar bark beetle is a main cause of the trees decline. Actually,
the barks and leaves of healthy yellow cedars contain a chemical that is poisonous to the insects feeding on them. Thus,
the cedar bark beetle can never possibly attack trees before they are sick or dead.
Second, the hypothesis in the text that brown bears are blamed for the overall population decline of the yellow cedar is
reputed by the point that bears are only responsible for the accidental dying of some individual plants, as in the places
without bears, the trees are still declining. From this it can be seen that bears are by no means the cause of the overall
decline.
Third, different from the text, the lecture holds that the freezing climate is also not responsible for the general decline of
the species. Its proven by the fact that many more trees die at lower elevations, where its warmer than that at higher
elevations. The cold weather may only have made the tree roots more sensitive rather than cause their decline.

Sample Answer-2
In the lecture, the professor points out that the hypothesis made by the reading passage is not adequate to explain why
the population of Yellow Cedar is on the decline. He points out several weaknesses of the statements made by the reading
passage.
On one hand, the professor claims that insect parasite may not be the fundamental cause of the decline. In fact, he argues
that healthy yellow cedar can secrete chemicals which can kill the cedar bark beetles so that they are unlikely to suffer
from insect parasite. The trees that suffers insect parasite have already been sick or on the edge of death.
On the other hand, the professor asserts that brown bears cannot cause overall decline in yellow cedar population. There
is decline both on the mainland of North America and islands along the coast. However, there is no brown bear on the
islands at all.
Whats more, the professor doubts the hypothesis that the change of climate is to blame for the declination. He admits
that the climate has changed rapidly recently. And, if the climate change is the real cause, the number of trees died on the
higher elevation should be larger than that on a lower elevation. The fact is, however, on the opposite.

Level 2 Level1-Level5
TPO 23-
In todays world, it is more important to work quickly and risk making mistakes than to work slowly and
make sure that everything is correct.

10. the development of modern technology has made our life faster than ever before.
11. we are living in a society which develops rapidly

Sample Answer
Modern technological development has made our lives faster than ever before. Under this circumstance, people nowadays
tend to live with a faster pace, and people are now concern more about efficiency. As a result, doing things slowly and
making almost everything correctly seems to be sort of out of date. In a modern society, people should, I believe, work at a
higher speed and risk taking mistakes.
As Ive mentioned in the first paragraph, the development of modern technology has made our life faster than ever before.
With the assistance of modern techniques, we consequently should work faster than ever before. For instance, in the old
days people made their own clothes by hand. But now, machines in the factories have made sewing and weaving at a great
speed and of great quality. We do not have to waste any time doing cumbersome works because of modern technology.

Therefore, why should we insist to work at a slow pace instead of enjoying the convenience that is brought by technology?
Another important aspect is, now we are living in a society which develops rapidly. To meet the rapid change and
improvement, all of us need to improve our own efficiency. As we can see, almost every factory wants to increase its
productivity and almost all the employer wants employees to generate greater value within the time limitation. Nobody
wants to be a loser in the society, so the only way to keep pace with the whole society is to accelerate, which is, working
faster.
By then, some people may counters that people may face greater risks of making mistakes. Admittedly, it is highly possible
for us to make mistakes when working at a high speed. But in fact, we human beings always make mistakes, so even if we
work as slowly and carefully as possible, we still risk making errors. However, by taking the advantage of modern
technology and high self-efficiency, we can do our work more precisely than ever before.
All in all, in todays world, people should work at a higher speed. It is so important that everyone in a modern society
should be like that. It is a general tendency of our world.

2005.12.16 / 2006.06.16: Some people believe that success in life comes from taking risks or chances. Others believe that
success results from careful planning. What do you think?

Level 4 Level1-Level5

TPO 23
TPO23-

TOPIC
-

1. 1.
cedar bark beetle)

2. 2.

3. 3.

Sample Answer
The passage gives three hypotheses to explain the declining population of yellow cedar. But the lecture argues against
them all and gives several reasons to doubt the feasibility of the hypotheses.
The first hypothesis mentioned in the passage is the cedar bark beetle since the trees are attacked by the beetles. The
lecture, on the other hand, points out that healthy yellow cedars have leaves with powerful chemicals that are poisonous
to insects. So they are unlikely to suffer from insect damage. The trees that have suffered from insect parasite are the
already sick ones and they might die anyway.
The second hypothesis is that brown bears claw at cedars, making the yellow cedar weaken. The lecture considers this
explanation lack of evidence. The main reason is that yellow cedar population declines all across the northwestern coast of
North America on mainland and islands. But there are no bears on the islands. Therefore, bears are not responsible for the
decline.
And the final hypothesis attributes the dropped amount to frost damage of roots due to climate changes. However,
according to the lecture, although climate changes makes the cedar root sensitive, more trees are dying at lower
elevations where the climate is warm than cold higher elevations, which is against the usual situation. So the frost damage
hypothesis is not convincing.

Level 3 Level1-Level5
TPO 23-
Do you agree or disagree with the following statement? In todays worlds, it is more important to work quickly
and risk making mistakes than to work slowly and make sure everything is correct. Use specific reasons and examples to
support your answer.

-
1
2
3)

Sample Answer
No one can deny the decisive factor of time in work and study. As a primary concern, the concept of time is often raised
frequently by our employers, who stress the significance of working efficiency. Nevertheless, high speed, sometimes, may
be compensated by risks in making mistakes. Thus, while someone doubts whether efficiency is that important, for me, it
is a better choice to work as quickly as we can without ignoring accuracy.
Initially, no one can deny that efficiency plays an indispensable role in our work and study. Once a task is initiated, every
one is supposed to make the most of their time and energy to finish it quickly since a second saved is a second gained.
With tardiness, one may fail to accomplish the task one time and be criticized by the employers. Also, it is likely that ones
procrastination incur other members progress in a group work and thus exert an adverse consequence on the whole. In
order to finish work rapidly and even in ahead of time, it is of great necessity for us to draft feasible plans and schedules
and strictly follow them step by step. Moreover, one thing noticeable in efficient working is that we should not turn a blind
eye to accuracy. Despite the fact that quick work may risk in errors and mistakes, we can still spare no effort checking each
possible mistake in the process. So, what we need to do is to maintain patience and carefulness.
In addition, there are certain ways to help us to avoid mistakes if the tasks are so important that we are not allowed to
make any mistakes. In such case, attentiveness and care assist in reducing the possibility of mistakes if speed is required.
Most of the time, when we concentrate on one thing at a time, both efficiency and accuracy can be guaranteed. For
instance, I used to try two working ways. The first is to do two things simultaneously, say, writing a paper while listening to
my favorite music. However, while this had always been regarded by the majority as an efficient way, I realized that it made
my project full of mistakes and also greatly lessen the efficiency. Thus, I tried the second way- by just focusing on one thing
at a time. Eventually, the second proved to be more appropriate since it enhances the efficiency and reduces the errors.
Accordingly, immersing oneself in the task at hand can help avoid unnecessary mistakes and one can have a high sense of
fulfillment when the work is done beforehand.

Furthermore, even if one has to sacrifice a few mistakes for efficiency, one needs not to worry too much about taking the
risks. The valuable experience and benefits we gain in the process of making mistakes contribute a lot for our personal
development. Needless to say, one would never make progress unless he commits certain mistakes and knows his own
weaknesses. Only in this way would he have a better understanding of himself and strive to achieve a better well-being.
Examples of numerous scientists prove the worth of enduring mistakes and failures. Without trial and errors, Madam Curie
would never have discovered the radius, a significant chemical element; similarly, Thomas Edison would never have found
the materials used in light bulbs. These stories all tell us one thing- mistakes is a precursors to success.
In a word, maintaining a reasonable speed while reducing the chances of mistakes is strongly recommended. In order to
achieve both efficiency and free of errors, care, patience as well as attentiveness should be stressed.

1.
2.
3.

Do you agree or disagree with the statement? A leader should admit if they made a bad decision. (2013.3.17)
When you are assigned an important presentation for work or school, you prefer towork on it right away so that you c
an work on it a little bit every day, or wait until you have a good idea about the presentation? (2013.9.28)
Do you agree or disagree with the following statement? Because modern life is complex, it is essential for young
people to have the ability to plan and organize. (2014.10.26)

Level 4 Level1-Level5

TPO 24
TPO24-

TOPICThe actual animal tissues of T Rex.

1. the soft, flexible organic substance found in the


branching channels are remains of blood vessels.

1.they might just be traces of soft and moist organic


materials left by the bacteria colonies which once lived
inside the bones after the T. rex died.

2. the presence of spheres proves that there are remains


of red blood cells

2. identical reddish spheres are found in the fossils of other


primitive animals which do not have red blood vessels. they
might just be pieces of reddish mineral.

3. collagen is a convincing evidence of bone tissue.

3. Scientists have never found any collagen in animal


remains that are older than one hundred thousand years.
Finding it in an animal fossil that are seventy million years
old contradicts their conclusion of how long collagen can
last for. it is more likely that the collagen is from much more
recent resource, for example, from the skin of researchers
who are handling the bone.

Sample Answer-1
The lecture is skeptical to the thesis in the reading that the fossil of a T.rex contains the remains of actual tissues with the
following reasons.
First, the soft, flexible organic substance found in the branching channels are not necessarily remains of blood vessels.
According to the lecture, they might just be traces of soft and moist organic materials left by the bacteria colonies which
once lived inside the bones after the T. rex died.
Second, the presence of spheres cannot prove that there are remains of red blood cells, either. In fact, scientists have
found identical reddish spheres in the fossils of other primitive animals which do not have red blood vessels. Therefore,
the spheres in T. rex are not necessarily remains of blood cells. Actually, they might just be pieces of reddish mineral.
Third, collagen is neither a convincing evidence of bone tissue. Scientists have never found any collagen in animal remains
that are older than one hundred thousand years. Finding it in an animal fossil that are seventy million years old contradicts
their conclusion of how long collagen can last for. Actually, it is more likely that the collagen is from much more recent
resource, for example, from the skin of researchers who are handling the bone.
In a word, the lecture suspects the argument in the reading that there are remains of blood vessels in T. rex and overturns
the three evidences in the reading respectively.

Sample Answer-2
The reading passage points out three evidences that indicate the existence of actually tissue in dinosaur fossil. However,
the professor doubts the accuracy of these evidences. In fact, he offers some alternative explanations for the substances
found in the fossil.
First and foremost, he challenges the existence of blood vessels. Instead, he points out that bacteria may occupy the
hollows inside the bones. Therefore, it is highly possible that the soft substance in the branching channels of the bone is
the moist residue of bacteria, rather than blood vessels of the dinosaur.
Moreover, the professor casts doubt about the red substance in sphere, which according to the reading passage is the
remain of red blood cells. In fact, the professor claims that this red substance was also found in some other animals, which
lived in the same place but had no red blood vessels at all. The professor argues that the sphere can be pieces of red
minerals.
Additionally, the professor doubts the existence of collagen in the fossil. On the contrary to the reading passage, he points
out that the earliest collagen that has ever found is in fossils 100000 ago. It is unlikely for collagen to last for more than 70
million years. As a result, the professor suggests that the collagen may come from recent sources, such as the skin of
researchers who handed the fossil.

Level 4 Level1-Level5
TPO 24-
One of the best ways that parents can help their teenage children prepare for adult life is to encourage them to
take a part-time job.

12. Part-time jobs will help teenagers to learn communicative skills.


13. Working at part-time jobs can impart students knowledge and abilities that they cannot gain from school, which help
them adapt to the society quickly.
14. Working at part-time jobs can give students character education on some important values and principles like hard
work, independence, and responsibility..

Sample Answer-1
According to a recent statistic on the internet, the number of single-child families occupies 75 percent of Chinese families,
and undoubtedly have a tendency of increasing. It follows that parents or other adult relatives may likely pay more
attention to their children, for the reason that the energy of parents has been saved a lot compared to those who own
several children. They insist that teenage children should not take a part-time job, considering that they are too young to

adjust themselves to complicated society and taking a job may disturb their study which is most important in this stage of
life. However, others believe that encouraging teenage children to take a part-time job is one of the best ways that parents
help them to prepare for adult life. From my part of view, I completely agree with the latter idea.
To begin with, the significance of taking a part-time job to developing independent personality and sense of responsibility
can never be exaggerated. My friend, Tom, was a spoiled boy a few years ago. His parents did everything for him: cleaning
his room, making his bed, and rescuing him from troubles. After Tom graduate from high school, his parents encouraged
him to take a part-time job in summer holiday. It was so boring for him to stay at home all day, so he took the advice and
worked as a waiter in a restaurant. At first, he could not adapt himself to the work he is doing and always got scolded by
the boss. But he worked hard and learned quickly. After one month, he could deal with all the problems during work and
he changed greatly, being responsible for his action and doing his work independently without others demands, and these
are absolutely the prerequisites for adult life. What happened to Tom struck me tremendously and made me realize that
how important taking a part-time job is for teenagers.
In addition, taking a part-time job can offer teenagers an opportunity to make friends with different people outside school
and to widen their horizon. Teenage children spend most of their time studying and hardly have time to keep abreast of
whats going on in the world. To some extent, it is very bad to teenagers development if too much stress is laid on school
work, for they have to put the theory into practice when they grow up to be an adult. Consequently, if you want your
children to prepare for future career, please encourage them to take a part-time job, and there they can meet interesting
people and learn something new, in the belief that they may one day need just those people or things.
Finally, it is easier to find a satisfactory job for the college graduates with part-time job experience. It is known to everyone
that graduates face with a brutal competition in job marketing. Only ones who are more competent can won a great job
and a brilliant future. And a background with doing a part-time job in the past can give you a license of becoming one of
promising candidates for the job that numerous applicants are desiring.
In a word, there is no better way to help teenage children to get ready for adult life than encourage them to take a
part-time job. It has always been a mystery to me why some parents, who desperately want their children to be
responsible, independent and successful, obstinately maintain that teenagers should not take a part-time job.

Sample Answer-2
Before a teenager becomes a young adult and earn their own living, his parents must help them to prepare for the future
life so that they could become totally independent. There must be many ways to help these ambitious young people, but
in my point of view, I believe that part-time jobs should be the most appropriate way to help them. By taking part-time
jobs at a young age, the young generation can be involved in the society without suffering.
By taking part-time jobs, teenagers could appreciate hard working and cherish the money theyve made. Therefore, in the
future, they are more likely to become hard-working adults and know how to do finance management. Take myself as an
example. I used to be very lazy and usually wasted my pocket money on useless things. But one summer vacation, I took a
part-time job as a tutor. It was not an easy job because the kid I taught was really naughty and the weather is really hot,
but finally I got good pay. After that, I did not waste any money, and I knew the value of hard-working.
Additionally, by taking some part-time jobs, teenagers will learn some useful communicating techniques. This is very
essential since we all live in a society and we have to communicate with different kinds of people almost every day. One of
my friends took a part-time job as a waiter. He had to deal with a lot of different situations and have to cater to different
customers. Time passed, he became very good at communicating with different people, but in the past he was really shy

and unwilling to speak to strange people! The part-time job helped to lift him to a new height.
Moreover, since teenagers accumulate useful experiences from part-time jobs, they are more likely to gain success in their
future career. In addition, sometimes taking part-time jobs may offer teenagers good opportunities. My sister, when she
was a junior, took a part-time job in a big company. During that time, her hard work and intelligence impressed her boss.
Then her boss gave her and offer and after her graduation, she became one of the employees of the big company. Now she
makes a very good living and works happily in this company. Chances are invaluable and we need to find ways to get the
chances.
So, to conclude, though there may be other ways, I believe that taking part-time jobs will be the best way to prepare a
teenager to be an ambitious adult. Parents who plan to help their teenagers prepare for the future life could take
part-time jobs into consideration.

2009.10.24
2011.05.13

Level 3 Level1-Level5

TPO 25
TPO25-

TOPICA set of clay jars of about 2200 years old

1. no electrical conductors to which the batteries


would have been attached, were found.

1. the site was found by untrained local people. They were


not archaeologists, so they would have not realized the
importance of other materials found on the site. They
would have overlooked these materials as uninteresting and
thrown them away.

2. the function of the vessels was the same as that of


those found in the ruins of Seleucia, namely to hold
scrolls of sacred texts, since they looked similar

2. the vessels were originally designed to hold the scrolls,


but later inventors discovered that with iron rods and some
liquid, they could be used to generate electricity; that was
how the first battery was born.

3. the vessels would have been completely useless to


ancient people as they had no devices that replied on
electricity.

3. the battery could generate some mild shock and this also
interprets evidence of some invisible power that how people
convince others they had the magic power. Also it could be
used for healing. In modern society doctors would use
batteries to stimulate muscles and release pains. In ancient
times people could also do that.

Sample Answer-1
While the reading asserts that the vessels were not used as electrical batteries by ancient people, the lecture proves the
statement to be unconvincing. According to the lecture, the battery theory is actually correct. The three reasons are
elaborated as follows.
First, the lecture overturns the point in the reading that no electrical conductorsto which the batteries would have been
attached, were found. It should be remembered that the site was found by untrained local people. They were not
archaeologists, so they would have not realized the importance of other materials found on the site. They would have
overlooked these materials as uninteresting and thrown them away. In this way, the lecture rebuts the first argument in
the reading by proving that the possibly existing electrical conductors could have been discarded.
Second, the lecture dissolves the assumption in the reading that the function of the vessels was the same as that of those
found in the ruins of Seleucia, namely to hold scrolls of sacred texts, since they looked similar. According to the lecture, the
same look does not prove anything. It is possible that the vessels were originally designed to hold the scrolls, but later
inventors discovered that with iron rods and some liquid, they could be used to generate electricity; that was how the first
battery was born. In a word, the lecture disagrees with the counterpart in the reading by saying that to hold scrolls was
probably the original purpose, but later ancestors adapted them to fulfill the purpose of producing battery.

Third, the lecture proves that the vessels were not as useless as the reading had thought. On one hand, they could be used
to show some invisible power and convince others that someone has magical power, because the battery can produce a
mild shock and tingling sensation when people touched it. On the other hand, they could also be used in healing. Just as in
the modern medicines, maybe they were adopted to stimulate muscles and ease peoples pain. All in all, the battery would
certainly be very useful, which is contrary to the reading.

Sample Answer-2
In the reading material, the author states that the vessels found in Iraq in 1938 were not actually used as electric batteries
in ancient times. However, in the listening material, the professor refutes that the argument is unconvincing as it was used
as batteries.
First, according to the reading passage, the author suggests that if they were used as batteries, they would have been
attached to some electricity conductors. However, in the reading, the professor claims that we should remember that the
discovery was made by local people along with some other materials. As they were not trained as archaeologists, they
could not recognize the importance of some certain excavations. Perhaps they were overlook as something uninterested
and then thrown away.
Second, the author in the reading material mentions that the vessel were likely used for holding scrolls. Unfortunately, the
professor argues that it could not prove anything. It is possible that the vessels may be originally designed to scrolls.
However, ancient inventor then discovered that if the vessels were used with iron rod and some liquid, it could generate
the electricity. So the copper cylinders may be originally used for one purpose but adapted for another purpose.
Finally, the author of the reading passage asserts that the vessels would have been completely useless to ancient people as
they had no devices that replied on electricity. In the contrary in obviously contradicts with the listening passage in which
the professor contends that the battery could generate some mild shock and this also interprets evidence of some invisible
power that how people convince others they had the magic power. Also it could be used for healing. In modern society
doctors would use batteries to stimulate muscles and release pains. In ancient times people could also do that.
In conclusion, according to the listening material, the argument that the vessels could not be used as batteries is
unwarranted.

Level 4 Level1-Level5
TPO 25-
Young people nowadays do not give enough time to helping their communities.

15. more people get higher education and they have the sense to help others;

16. young people today are more extroverted and willing to help others;
17. advanced technology provides people with more approaches to help others.

Sample Answer
Since the end of last century, a lot of criticism of the young generation has diffused trough out the world. Till now those
young people have been yet labeled as selfish, apathetic and lazy. Particularly, someone point out that, the youth are
tepid in their participant in community activities. However, such a presumptuous judgment neglects the simultaneous
change of human society, and deliberately covers the contributions made by the young.
Decades ago, frequent exchanges within one community were significantly in need as to ensure the quality of peoples
daily life. The youth regarded as the most energetic group, therefore, should take more responsibilities to contribute to the
whole community. Whereas, time has changed, with the boom of service industry and the great development of artificial
intelligence, people became more autonomicwe are not like people before need much physical help from our neighbors.
For example, before, the old in one community might need young people to take care of their daily life. But recently, we
have more service agencies to look after the old. Staffs there are well educated and have rich professional knowledge
about how to take account of the aged. Moreover, people are liberated from tedious houseworkwe have cleaners to
sweep the floor and dishwasher to wash the dirty plates, and, those artificial machines are also easy-use for the old. In this
sense, people now are more independent than ever before, and need less help from the outside. Hence, it could be
understandable that the youth reduce their work time in the communities.
Although the development of technology and a finer division of labor free us from some bland work in a community,
young people attempt to contribute more to the community and do what they could do as they are. If we look around, we
could find those cherubic teenagers active in kindergarten, nursing home and orphanageplaying with kids,
accompanying the lonely old and bringing their own books and clothes to those orphans. For instance, several friends of
mine, even though they are quite busy with their academic study, they still keep going to a nursing home every weekend to
cook with those old, and have lunch together. Also, they go to a local kindergarten each month to play with those kids:
teach them drawing and recognizing the national flags. The youth are doing more than we expect, and they do care about
the others, especially those who are vulnerable. Therefore, it would be prejudicial and unfair if we yet claim that the young
people nowadays have not put in the time to their communities, and ignoring effort they made.
Society should be more lenient to the youth, at least, not harsh. On one hand, we should understand the change of the
world, and the potential influence such a change might bring to our life. We could not falsely conclude the development of
our human society into complain about the teenagers. On the other hand, we should notice and admit every attempt
made by the young peoplethey are doing their best.

2012.03.19 / 2013.01.26: Young people now are more likely to help others than those in the past.

Level 4 Level1-Level5

TPO 26
TPO26-

TOPIC

1. Zebra mussels invasion from Easter Europe to North


America is unstoppable because it can survive in ballast
water used to balance cargo.

1. Zebra mussels can be controlled if the ship can empty the


original ballast water into the ocean rather than American
waterways and refill with salt water which can kill them.

2. No predators in a new habitat contribute to zebra


mussels domination there.

2. Zebra mussels domination in a new habitat wont last


long because their predators will sooner or later appear. For
example, local European aquatic birds that can eat a lot of
mussels can notice them and start to eat them.

3. The plankton-eating mussels will compete for food,


causing decline of overall fish population in a new
habitat.

3. Although plankton-eating fish population may decrease,


the bottom-feeding fish populations may increase owing to
nutrients generated by zebra mussels.

Sample Answer
The passage claims that Zebra mussels invasion from Easter Europe to North America is unstoppable and threatening to
freshwater fish there, while the lecture argues that these problems can all be solved.
In the first place, the author asserts that zebra mussels can transport by attaching to the bottom of the ship or living in
ballast water. In contrast, the speaker believes that people in the past didnt have enough knowledge to control them. If
the ship can empty out the fresh ballast water and refill with ocean water while still out in the ocean, the mussels in ballast
water wont reach American waterways because the salt water will kill them.
Besides, the reading passage states zebra mussels can dominate their new habitat without natural predators, contradicting
what is stated in the listening that this domination is only in the beginning. For instance, once the local aquatic birds in
Europe notice a new food source, they will change their habits to exploit by switch from whatever they eat before to
eating zebra mussels. Since birds can eat a lot of them, zebra mussels arent likely to dominate all the time.
Finally, the writer points out that zebra mussel may cause a decease of overall fish population in new habitats, while the
professor claims that zebra mussels may only have negative effect on plankton-eating fish population because of their
competition for food. For other fish, like the bottom-feeding fish, the mussels can generate nutrients for them. So
population of fish that feed near the bottom of water may increase.

Level3 Level1-Level5
TPO 26-

Do you agree or disagree with the following statement?


It is better for children to choose jobs that are similar to their parents jobs than to choose jobs that are very different from
their parents jobs.
Use specific reasons and examples to support your answer.

Agree
1.
2.
3.

Disagree
1.

2.
3.

Sample Answer
In many countries, inheriting the tittle and even the career of parents can date back to centuries ago. The reason why this
tradition has been prevailing all the time is the conspicuous benefit it brings to the family, the offspring in particular. In this
case, though children are given numerous choices, taking jobs that are similar to their parents, in my opinion, is
advantageous for them to hone skills and make progress.
To begin with, ones parents can contribute a lot to his or her gaining of interest and proficiency. Generally speaking,
people are more likely to be inspired from surrounding environment among which parents are the most influential factor.
Since it is almost inevitable for children to get to know their parents working content, those curious kids may easily
develop great interest in participating in the work. Then parents can take advantage of their interest, the best teacher, to

guide kids to learn more systematically and profoundly. Moreover, some parents are even proficient enough to play the
role of teaching, let alone address actual problems for their children. Aunt Jean is such a skilled engineer who has not only
inspired her son Louis by showing him explicit building models but also taught him to design complicated blue print that
even his professor in college has failed to make. Later during Louis internship in his senior year, aunt Jean pulled my cousin
out of trouble by advising him where to buy qualified stuff and how much he should order when his manager appointed
him to purchase construction materials for the first time. His excellence finally brought him several decent job offers from
well-known enterprises. But for doing the same work as his mothers, Louise can hardly win at starting line, possessing
more expertise than his peers and gaining appreciation from employers.
Additionally, parents can offer their children valuable resources if they take similar jobs. Undoubtedly, decades of working
have already equipped those sophisticated adults with abundant resources, like social network and working opportunities,
both of which are indispensable for a novice at his preliminary development. And parents, for the sake of their
sweethearts, often commit themselves to seeking and excavating those valuable resources. My classmate Jeremys success
is a case in point. When I first took computer major courses two years ago, Jeremy, who was already an intermediate-level
programmer, was introduced to the administrator of our Campus Wide Information System thanks to his parents business
partnership with various elites in computer field. He usually spent the whole weekend working there and learning practical
skills from senior employees. Obviously, without specialized knowledge and reliable platform that his parents guaranteed,
Jeremy may have to struggle for a satisfying internship in his senior year like we did. Opportunities are for those who are
well prepared, and opportunities need to be created. Once children engage in occupations that are similar to their parents,
they are presented with chances they deserve due to their full preparation.
Admittedly, choosing a career different from ones parents may also have positive effects. For instance, children can be
motivated to achieve success equal to their parents in their own fields. However, such kinds of enterprise can also be
cultivated if the young aim to surpass their parents achievement in the same field.
In short, it is a superior choice for kids to undertake a vocation similar to their parents owing to both proficiency and
efficiency they can gain from their parents on the way to success.

Do you agree or disagree with the following statement?


Young people today are less dependent on their parents than in the past.

Level3 Level1-Level5

2014322

TPO 26
TPO26-

TPO26 Reading Part:


The zebra mussel, a freshwater shellfish native to Eastern Europe, has long been spreading out from its original
habitats and has now reached parts of North America. There are reasons to believe that this invasion cannot be stopped
and that it poses a serious threat to freshwater fish populations in all of North America.
First, the history of the zebra mussels spread suggests that the invasion might be unstoppable. It is a prime example
of an invasion made possible by human transportation. From the zebra mussels original habitats in Eastern Europe, ships
helped spread it out along new canals built to connect Europes waterways. The mussel can attach itself to a ship s bottom
or can survive in the watercalled "ballast water"that the ship needs to take on to properly balance its cargo. By the
early nineteenth century, the mussel had spread to the whole of Europe. It was later carried to the east coast of North
America in the ballast water of ships traveling from Europe. The way ships have spread the zebra mussel in the past
strongly suggests that the species will soon colonize all of North America.
Moreover, once zebra mussels are carried to a new habitat, they can dominate it. They are a hardy species that does
well under a variety of conditions, and they have a high rate of reproduction. Most important, however, zebra mussels
often have no predators in their new habitats, and species without natural predators are likely to dominate their habitats.
Finally, zebra mussels are likely to cause a decline in the overall fish population in habitats where they become
dominant. The mussels are plankton eaters, which mean that they compete for food with many freshwater fish species.
TPO26 Listening Part:
Professor
Contrary to what you just read, there are ways to control the zebra mussel's spread. What's more, it is not so clear
that the mussel is a serious threat to fish populations.
True, the spread of zebra mussels couldn't be controlled in the past, but that's because people didn't have enough
knowledge. In fact, there are effective ways to stop ships from carrying the mussels to new locations. Here's an example.
The way zebra mussels usually travel across the ocean is that a ship takes on some fresh "ballast water" in Europe and then
empties that water into American waterways when it arrives. Full of zebra mussels, but the ship can be required to empty
out the freshwater and refill with ocean water while still out in the ocean. Salt water will kill the mussels.
Second, it's true that zebra mussels often don't have predators in their new habitats, but that's only in the beginning.
What's been happening in Europe is that local aquatic birds sooner or later notice there's a new food source around and
change their habits to exploit it. They switch from whatever they were eating before to eating zebra mussels. And birds can
eat a lot of mussels. So zebra mussels aren't so likely to dominate their new habitats after all.
Finally, even in habitats where zebra mussels become dominant, is the overall fish population likely to decrease. It's
true that zebra mussels may have a negative impact on fish that eat plankton. But on other fish, they can have a positive
impact. For example, the mussels generate nutrients that are eaten by fish that feed near the bottom of the lake or river.
So bottom-feeding fish populations may increase, even if plankton-eating fish population decrease.
TPO26 Question:
Summarize the point made in the lecture being sure to explain how the cast doubt on specific points made in the
reading passage.

TOPIC

1.the invasion of zebra mussel is unstoppable

1.The invasion of zebra mussel is stoppable

2. zebra mussel dominant the new habitat

2.zebra mussel just dominant the new place at the


beginning but will face the predator in a short period of
time

3.zebra mussel cause the number of other fishes to


decline

3. Zebra mussel can even has positive effect to bottom


feeding fishes.

Sample Answer245
Both the reading material and the lecture talk about the invasion of zebra mussel. Reading material has pessimistic opinion
about the invasion. However, the professor from the lecture disagrees with the opinion given by the reading material.
First, the reading material indicates that the invasion of zebra mussel is unstoppable. Human activities help zebra mussel
transport from Europe to the rest of world. However, lecture says that the reason why we cannot stop this invasion is
because we have not enough related information. For example, European used to carry fresh water inside their boat. The
fresh water became a perfect habitat for zebra mussel. Lecture suggests that people can replace fresh water with ocean
water so that we can eradicate zebra mussel from sailing.
Second, the reading material states that zebra mussel dominants new places and leaves no space for the original spices.
The lecture, however, holds an opposite opinion. The lecture believes that the dominant situation will just last for a short
moment. At first, there are no predators in the new habitats. But birds in that place will find out that there is a new type of
food resource and change their habits to exploit it. The dominant situation will not last forever.
Third, lecture does not believe that zebra mussel only causes negative effect to all spices in the new environment. Lectures
suggest that zebra mussel is good for the bottom feeding fishes and actually creates positive effect for those animals there.

Level3 Level1-Level5
Level 3.
mussel

TPO26-

Do you agree or disagreeIt is better for children to choose the jobs similar to their parents.

300

Sample Answer425
Parents are two of the very important people in any childs life. They teach us how to live and love. We all have strong
connections to our parents. Their life experiences and lessons can help us avoid unnecessary mistakes. Thats why I believe
children should choose similar jobs to their parents in order to keep learning from them. Avoiding redundant problems,
getting adaptation to the new environment and creating common topics are the three reasons why I support this idea.
Now, let me begin with avoiding redundant problems.
When children choose jobs that their parents are familiar with, they can teach you lots of useful lessons and help you
avoid unnecessary mistakes. China now has a very competitive job market and employers are very picky at this situation. In
order to stand out in this competition, we need to not only finish our jobs outstandingly but also avoid any social or
professional faults. If our parents have the knowledge about what the most common mistake people ever make is in this
area or about what mistakes they made when they first came into this industry, we can have very useful heads up at the
beginning. There is a saying that details determine success. The time you save for exploration is good for studying or
building social networks.
Second, Even if your parents are not at the same company you plan to work with. They might know somebody works in
that company. Common knowledge tells us that it is extremely important to have a mentor at the beginning of the career.
A great mentor can shape your personality, help you understand what the culture is of this company and push you to the
next level. Parent would be a great channel to connect you and your mentor.
Third, when children enter a similar area about which parents have background knowledge, there will be common topics
between children and kids. Most of us, including me, just have few topics that can discuss with our parents. Even if we
want to communicate with our parents, it is hard to do so. Career development or industry situation would be great topics
that can help us better understand our parents. We can learn what they want, where their passion comes from and how to
communicate with them through talking and listening.
With these advantages, I strongly suggest children to choose a similar job that is similar to their parents. You will not only
face fewer challenges but also get to know your parents in a way you never experience before.


Do you agree or disagree with the following statement? It is easier for more people now than in the past to get an
education. 2014 3 15
Do you agree or disagree with the following statement: A leader should admit if they made a bad decision. Use specific
reasons and examples to support your answer. 2013 3 17

Level4 Level1-Level5
Level 4.

TPO 27
TPO27-

TOPIC

1. Melted glaciers owing to unusually warm weather


caused the currents disruption in Gulf Stream, leading to
cooling.

1. Little Ice Age affected not only Europe and North America
but also the southern hemisphere, like New Zealand and
South Africa, which couldnt be cooled by the disruption of
Gulf Stream.

2. Dust and sulfur sent by volcanic eruption blocked


sunlight and thus decreased global temperatures.

2. No reports of striking visual effect, like dramatically


colorful sunsets, and grey or brown snow, caused by large
amount of volcanic dust mean that the dust was not strong
enough to cool the climate.

3. Lower human population resulted in less agriculture


and more forest absorbing CO2, indirectly contributing to
climate cooling.

3. Forest were not there long enough to stop greenhouse


effect because, with human population growing quickly,
forest were soon cut down to clear fields for crops needed
to feed the growing population.

Sample Answer
Regarding to three possible causes of the Little Ice Age given by the author, the speaker rebuts that none of the ideas could
account for its occurrence.
Firstly, the writer thinks that the disrupting of ocean currents, especially the Gulf Stream, may be the cause, while the
professor argues that Gulf Stream would cause cooling only in Europe and North America, but the Little Ice Age also
affected the southern hemisphere, in places like New Zealand and South Africa. Since Gulf Stream theory cannot explain
the cooling of the southern area, it cannot explain the Little Ice Age.
Secondly, the listening material presents that volcanic eruptions were not strong enough to release the large amounts of
dust needed to lower global temperatures, which contradicts what is stated in the reading that volcanic eruption could be
a reason. According to the professor, large amounts of volcanic dust in the atmosphere would have produced striking
visual effect, like dramatically colorful sunsets and grey or brown snow. But there are almost no reports of anything like
that during the Little Ice Age, so the volcanoes theory is questionable.
Thirdly, the speaker claims that forests were not there long enough to stop greenhouse effect, refuting the writers
viewpoint that the decline of human population could have caused more forest and thus led to trees absorbing carbon
dioxide. He explains that human population grew back to previous levels fairly quickly, which means forest were soon cut
down to clear fields for crops to feed the growing population.

Level4 Level1-Level5

TPO27-

Do you agree or disagree with the following statement?


If people have the opportunity to get a secure job, they should take it right away rather than wait for a job that would be
more satisfying.
Use specific reasons and examples to support your answer.

offer offer
offer
offer dream job

dream job

Agree
1.

2.
3.
Disagree
1.
2.
3.

Sample Answer
Every senior year witnesses graduates-to-be submitting resumes, going for interviews, and competing in internships.
Usually, a fresh graduate can receive job offers from several companies that he or she has applied for. However, the more
satisfying one or the dream one often has, unfortunately, yet to come. Under this circumstance, though giving up ones
dream is to some extent cruel, accepting the offer that one has already wined, in my opinion, is more reasonable, allowing
for a stable income and a good learning chance.

In the first place, a secure job can provide stable income for people to support themselves. In contrast, if they choose to
await a more satisfying one, they are taking the risk of being unemployed for a long time. For fresh graduates or young
adults who are too proud to ask their parents or friends for help, unemployment means a lack of main source of income,
which may possibly exert negative effect on their basic living. Worse still, failing to gain financial independence is likely to
make people anxious or even unconfident about life or themselves. Accepting a secure offer immediately, on the contrary,
can help building ones confidence by providing both salaries and social status. Only in this way can one spare enough time
and energy to pursue more in either life or work. Take our internship in junior summer as an example. Most of our
classmates including me settled down to work for an ordinary local translation company soon after our final examination,
while two of the top students chose to wait for offer from the renowned China Daily. After waiting for two weeks, troubled
by the shortage of money, they started to question themselves about their competence and qualification and regret not
joining us. Thus, uncertain waiting, compared with working, is indeed an unwise thing.
Secondly, people can practice their skills by taking a job right away instead of wasting time on waiting. This is because a
secure job, though not that satisfying, does not mean an inferior one. In this case, people can take full advantage of this
vocation to hone their skills, thus preparing for a potential promotion in the future. Undoubtedly, practice is never a waste
of time since it always matters not only in the current work but also in the desirable one that we hope to get sooner or
later. Suppose a senior student who aims to be an English teacher has not heard any news from the most prestigious high
school in his hometown. Should he accede to a normal schools invitation right away? Obviously, when the dream work is
not guaranteed, he can at least practice his teaching ability and accumulate related experience by taking the current
position. After he has learnt enough there, the prestigious school may have already opened to him. So what people need
for a satisfying career is preparing not just waiting.
It is true that waiting for a satisfying occupation means following ones inner voice which, as we know, is a key stimulus for
ones persistence. Nevertheless, taking a secure job does not stifle an ambitious dream but pave way for it. Consequently,
choosing a secure profession right away, on balance, is relatively wise for the sake of both profit and preparation.

Do you agree or disagree with the following statement?


People should take several different kinds of job before they take a career in a long term. 2014/7/12

Level3 Level1-Level5

TPO 28
TPO28-

TOPIC

1. The National Geographic Society committee proved


Pearys arrival at the North Pole after their investigation
of his records and equipment.

1. The committees conclusions were not trustworthy


because it was composed of Pearys close friends and main
patrons who examine him carelessly within only two days.

2. The success of Tom Averys 37-day trip which was


similar to Pearys in equipment testified Pearys speed
and success.

2. Tom Averys carried less weight because his food was


dropped by airplane and favorable weather conditions also
made his trip different from Pearys.

3. The photographs taken by Peary support his claim


because shadows in the photos demonstrated the right
Suns position on the day he arrived.

3. Since Pearys photos were fuzzy, unfocused and faded, it


is not precise to use them to calculate the Suns exact
position.

Sample Answer
The passage presents three arguments to support Robert Pearys claim of reaching the North Pole, while the lecture argues
that these arguments are not convincing for a lack of evidence.
First, the author points out that the National Geographic Society committee can prove Pearys claim. In contrast, the
speaker believes that the committee was not completely objective since it was composed of Pearys close friends who had
contributed a lot of money to fund his trip. Moreover, their careless investigation only lasted two days.
Second, the reading passage asserts Tom Averys trip similar to Pearys can be a strong support, contradicting what is
stated in the listening that Tom Averys journey was different from Pearys in important ways. For example, Avery carried
less weight though he used similar sled because his food was dropped by airplane. Moreover, the weather condition of
Averys trip was much favorable than Pearys.
Third, the writer states that Pearys photographs taken at the North Pole can prove his arrival, while the professor claims
that the already faded and worn photographs do not prove anything. Since Pearys pictures were taken a hundred years
ago by a primitive camera, they are fuzzy and unfocused. So blurred shadows in these pictures are not precise enough for
scientists to calculate the Suns accurate position.

Level4 Level1-Level5
TPO28-

Do you agree or disagree with the following statement?


Parents today are more involved in their childrens education than were parents in the past. Use specific reasons and
examples to support your answer.

Agree
1.

2.
3.

Disagree
1.
2.
3.
4.

Sample Answer
With the whole society attaching increasing importance to the education of the young generation, the role that parents
and educators play is even fundamental to enhance educational quality. Though some people deny parents influence,
citing their absence in childrens education due to busy schedule, in my opinion, these responsible adults today, thanks to
their own education background and advanced teaching pattern, are actually more involved in their kids education than
before.

Firstly, parents today are educated enough to realize the significance of the youngsters education. It is generally
acknowledged that peoples education background is directly connected to their attitude towards the education of their
offspring. To be specific, the more they are educated the more they emphasize receiving education since poor-educated
people in the past, though think highly of money and status, often fail to understand education is often the key to success.
In this case, todays parents tend to spend a large amount of time on encouraging kids to learn, convincing them to learn,
or even pushing them to learn. For parents who are talented enough to do extra teaching, time and energy spent on
educating kids may even double. It is quite common in both China and America that one of the parents quits the job or
change to a easy one in order to better accompany kids and help them to learn. Those full-time moms or dads can exert
tremendous influence on kids learning at different stage of their growth. For young children, parents inspire them and
arouse their interest in knowing the unknown, laying a foundation for systematic teaching several years later. For school
age children, parents need to encourage them to work with teachers in school and help to cultivate their good learning
habit. For teenagers, parents will supervise their behavior to prevent troubles and distraction. All these devotions are
powerful evidence of their enormous involvement in childrens education.
Secondly, modern teaching patterns and technology enable parents to take part in childrens education. On the one hand,
parents have more access to know or track students learning progress. For instance, smart phones, like iPhone, allow
users to install various apps, many of which are used by Chinese students to learn English words. Parents can check the
record of these apps to see whether their children finish the daily task. Computer, another indispensable technological
device, is utilized by parents to learn educational methods and download valuable learning materials for kids to digest. On
the other hand, most schools, in contemporary society, often contact students parents for better supervision. Instead of
handling students independently, school authorities have already realized cooperation between educators and parents is
more effective. Consequently, parents have opportunities to talk to teachers, listen to school meetings, and even review
records of their kids performance. Thus, chances guaranteed by modern technology and teaching method promote
parents participating in kids education.
All in all, despite the fact that some parents today who are often occupied with never-ending working stuff can barely
spare time for their kids, they can still manage to get to know their kids education through other ways. After all, more
time is not definitely equal to more involvement, and vice versa. As long as they check their academic performance by
communication with kids themselves and their teachers, the problem can be addressed. Therefore, parents easy and
frequent participation in childrens education is undeniable.

Do you agree or disagree with the statement


Rather than help children with schoolwork, parents should let children work independently.2014/11/15

Level3 Level1-Level5

TPO 29
TPO29-

TOPIC

1. no plants during wintermigrating to find food

1. summer temperatures in the North Slope area were


warmerthe sun shines 24 hours a day incredibly good
growing conditions for plantsthere was a lot of nutritious
dead vegetation around in the winter.

2. many skeletons have been unearthed from the same 2. Animals lived in herds for many other reasons, such as
siteedmontosaurs lived in herds just like modern obtaining extra protection from predators. e.g. the Roosevelt
migratory animals doliving in herds helps animals elk
coordinate
3. physically capable: 45/km per hour

3. juvenile edmontosaur were not physically capable of


travelling in great distancesthe juveniles would not have
survived on their ownthe herd had to wait for the
juvenilesthe herds would be slowedfailed to reach their
destination

Sample Answer
Suggested response length is 150225 words;
however, there is no penalty for writing more as long as it is in response to the task presented.

[Version 1 - 278 words]


Both the reading passage and the lecture are discussing whether some dinosaurs migrated to more hospitable regions to
survive the winter. According to the article, the North Slope dinosaurs did migrate to those hospitable regions; however,
the lecturer asserts that they did not.
Firstly, the reading material suggests that since those dinosaurs depended on plants to survive and there were no plants
existing during the cold and dark North Slope winter, the dinosaurs had to migrate in search of food; however, the listening
material suggests that there was 24-hour sunlight in the North Slope which was favorable for the growth of the plants, and
dinosaurs could depend on the nutrition from the dead vegetation to live.
Secondly, the writer of the article asserts that the dinosaurs' skeletons suggest that just like caribou and buffalo, the
dinosaurs lived in the herd, which helped them coordinate their migration; however, the professor insists that the

dinosaurs might live in the herd to gain extra protection from their predators and benefit from other purposes. For
example, Roosevelt elks, the plant eaters in the Western United States, lived in herds for the purpose of obtaining
protection from predators.
Thirdly, the reading material claims that the edmonosaurs had the physical abilities to migrate long distances because they
could reach speeds up to 45 kilometers per hour, which far exceeded the required speed; however, the professor points
out that the younger dinosaurs were not strong enough to finish that distance and could not survive on their own if they
were left behind. So, adult dinosaurs needed to slow down and look after them. In this case, all the dinosaurs could not
reach the destination.

[Version 2 - 403 words]


The reading passage suggests that the edmontosaur survived the harsh winter by migrating to hospitable areas while the
listening material argues that the evidence provided by the passage is not convincing.
First, the author mentions that the diet of edmontosaurs lend strength to the migration hypothesis. This kind of animal
lived entirely on plants that would disappear during the extremely cold and the dark North slope winter and therefore
edmontosaurs had to migrate at least part of the year to warmer zones to search for food. The professor rejects this
hypothesis by mentioning that this animal didn't have to migrate. 100 million years ago, the summer temperature was
much warmer in Arctic regions than it is nowadays, especially at the peak of summer when the sun shined the North Slope
24 hours a day. This provided good conditions for plants to survive and a lot of vegetation were produced during
summertime. As a result, the edmontosaurs could enjoy the nutritious dead plants in wintertime without having to
migrate.
Second, the reading material suggests that the large amount of edmontosaurs skeletons unearthed from a particular site
indicates that this animal used to live in heard. And a lot of current migratory animals like caribou and buffalo also live in
herds which helps them to coordinate the migration. This means that edmontosaurs might also migrate during winter. The
listening section, however, rejects this opinion by mentioning that there are many other reasons for living in herds such as
the extra protection against predators. And these animals living in herds may remain in the same area all year round. The
Roosevelt elk, an animal that lives in the forest of western US, is a typical example because this animal lives in the herd,
but does not migrate at all.
Third, the writer holds his opinion that edmonosaurs had the physical capabilities of migrating long distances. Their fast
speed which could reach as much as 45 kilometers per hour allowed them to make a journey for several weeks in order to
get to a new place during the harsh arctic winters. The lecturer opposes this suggestion by arguing that some of the
juveniles were not capable of traveling at high speed. So they might slow down the herds so much that they could not
finish the long journey. It was also impossible for the adult edmonosaurs to leave those young ones because they would
not survive in such environment.

Level 3 Level1-Level5

TPO29-

Do you agree or disagree with the following statement?


To improve the quality of education, universities should spend more money on salaries for university professors.
Use specific reasons and examples to support your answer.

Locating Keywords
keyword

2. Brainstorming
2-1 case by case

+ 1+ 2+

2-2 sufficiency and necessity

+ 1+ 2+
2-3 assumption

+ 1+ 2+

3. Defining Position
3-1 case by case
yes agree no
disagree
3-2 sufficiency and necessity

agree
disagree
3-3 assumption

agree
disagree

4. Topic Sentence


4-1 case by case agree
4-1-1 To begin with, it is a higher salary that enables a university to attract excellent people to work as teachers.
4-1-2 Furthermore, hardly can a university stimulate its professors researching and teaching enthusiasm without offering
higher salaries.
4-2 sufficiency and necessity disagree
4-2-1 To begin with, raising the professors salaries will not necessarily improve a universitys teaching quality.
4-2-2 Furthermore, many other methods can be used to ameliorate the education quality.
4-3 assumption disagree
4-3-1 To begin with, university professors already have decent salaries, which are not the root causes of low quality
education.
4-3-2 Furthermore, many universities cannot afford higher salaries of professors.

5. Introduction
Introduction=General Statement+Thesis Statement
++
+
General Statement Thesis Statement Thesis Statement
The bulk of the people are
convinced that because I, however, believe that...
I strongly agree that, particularly in A; however, when it comes to B, I disagree.
Thesis Statement General Statement

5-1

Whether universities should spend more money on professors salary is a matter for debate for a long time. A number of
people believe that it is necessary to increase professors salary so that they can better focus on education and research;
however, the bulk of the people hold the opposite idea, for the higher salary may indirectly increase students economic
burden. In my opinion, increasing professors salaries is by all means beneficial insofar as it facilitates universities
development in the long run.
5-2
5-2-1
Is it effective for universities to increase their professors salaries if they wish to improve their education quality?
5-2-2 WhatWhichWhyHow
What is the most effective and reasonable method of improving a universitys education quality?
5-3
General Statement WHY/HOW/WHEN

The advanced education has been progressing by leaps and bounds over the past few decades, resulting in an increase in
the number of universities and colleges. Many educational institutions begin to find it increasingly difficult to recruit more

students. Under such a circumstance, improving a universitys education quality becomes so decisive insofar as it enhances
the universitys public recognition and social reputation.
5-4
5-4-1
General Statement keyword
keyword

Thesis Statement

University - an institution at the highest level of education where a student can enrich the mind, enlighten the mentality,
and hone the skills - has always been considered as a holy place in human history.

Therefore, the quality of university education


never fails to attract peoples attention.
5-4-2
keyword General Statement

As the Latin roots of the word suggeste, meaning out, duc, meaning lead, and ation, meaning be in a certain
conditioneducation brings people a sense of instruction, guidance and enlightenment. If teachers are not competent
enough to impart knowledge, a universitys reputation would be affected.
5-5
General Statement

As the speech(March 11, 1873) of the House of Commons mentioned, a university should be a place of light, of liberty, and
of learning.
General Statement Thesis
Statement
If the professor fails to inspire, to prod, to irritate, to create engaging environment that enables learning to take place that
cant happen simply from reading books or watching films or surfing the Web, his/her university will be reduced to a
collection of books, failing to supply oxygen to their students. Hence, improving the competence of the professor becomes
a demanding need.
Thesis Statement

6. Conclusion
6-1
In conclusion, I may reasonably advance the claim that; that; and that
the claim that=Reworded Thesis Statement
that=Reworded Topic Sentence1
and that=Reworded Topic Sentence2
, case by case agree
In conclusion, I may reasonably advance the claim that it is reasonable for universities to raise their professors salaries;

that an increasing number of talented people will apply for teaching posts in universities; and that professors teaching and
researching enthusiasm will be mustered.
6-2
In conclusion, (Reworded Thesis Statement)

7. Subtopic Sentence
7-1 Because, In such a case, , ......
WHY/HOW/WHEN

1To begin with, it is a higher salary that enables a university to attract excellent people to work as teachers.
higher salariessocial status and material comfortsno disturbance from earthly affairsconcentration on
worktalented people would choose to work as teachers
A well-paid salary can simply offer a desirable social status and more material comforts, people who receive
generous salaries are more likely to devote themselves to laborious and continuous work. In such a case, if a university
provides the faculties of the staff a more desirable income, more elites would apply for positions in the university.
7-2 Also, Most adj., though, is adj....
case by case

2Furthermore, hardly can a university stimulate its professors researching and teaching enthusiasm without
offering higher salaries.
stimulating researching and teaching enthusiasm
- affording costly tuition fees of refresher courses
- purchasing valuable but expensive reference books
- starting field trips to collect the indispensable data
An increase in professors salaries will readily enable them to afford those costly tuition fees of refresher
courses. They, without any hesitation, will also order some decisive reference books online or purchase some
thought-provoking collections in a bookstore. Most promotive, though, is that they can start field trips to collect some
indispensable data, which may serve as the cornerstone of creative innovation.

8. Exemplification
GRE personal experience

case by case agree


8-1 [Topic Sentence] To begin with, it is a higher salary that enables a university to attract excellent people to work as
teachers.
For instance, my cousin Robin Glen Miller often described to me about his economics teacher Terry Andrew Thompson. Mr.
Thompson was provided with a decent salary, which enabled him to rent a fantastic researching lab off campus, and
allowed him to hire three researching assistants. With these advantages, Mr. Thompson could efficiently conduct his
experiments without being disturbed. No sooner had he been working in the university for 3 years, he began to excel
others at the Game Theory, and could always churn out up-to-date researching achievements on annual basis, making it
possible for him to readily advance his career in socioeconomic analysis and gradually win an academic reputation for his
university. Upon his success, many college graduates, and even experienced professors and associate professors in other
universities, began to apply for positions in his university. Apparently, his university will become increasingly influential in

the foreseeable future.


8-2 [Topic Sentence] Furthermore, hardly can a university stimulate its professors researching and teaching enthusiasm
without offering higher salaries.
For example, Graham Currie, one of my fathers acquaintance, used to be an excellent young Computer Science teacher in
Peking University. In the first three months after his entering in the university, he spent all his spare time in the school
library and spared no effort in collecting and compiling relevant data: he wished to win a national fund and organize some
of his students to design a software for a local dam. However, Grahams monthly income, some 4500 RMB, failed to meet
his expenses - 2260 RMB for his mortgage loan, 2500 RMB for his food, 350 RMB for electricity and water charges. Under
such a circumstance, his mother suggested him to take a part-time job in a nearby educational institution, which promised
to pay him 6000 RMB per month for his weekend instruction. Graham followed his mothers advice, and, from then on, he
shifted his attention from academic researches to earthly affairs.

9. Rebuttal
+ 1+ 2+
2
Admittedly, a minority of people are convinced that raising the professors salaries will not necessarily lead to a desirable
increase in education quality; in contrast, introducing advanced teaching equipments into laboratories and constructing a
modern library containing sufficient reference books seem to be more effective. However, these people over-simplified the
function of professors. In other words, neither will college students properly control the sophisticated devices without
competent teachers guidance, nor can they accurately find out the most helpful books without consulting a passionate
and enthusiastic teacher.

Introductionraising professors salaries


Body
1. a higher salarymore competent professors
a higher salarybetter working conditionacademic achievementsattract elites to work in the school
2. a higher salarystimulating professors working enthusiasm
a higher salaryprofessors do not need to strive for their basic needssaving more time conducting
experimentsenthusiastic and passionateimproving the teaching quality
ConclusionReworded Thesis Statement

Sample Answer
[607 words]
As the speech(March 11, 1873) of the House of Commons mentioned, a university should be a place of light, of liberty, and
of learning. If the professor fails to inspire, to prod, to irritate, to create engaging environment that enables learning to

take place that cant happen simply from reading books or watching films or surfing the Web, his/her university will be
reduced to a collection of books, failing to supply oxygen to their students. Hence, improving the competence of the
professor becomes a demanding need. As far as I am concerned, raising the professors salaries is an effective way of
improving the quality of university education.
To begin with, it is a higher salary that enables a university to attract excellent people to work as teachers. A well-paid
salary can simply offer a desirable social status and more material comforts, people who receive generous salaries are
more likely to devote themselves to laborious and continuous work. In such a case, if a university provides the faculties of
the staff a more desirable income, more elites would apply for positions in the university. For instance, my cousin Robin
Glen Miller often described to me about his economics teacher Terry Andrew Thompson. Mr. Thompson was provided with
a decent salary, which enabled him to rent a fantastic researching laboratory off campus and allowed him to hire three
researching assistants. With these advantages, Mr. Thompson could efficiently conduct his experiments without being
disturbed. No sooner had he been working in the university for 3 years, he began to excel others at the Game Theory, and
could always churn out up-to-date researching achievements on annual basis, making it possible for him to readily advance
his career in socioeconomic analysis and gradually win an academic reputation for his university. Upon his success, many
college graduates, and even experienced professors and associate professors in other universities, began to apply for
positions in his university. Apparently, his university will become increasingly influential in the foreseeable future.
Furthermore, hardly can a university stimulate its professors researching and teaching enthusiasm without offering higher
salaries. The low salaries prevent professors from affording those costly tuition fees of refresher courses. Also, they cannot
order enlightening reference books online or purchase some thought-provoking collections in a bookstore. Most disastrous,
though, is that they will not be able to conduct field trips to collect some indispensable data, which may serve as the
cornerstone of creative innovation. For example, Graham Currie, one of my fathers acquaintance, used to be an excellent
young Computer Science teacher in Peking University. In the first three months after his entering in the university, he spent
all his spare time in the school library and spared no effort in collecting and compiling relevant data: he wished to win a
national fund and organize some of his students to design a software for a local dam. However, Grahams monthly income,
some 4,500 RMB, failed to meet his expenses - 2,260 RMB for his mortgage loan, 2,500 RMB for his food, 350 RMB for
electricity and water charges. Under such a circumstance, his mother suggested him to take a part-time job in a nearby
educational institution, which promised to pay him 6,000 RMB per month for his weekend instruction. Graham followed
his mothers advice, and, from then on, he shifted his attention from academic researches to earthly affairs.
In conclusion, I may reasonably advance the claim that it is reasonable for universities to raise their professors salaries;
that an increasing number of talented people will apply for teaching posts in universities; and that professors teaching and
researching enthusiasm will be mustered.

2010/11/13, 2011/12/10, 2012/5/12

1. apply for
2. sb. spare no effort in v.ing
3. churn out
4. sophisticated
5. decent

6. be reduced to ......

7. A excels B at sth. A B
8. compile
9. enlightening
10. thought-provoking
11. prod
12. engaging

Level 3 Level1-Level5

level 3

TPO 29
TPO29-

TOPIC

1.Edmontosaurus fed on plants. There is no plants


growing during the cold and dark .

1. The edmontosaurs not have to migrate to find food


long time ago the summer temperatures in the North Slope
area were warmer than they are today. good growing
conditionsa lot of nutritious dead vegetation around in
the winter.

2. Many edmontosaurus skeletons have been unearthed


from the same site which suggests that edmontosaurs
lived in herdcoordinatesupport
3. Edmontosaurus were physically capable of migrating
long distances.

2. Edmontosaurs lived in herds doesnt mean they migrated.


Animals lived in herds for many other reasons. Living in
herds can provide extra protection.
3. Juvenile edmontosaurs were not capable of travelling in
great distances reach warmer territories and would have
slowed the herds so much that the herd never would have
made to its destination.

Sample Answer
In the reading passage, the writer claims that the Edmontosaurus survived the winter by migrating south to more
hospitable regions. However, the speaker denies the idea made in the reading that the edmontosaurs migrated every
winter is not convincing.
First of all, the reading passage contends that edmontosaurus fed exclusively on plants and there is no plants growing
during the cold and dark North slope winter but the lecturer argues that the edmontosaurs did not have to migrate to find
food because long time age the summer temperatures in the North Slope area were warmer than they are today. So there
is no need to move to other warm places to find plants.
Second, the author states that many edmontosaurus skeletons have been unearthed from the same site which
suggests that edmontosaurs lived in herd. The speaker says, however, that edmontosaurs lived in herds doesnt mean they
migrated. Animals lived in herds for many other reasons. Living in herds can provide them extra protection. Therefore,
edmontosaurs might live in herds for the more protection for themselves.
Lastly, in the reading passage, the author believes that edmontosaurus were physically capable of migrating long
distances. On the other hand, the professor in the listening argues that juvenile edmontosaurs were not capable of
travelling in great distances to reach warmer territories and would have slowed the herds so much that the herd never
would have made to its destination.

The reading passage contends that but the lecturer argues that.

According to the professor in the lecture,.This directly contradicts what the reading passage indicates, because.

Level3 Level1-Level5

TPO1-

Do you agree or disagree with the following statement?


To improve the quality of education, universities should spend more money on salaries for university professors.

agree/disagree

agree/disagree

Sample

support

support

Sample Answer
Should universities spend more money on salaries for university professors? I agree with the speaker's broad statement
on the basis that when professors get more salaries they will have more power for their work. Nevertheless, considering
high salary is not the key point to attract outstanding professors.
I concede that it is obvious that a higher salary can strongly relieve the economic burden on professors, especially for
those young ones, thus they can concentrate more on enhancing their teaching skills. Think about this, if the salaries from
universities for the professors are not enough for their daily life or they can just live with it. They cannot focus on the
research or class and they will do something else to earn the money which will distract their attentions. Nowadays, to
survive in the high pressure city, more professors choose to take an extra job in other colleges or companies. What's more,
Higher real estate price, the increasing price of clothes, money for the social, all these become impetus for professors to
earn extra money out of salary itself. Therefore, the universities can adjust the income for professors directly and they can
pay more attention on instructing and academic researching.
However, even if the universities double the salaries for professors, those who do not focus on their research will still

not change for anything. These people just do not like their career and maybe they think the professors have high
reputation in the society. With the high reputation, they can do something else which can earn much more money. In this
case, they wont stay focus on their professor job and the professors who have the sense of responsibility always work
hard on their job. For example, a solider would think it is duty to obey the rule without any hesitation; a surgeon would
consider healing the patients as his accountability; a historian would feel it is his responsibility to discover that is behind
the historical description. The professor with great sense of responsibility will not change for the more money and they
will always love what they do.
Based on the claims above, we should see that universities spend more money on salaries for university professors is
in a flexible way since it is so hard to agree or disagree with the statement. Nature serves as a good example of how trial
and error can be used to make changes. Every now and then, the genetic mutations would occur error in the gene
reproduction. Most of the time, the genetic mutations have a deleterious effect on the species and they will drop out of
the gene pool. But occasionally, the genetic mutation would occur something beneficial for the species and it will be
passed on the future generation. So we should make different standard in different regions and we could not just let the
universities spend more money on salaries for university professors to stimulate their performance without thinking about
the developed condition in that region.

2013/4/20
Its more important for the government to spend money to build art museums and music performance centers than to
build recreational facilities (such as swimming pool and playgrounds)
2013/12/1
All school teachers should be required to take courses.
2014/4/12
Do you agree or disagree: the universities should spend more money in improving facilities (libraries, computer labs) than
hiring famous teachers
2014/12/28
It is important for government to provide money for things that are beautiful not just for things that are practical.

Level4 Level1-Level5

TPO 30
TPO30-

TOPIC

1. The ancient Greeks were not technologically advanced


enough to make the burning mirror.

1. Greek mathematicians could assemble small mirror


pieces into a burning mirror.

2. The burning mirror would take a long time for people


to set fire and therefore not a desirable weapon.

2. The burning mirror could make pitch used in Roman boat


quickly on fire, and it is an effective weapon.

3. The burning mirror were no better than existing


weaponflaming arrows.

3. Romans were familiar with flaming arrows and were


cautious about them; Romans were less familiar with
burning mirrors and tend to consider them like ordinary
mirrors. However, the ordinary-looking mirrors might set
fire.

Sample Answer
The reading passage believes that burning mirror is just a Grecian myth and ancient Greeks never devised such a weapon
and gives three evidences. The speaker in the listening, however, believes none of the three reasons in the reading is
convincing and refutes the claims one by one.
To begin with, the speaker claims that it was not necessary for Greeks to form a single sheet of copper to device such a
burning mirror. An experiment shows that small flat pieces of polished copper can be arranged into a parabolic shape and
therefore create a huge burning mirror. Besides, Greek mathematicians knew the properties of the parabola so well that
they could make small mirror pieces into the parabolic shape. The speakers statement, therefore, is a sharp contrast to the
point made in the reading saying that Greeks were not technically advanced to build such a burning mirror.
Additionally, the speaker further explains that the reading underestimates the power of burning mirror. It does take ten
minutes for a burning mirror to make a wood boat on fire. Roman boats, however, were not made just of wood. Other
materials were used as well. To be specific, to seal the spaces between wooden boards and make them resistant to water,
the ancient boat designers used pitch, a sticky substance that caught fire fast. Experimental results showed that a burning
mirror could make pitch on fire within seconds. Besides, once the pitch got burned, the fire would spread to the wood in a
moving ship. The burning mirror, according to the speaker, was very effective. And what the speaker says, undoubtedly,
contradicts the point made in the reading maintaining that using a burning mirror to set fire was time-consuming.
Finally, the speaker suggests that Roman soldiers were so familiar with flaming arrows that they would watch out and be
ready to put out the fires flaming arrows might cause. Burning mirror, on the other hand, was not easily detected. Romans
would view burning mirrors as ordinary mirrors. But the ordinary-looking mirror could magically set fire at some
unobserved place on the ship. The reading, in contrast, indicates the flaming arrows did a better job than burning mirror
did. The speaker refutes the author in this way.

374

Level 5 Level1-Level5
/
burning mirror

1.
2.
3.

burning mirror 10
pitch burning mirror pitch

4. 3
150-225 OG 199
150~225

275 good
/
believe, state, claim, maintain, illustrate, indicate, suggest

oppose, contradict
however, in contrast, is a sharp contrast to

TPO30-
.

Do you agree or disagree with the following statement? It is more enjoyable to have a job where you only
work three days a week for long hours than to have a job where you work five days a week for shorter hours. Use
specific reasons and examples to support your answer.

AB AB

AB AB A three-day work week


with longer work hours
longer work hours
5 8 40 40 3 3 13.3
8 9 13
11 B five-day work week

5
4 2
A
B AB AB

2010.10.31
Which kind of job do you prefer, a job with high salary that is easy to lose, or a job with less pay but is stable?
2011.04.23
Do you agree or disagree with the following statement? A job with more vacation time is better than a job with high salary
but less vacation time.
2012.07.28
Some jobs pay a high salary to employees but require them to live far away from their family and friends. Some jobs pay
low salary but allow employees to live closer to their families and friends. Which do you prefer
4 13

AB
AB should people take a job whereor
should people take a job where? which way/job do you prefer?AB
AB

AB AB
AB
AB ---which way do you prefer
A B

--which way do you prefer A B


A B people
help me Agree 1. Disagree 1

Agree 1 AB AB
TS1: choosing a three-day work week with longer work hours can better help fresh graduates do a good job.
TS2: choosing a three-day work week with longer work hours can better help job hoppers do a good job
TS3: choosing a three-day work week with longer work hours can better help housewives coming back to career do a good
job.
disagree five-day work week Agree
Disagree 1 AB AB
TS1: choosing a five-day work week with shorter work hours can better help fresh graduates do a good job.
TS2: choosing a five-day work week with shorter work hours can better help job hoppers do a good job
TS3: choosing a five-day work week with shorter work hours can better help housewives coming back to career do a good
job
Ps: firstsecond

AB
AB
A A A

Agree
2

Agree 2 AB , body body3


TS1: choosing a three-day work week with longer work hours can better help people enjoy their personal lives.
TS2: choosing a three-day work week with longer work hours can better people make extra money during the free time.
TS3: Admittedly, working extended hours may make people feel exhausted or stressed out. Besides, people working
for longer work hours may fail to finish tasks assigned efficiently. People, however, can solve these two problems
Disagree 2 AB body body3

TS1: choosing a five-day work week with shorter work hours can better help people ensure their work efficiency.
TS2: choosing a five-day work week with shorter work hours can better help people relax after work.
TS3:
Admittedly, choosing a five-day work week may make people endure five days job and have only two days off.
People, however, can find solutions to relieve the pressure caused by having longer workweeks.
Ps: firstsecond
AB

Sample Answer
Some individuals are convinced that working five days a week with shorter work hours make employees feel better. People
holding this view believe five-day work weeks can save people from costly errors during the daytime. From my perspective,
however, working three days a week with long work hours is far better than working five days a week with shorter work
hours.
First, choosing a three-day work week with longer work hours can better help people enjoy their personal lives. Employees
choosing five-day work week or the eight hour day typically fail to invest their free time in families or pursue private
interests. Employees choosing a three-day work week and having four days off, in contrast, can greatly improve their
personal lives. My cousin Murphy, for instance, values quality of life very much. His last job teaching mathematics in a
public high school, however, required him to work five days a week and eight hours per day. Each time during the weekend,
he still had to use roughly five hours preparing for power points used for showing kids how to add auxiliary lines. Besides,
he had to score students assignments in his spare time. The trivial things really occupied most of Murphys spare time and
make it impossible for him to enjoy his favorite movies. Fed up with the tedious life, he decided to work for an educational
institution providing specialized mathematic training for students two months ago. This new job, in contrast, only requires
him to work three days from eight a.m. to nine p.m. From then on, after arduous tutorials, Murphy utilizes the rest of the
week to enjoy hiking, movies, barbeques, and parties. Although preparing for lessons and scoring students assignments
still occupy his personal time, he does have more free time since he has four days off. Now, he can go to latest movies like
Fast and Furious, pay visits to museums, or join night-run clubs. In this way, a three-day work week helps people improve

the quality of live.


Aside from enjoying personal lives, choosing a three-day work week with longer work hours can better people make extra
money during the free time. Typically, employees choosing a five-day work week are less likely to make extra money when
they are having two days off. If they choose to work three days per week with longer work hours, they have more chances
to run a small business or take part-time jobs when they are having four days off. To be specific, female employees can
choose to be part-time buzzers in Walt-Mart or Seven-Elevens. Male employees, on the other hand, can choose to be Uber
drivers and make the full use of their cars during their free time. What is more, employees can also take other long-term or
more stable part-time jobs like being accountant for other companies or serving as coaches in fitness clubs.
Admittedly, working extended hours may make people feel exhausted or stressed out. Besides, people working for longer
work hours may fail to finish tasks assigned efficiently. People, however, can solve these two problems. The first problem,
tiredness or pressure caused by overwork, can be solved when people take a nap during the extended work hours,
especially during the noon break. When it comes to the second problem about low efficiency, people can use some
interesting cellphone applications like Tomato Clocks to help them categorize assignments into the most important ones
and the less important ones. With the two possible problems solved, working three days a week with longer work hours is
still a good choice for most employees.
595

2015.01.24 SAT essay11 11 2014 SAT

SAT SAT CB
ETS SAT essay native speaker
SAT
with the development ofsome people believe while others believe
good

part-time jobs
teaching job
showing how to add auxiliary lines

part-time jobsUber drivers


part-time buzzers in Walt-Mart
or Seven Elevens

like food like Lays Chips, M&Mcelebrities


like Justin Bieber and Taylor Swift.
admittedly

are convinced that


arduous
specialized
tutorials
extended
costly errors


AB AB
2010.10.31
Which kind of job do you prefer, a job with high salary that is easy to lose, or a job with less pay but is stable?
2011.04.23
Do you agree or disagree with the following statement? A job with more vacation time is better than a job with high salary
but less vacation time.
2012.07.28
Some jobs pay a high salary to employees but require them to live far away from their family and friends. Some jobs pay
low salary but allow employees to live closer to their families and friends. Which do you prefer
AB ---which way do you prefer
which way do you prefer

2013.06.08
Agree or disagreeit is more fun to see a movie in a cinema (theater) with other people than see a movie at home.
2013.09.28
When you are assigned an important presentation for work or school a month in advanceand you are required
to write a presentation. Which way do you prefer1. To start to work on it right awayso you can work on it a little bit
everyday2. To wait until the end of the month when you have a good idea about the presentation.
2013.10.20A
Which way do you prefer? To shop in big stores or to shop in small stores?
2013.11.02
Which kind of friend do you like? Intelligent friends or humorous friends?
2013.11.09B
Which way is better for busy parents? To play with their children or to study with their children?
2014.04.12
Agree or disagree, universities should spend more money on improving facilities (libraries, computer labs) than hiring
famous teachers.
2014.03.16
Agree or disagree, it is easier for people today to receive education than they were in the past?
2014.06.15 2011.6.25NA
Some people prefer to buy technological devices as soon as they are available to the public, while other people prefer to
wait. Which do you prefer?

2015.01.25
Agree or disagree, classmates and partner can communicate with each other face to face to finish the project more better
than by sending e-mail
2015.03.28
Agree or disagree, it is better to finish a project completely and then do another project than do two or more projects at
the same time.

Task2 (AB ---which way do you prefer


)

2014.01.18
Some teachers allow students to send questions about coursework or assignments by e-mail, while other teachers prefer
these students ask them in person. Which way do you think is better?
2014.05.11
When traveling, some people prefer to go to new places, others prefer to visit familiar places, which way do you prefer?
2014.05.17
When getting advice, some people prefer to go to their parents, others prefer to go to their friends. Which way do you
prefer?.

Level5 Level1-Level5
AB AB
task1&2

AB
AB
AB

AB

---

TPO TPO1 four-day workweek

TPO 31
TPO31-

TOPIC

1. The lines in Sinosauropteryx are postmortem rather


than evidence for functional structures like feathers.

1. Other animals fossils buried at the same site have not


shown such kind of decomposition; lines in Sinosauropteryx
are feathers instead of fibers caused by decomposition.

2. The lines may be the frills rather than remains of


feathers.

2. The beta-keratin in lines suggests that lines are feathers


instead of frills.

3. The location of lines in the Sinosauropteryx indicates


the lines are useless for flight and thermoregulation and
therefore not feathers

3. Feathers can be used for other functions than flight and


thermoregulation.

Sample Answer
The reading passage opposes the idea that the lines in the Sinosauropteryx fossil indicated a feathered dinosaur by citing
several evidences. The speaker in the listening, however, believes none of the three evidences in the reading is convincing.
Instead, the speaker suggests the lines represent feathers.
To begin with, the speaker claims that other animals fossils buried at the same site have not shown such kind of
decomposition; instead, their functional skin structures have been perfectly preserved in the volcanic ash. Now that skin
structures of other animals are preserved in good conditions, those lines in Sinosauropteryx are likely to be
perfectly-preserved feathers instead of fibers caused by decomposition. The speakers statement, therefore, is a sharp
contrast to the point made in the reading saying that those lines may be postmortem fibers derived from decomposition
rather than functional feathers of living dinosaurs.
Additionally, the speaker further explains that chemical difference can distinguish feathers from frills. Feathers contain a
protein called beta-keratin while frills do not. And a chemical analysis suggests that Sinosauropteryxs structures do
contain considerable beta-keratin. Accordingly, the lines are indication of feathers. What the speaker says, undoubtedly,
contradicts the point made in the reading maintaining that the lines may be the frills rather than remains of feathers.
Finally, the speaker suggests that feathers can perform other functions like display function other than flight or
thermoregulation. A bird peacock, for instance, has colorful feathers in its tail for mate attraction. A recent analysis
illustrates that Sinosauropteryxs structures are colorful; it is strong evidence that feathers are used for display. The reading,
in contrast, indicates the lines are not feathers by emphasizing the location of lines in the Sinosauropteryx is useless for
flight and thermoregulation. The speaker refutes the reading.
290

Level 4 Level1-Level5
Sinoauropteryx

Sinoauropeteryx

/
3

lines location

150-225 OG 199
150~225
275 good
/
believe, state, claim, maintain, illustrate, indicate, suggest

oppose, contradict
however, in contrast, is a sharp contrast to

TPO31-
.

Do you agree or disagree with the following statement? Because the world is changing so quickly, people now are
less happy or less satisfied with their lives than people were in the past. Use specific reasons and examples to support your
answer.

brainstorm

/
//

Agree 1
TS1: People today are less happy and satisfied with their lives in friendship than they were in the past.
TS2: People today are less happy and satisfied with their lives in communication.
TS3: People today are less happy and satisfied with their lives in education.

Agree 2
TS1: Students today are less happy and satisfied with their lives than they were in the past.
TS2: Working groups today are less happy and satisfied with their lives than they were in the past.
TS3: Retired people today are less happy and satisfied than they were in the past.
PS

Disagree 1
TS1: People today are happier and more satisfied with their lives in friendship than they were in the past.
TS2: People today are happier and more satisfied with their lives in communication.
TS3: People today are happier and more satisfied with their lives in education.
Disagree 2
TS1: Students today are happier and more satisfied with their lives than they were in the past.
TS2: Working groups today happier and more satisfied with their lives than they were in the past.
TS3: Retired people today are happier and more satisfied than they were in the past.
PS:

Sample Answer
In society characterized by Big Bang Disruption where ideas and products are exponentially replaced by their newer and
more advanced counterparts, people seem to be contented with rapid development along with the fast-paced life today.
People holding this view are convinced that the technologies designed to make our lives easier are necessarily make our
lives better. From my perspective, however, individuals today, far from being happy or satisfied, are less happy and less
satisfied with their lives than they were in the past.
People today are less happy and satisfied with their lives in friendship than they were in the past. Ten years ago when my
father decided to have a gathering with his friends, for instance, he would invite them to go fishing or hiking together.
During the fishing time, my father and his friends would talk about current affairs like increased price of petrol. Besides,
they would share the most exhilarating parts in their life like someones giving birth to a new baby girl during the hiking.
The chatting made each of them feel closer and happy. Ten years later when I hang out with my friends and sit in a
Starbucks, in contrast, we neither see in each others eyes nor talk to each other. Instead, we keep lowering hour heads
and updating our WeChat, Instagrams Chinese counterpart, to see how many likes we have got of our last posting of the
Latte. Rather than exchanging ideas on the new flavor of coffee face to face, we just post pictures with words like yummy
online. Indeed, technologies designed for making friends closer, unfortunately, have ruined the basic pattern of human
interaction and made us annoyed and dissatisfied.
Aside from the field of friendship, in communication, people today are less happy and satisfied than they used to be either.
A couple years ago since the prevalence of internet especially search engines like Google and its Chinese counterpart Baidu,
I could visit the National Geographic website to see the updated videos about how Mongoose would react to a cobras
attack and how jaguar kills a crocodile in one bite. Additionally, as an ABC learner of making up, I could easily find more
than hundreds of YouTube videos teaching me how to make myself more like Kate Perry. I was quite delighted and satisfied
with social media and search engines. Years later, however, it is another story when I found I am overwhelmed in an
immense sea of unsubstantiated and misleading information. Every single time when I update my WeChat, Instagrams
Chinese version, I only find myself totally lost and confused by conflicting claims like genetically modified food is killing us
or embrace genetically modified food. In other cases, I feel befuddled when seeing an online article like Eight Hours
Sleep Makes People Smart and another article that maintains Six Hours Sleep is fairly enough. Today self-generated
real-time contents, either true or false, make me perplexed since I cannot distinguish the real one from the fake one or
even half-truths. Sadly, I am unhappy and dissatisfied with internet today.
Admittedly, the world is changing quickly and the advent of internet seems to make friends closer and bring convenience
when we want to know the world better. The changes expected to make our life for the better, however, have actually
made people less satisfied and happy because of technologys erosion of our friendship and communication.
564

2015.01.24 SAT essay11 11 2014 SAT

SAT SAT CB
ETS SAT essay native speaker
SAT
with the development ofsome people believe while others believe
good

half-truthunsubstantiated and misleading


information
SNS
YouTube
Kate Perry

SAT OG 4: Do changes that make our lives easier not necessarily


make them better? OG 8 : Is the world changing for the better? 2005 12 Do the benefits of scientific and
technological developments come at the cost of undesirable changes to peoples lives?
8

be characterized by: be labeled as, be marked as: In a


society characterized by Information Era
Big Bang Disruption:
www.forbes.com

exponentially:
fast-paced:
for instance : Ten years ago when my father decided to have a gathering with his friends, for instance, he
would invite them to go fishing or hiking together.
in contrast : Ten years later when I hang out with my friends and sit in a Starbucks, in contrast, we
neither see in each others eyes nor talk to each other.
counterpart: / Tang Xianzu, Shakespeares
Chinese counterpart Taobao, Amazons Chinese counterpartWeibo, Twitters Chinese counterpart
counterpart Task1 & 2 describe the city you live in describe two different singers and tell about
their differences

counterpart , TPO21 genetically modified trees are


more likely to survive than their unmodified counterparts.

hybrid: Task1&Task2
WeChat Moment, the Chinese hybrid of
Instagram and mobile Facebook
likes:
QQ Facebook
self-generated:
real-time:
be lost and confused:
be befuddled:
be perplexed:

2013.01.26
Agree or disagree, young people today are more likely to help others than they were in the past?
2013.03.03
Agree or disagree, young people today are more likely to give time and effort to improving the world than young people
were in the past?
2013.07.13
Agree or disagree, people today rely less on neighbors than people did in the past?

2013.07.14
Agree or disagree, people in the past are friendlier than they are now?
2013.12.15A
Agree or disagree, the food people eat today is healthier than it was in the past?
2013.12.15B
Agree or disagree, students today do not respect their teachers as much as they did in the past?
2014.03.02
Agree or disagree, the rules that the society expects young people to follow and obey today are too strict?
2014.03.16
Agree or disagree, it is easier for people today to receive education than they were in the past?
2014.03.22
Agree or disagree, young people today are less dependent on their parents than they were in the past.
2014.08.23 ( 2013.03.03 )
Agree or disagree, young people today are more likely to give time and effort to improving the world than young people
were in the past?
2014.09.27A 2013.01.26
Agree or disagree, young people today are more likely to help others than they were in the past?
2014.12.13
Agree or disagree, students today are more interested in politics than they were in the past?
2014.12.27 2013.01.26 2014.07.27A
Agree or disagree, young people today are more likely to help others than they were in the past?
2015.01.31 2014.03.02
Agree or disagree, the rules that the society expects young people to follow and obey are too strict?
2015.03.07B
Agree or disagree, people will spend less time on cooking food in twenty years than they do now?
2014.01.12

SAT

OG 4: Do changes that make our lives easier not necessarily make them better?
OG 8: Is the world changing for the better?
2005 12 Do the benefits of scientific and technological developments come at the cost of undesirable changes
to peoples lives?
Task2

2014.02.22
Agree or disagree, people in the future will read fewer than they are now
2014.12.21
Agree or disagree, young people today know more about the world than their parents were at the same age.
2014.03.22
Agree or disagree, university education is more important than it used to be?

Level 4 Level1-Level5

1.

2.

3.

TPO 31
TPO31-

TOPIC S

- S

- S

1. S 1.

S S

2. S 2.(frills)Frills

(frills),

Beta-Kareten S
S S
S

3. S 3.

S
S

Sample Answer
The reading passage mentions a fossil skeleton of a dinosaur called Sinosauropteryx and raises three explanations to
oppose the idea that Sinosauropteryx was a feathered dinosaur. However, the lecture, by citing several reasons, indicates
that the dinosaur has features.
First of all, the author in the reading points out that the fine lines may not represent functional structures of a living
dinosaur. Rather, the structures were formed after the animals death and their skin may have decomposed into fibers. On
the contrary, according to the lecture, other animals buried on the same site proves to have well-preserved skin structure.
Those animals were beautifully preserved in the volcanic ash and did not experience any decomposition. Thus, it is very
likely that the Sinosauropteryxs structure also did not decompose into fiber and the fine lines may represent features.
Second, as is mentioned In the reading, the fine lines are more likely to be frills rather than features. Nevertheless, the
lecture disapprove of the statement by claiming that frills and features contain certain chemical differences. While features
have a protein called beta-keratin, such a protein is absent in frills. Moreover, the chemical analysis of Sinosauropterux
proves that the dinosaur contains beta-keratin. Therefore, the structures were more likely to be features rather than frills.
Finally, while reading points out two primary functions of features- flight and thermoregulation and further dismisses the

likelihood of features in Sinosauropteryx. On the other hand, the professor indicates that features can also be used to
display in order to attract mate. He quotes peacock as an example. With colorful features, peacock always show such a
display function. Therefore, given that the structure of Sinosauropteryx is colorful- orange and while, the dinosaur is more
possible to have features.

Level 4 Level1-Level5
TPO 31-
Do you agree or disagree with the following statement? Because the world is changing so quickly, people now are
less happy or less satisfied with their lives than people were in the past. Use specific reasons and examples to support your
answer.

1
2
3

Sample Answer
21st century has witnessed an enormous advancement in science and technology. Accompanied with it are more
fashionable laptop computers, multi-functional cellphones and other electronic devices. These innovation, to some extent,
makes our life colorful. Nevertheless, convenient as it brings about, it makes our life more complicated and people
nowadays tend to own less happiness and satisfaction. Thus, I disagree with the statement above.
For one thing, up-to-date technological devices may enlarge the gap between people and make them less happy. In
contemporary society, cellphones stand out as one of the necessities in such an age of knowledge and technology. People
all around the world, especially young people aged 20 to 30, even cannot live without cellphones. In each occasion, say,
parties, travelings, classes and meetings, they are always staring and slicing on the smartphone screen. This all attributes
to the fabulous functions modern cellphones provide; apart from making phone calls and sending messages, some other
functions like linking to the Internet, looking for information, APP like WeChat and all sorts of games stand out. While it
renders the youth a wonderful world, it hinders their communication with the rest of the world. No one would like to talk
to friends face to face or meet in person, which might give rise to a damaged interpersonal relationship. While each one is
scrolled in his little world with no real social circle, they will soon be tired of the temporary enjoyment brought by
those electronic devices and feel unhappy at all. Thus, it is the advancement of technology that contributes to peoples
lack of felicity.
For another, social progress is accompanied with less understanding among individuals and mounting pressure. The
enlarged gap, including generation gap, may lead to less harmony among people and family members. Due to the

quickening pace of modern life, the fierce competition forces people to devote much more time and energy than before to
their work. As a result, the heavy workload may lessen the opportunity for parents to have a more frequent conversation
with their children despite various communication tools offered. Furthermore, since modern world attaches more
importance to productivity and efficiency, those who fail to accomplish tasks within limited time span may be crowded out
in competition and even become unemployed. Therefore, everyone, ranging from teenagers to white collars, strive to
study and work harder in order to survive from the fierce competition. All of these deprive of peoples leisure time and lay
a burden on everyone and every family.
Admittedly, there are some benefits involved in technological development and social competition. One one hand, thanks
to the advanced products, numerous troubles could be resolved easily. For example, when one gets lost in an unfamiliar
city, he can soon find the way out just by clicking on the button of Map on the cellphone or searching engines. Besides, it
is hard to imagine how manufactural business can increase productivity without the developed machines. On the other
hand, it is social competition that motivates people to get one step further to attain their goal. Once they survive in the
flood of contend, they may achieve a better well-being and have more sense of accomplishment. Nevertheless, all of the
advantages may be in exchange of peoples leisure time and family harmony and therefore may be overwhelmed by the
disadvantages discussed before.
To put everything in a nutshell, modern society offers more entertainment and convenience, along with huge pressure and
potential risks in relationship. Only if people take advantage of the benefits and try their best to overcome the drawbacks
can they lead a happier and more satisfactory life.

Do you agree or disagree with the statement? Young people today are more likely to help others than young people in the
past. Use specific examples to support your answer.(2013.1.26)
Do you agree or disagree with the statement? The place you choose to live has a greater effect on your overall happiness
than the job. (2013.1.27)
Do you agree or disagree with the statement? Young people today are more likely to give time and effort to improve the
world than young people were in the past.(2013.3.3)
Do you agree or disagree with the statement? People will feel happier if they have fewer possessions?(2013.6.30)
Do you agree or disagree with the statement? People are more friendly in the past than they are today.(2013.7.14)
Do you agree or disagree with the following statement? People who leave home for the city are more likely to be
successful and happier than people who remain in village. (2013.11.24)
Do you agree or disagree with the following statement? It is easier to maintain good health nowadays than it was in the
past. (2013.12.15-A)
Do you agree or disagree with the following statement? Your job has more effect on your happiness than your social life
does. Use specific examples to support your answer.(2014.4.27/2014.11.23)
Do you agree or disagree with the following statement? To remain happy and optimistic when you fail is more important
than achieving success. Use specific examples to support your answer.(2014.6.29)
Do you agree or disagree with the following statement? The ability to maintain a small number of friends for a long time is
more important to happiness than the ability to make new friends easily.(2014.8.30)

Level 3 Level1-Level5

TPO 32
TPO32-

TOPIC

1. The odd sounds that Russian submarine crew heard


may be caused by ocras or whales courtships.

1. Orcas live in the surface part of the ocean while


submarines dive into deep parts of ocean; the sounds
are not likely the sounds during courtships.

2. The odd sounds that Russian submarine crew heard


may be caused by giant squids.

2. The squids dwelled upon and will continue to dwell upon


the same area where submarines patrolled; however, the
squids appeared suddenly and disappeared two decades
later.

3. The strange sounds that Russian submarine crew


heard may be caused by submarines secretly used by
other countries.

3. Foreign submarines were not technologically advanced to


move or change direction very fast.

Sample Answer
The reading passage introduces three theories that could explain why Russian submarine crew heard strange sounds
during 1960s to 1980s. The speaker in the listening, however, believes none of the three reasons mentioned in the reading
is convincing. Instead, the speaker indicates that people still do not know what caused the odd sounds.
To begin with, the speaker points out orcas live near the surface of the water. The submarines, in contrast, dived deep in
the ocean. Accordingly, submarine crew were not likely to hear the whale sounds from somewhere near the ocean surface.
Plus, once oracs were nearby, they would have been detected by sonar. The speakers statement, therefore, is a sharp
contrast to the point made in the reading claiming that whales courtship caused the quackers.
Additionally, the speaker further explains that the squid hypothesis fails to take the timeline people detected odd sounds
into account. Submarines first heard quacker sound in the 1960s. The strange sounds, however, disappeared by the 1980s.
Now that squids lived in the areas where submarines patrolled and will continue to live there, it is not possible that sounds
suddenly began and then vanished after two decades for unknown reasons. What the speaker says, undoubtedly,
contradicts the point made in the reading maintaining that the sounds were caused by squids.
Finally, the speaker suggests that sounds moved around and changed direction very quickly. Submarines could not move or
change directions at the same pace. Besides, quacker sounds were different from engine noise of submarines. Today
people do not have advanced technologies to make submarines with high speed and silent engines. The reading, in
contrast, indicates that the sounds were secret submarines of other countries. The speaker, obviously, refutes the author
in this way.
290

Level 3 Level1-Level5

theories
speaker quacker sounds
timeline two decades later suddenly began
TPO21
technology

150-225 OG 199
150~225
275 good
/
believe, state, claim, maintain, illustrate, indicate, suggest

oppose, contradict
however, in contrast, is a sharp contrast to

TPO32-
.

Do you agree or disagree with the following statement? Young people today have no influence on the
important decisions that determine the future of society as a whole. Use specific reasons and examples to support
your answer.

young people today they


were in the past today previously
young peopleyoung people today


---
/

//

Disagree 1
TS1: High school students today have huge influence on the important decisions that determine the future of society as a
whole.
TS2: College school students today have huge influence on the important decisions that determine the future of society as
a whole.
TS3: Young working groups today have huge influence on the important decisions that determine the future of society as a
whole.
Disagree 2
TS1: Young people today have huge influence on important environmental-protection-related decisions that determine the
future of society as a whole.
TS2: Young people today have huge influence on important academic-related decisions that determine the future of
society as a whole.
TS3: Young people today have huge influence on important politics-related decisions that determine the future of society
as a whole.
PS TS firstsecond

Agree 1
TS1: High school students today have no influence on the important decisions that determine the future of society as a
whole.
TS2: College school students today have no influence on the important decisions that determine the future of society as a
whole.
TS3: Young working groups today have no influence on the important decisions that determine the future of society as a
whole.
Agree 2
TS1: Young people today have no influence on important environmental-protection-related decisions that determine the
future of society as a whole.
TS2: Young people today have no influence on important academic-related decisions that determine the future of society
as a whole.
TS3: Young people today have no influence on important politics-related decisions that determine the future of society as
a whole.

PS:

Sample Answer
In society characterized by Big Bang Disruption where products are exponentially replaced by their newer and more
advanced counterparts, more new problems arise and are needed to be solved. Some individuals believe that young
people today do not make any difference on the important decisions determining our future as a whole. People holding
this view are convinced that in science, politics, economy, or even environmental protection, young people seldom devote
their time and effort while their elder counterparts contribute more to the general good. From my perspective, however,
young people today do influence the important decisions that determine the future of a society as a whole.
First, high school students today have huge influence on important decisions that determine the future of society as a
whole. With the economic growth, air pollution, water pollution, and desertification are increasingly serious since some
governments strive for prosperity at the cost of deteriorating environment. The highly educated high school students today,
however, realize the long-term effects of unhealthy economic growth and endeavor to do some things to change the
future of environmental protection. Social networking sites, undoubtedly, are desirable channels for those teenagers to
arouse peoples awareness and change the status quo. Last October, for example, a leather products manufacturer
announced its decision to cooperate with Sichuan government and move its headquarter to Pixian County, Sichuan, via
Weibo, the Chinese hybrid of Twitter and Instagram. After seeing this Weibo, many high school students in Sichuan
spontaneously decided to start a petition to stop this action on major Chinese social networking sites like Weibo, QQ, and
Wechat. In addition to petition, students posted pictures, shared links, and uploaded videos warning people the possible
negative consequences of water pollution and air pollution. When citizens from different age groups saw the starling
pictures and articles online, they joined this action to guarantee their future health. Sichuan government, ultimately, gave
up this un-eco-friendly project under huge pressure. In this way, high school students as the major spokesmen for
environmental protection do greatly influence important decisions that determine the future of society as a whole.
Aside from high school students, college school students today also have huge influence on important decisions that
determine the future of society as a whole. As opposed to high school students who are typically required to prepare for
Gaokao, SATs Chinese counterpart, and therefore less politically engaged, college school students, in contrast, plays a
more significant role in shaping our future development. For one thing, college school students have more free time when
they are not having classes. For another, college students are much more psychologically mature and financially
independent. Accordingly, college students can stage demonstrations to pressure local government to stop social wrongs
more effectively. A famous Chinese dairy company, Sanlu, for example, used to add illegal materials in milk powder for
infants. When this scandal broke in 2008, some college students pressured the local government to shut down the Sanlu
factories although Sanlu contributed a lot to local economy. Three years later when a high-profile event Wenzhou high
speed rail crash happened, college students from across the country marched for more safety concerns. From then on,
Chinese people emphasize more on safety rather than on speed in transportation. In this way, college school students
today do shape the values and future of the society as a whole.
Admittedly, young people today are not independent psychologically and financially in general. These youngsters, however,
do a better job in making their voice heard in determining the well-being of the society as a whole than young people in
the past did.

596

2015.01.24 SAT essay11

11 2014 SAT

SAT SAT CB ETS SAT essay


native speaker SAT
with the development ofsome people believe while others
believe
good

30

social networking sites Weibo, WeChat, QQ once


last October

like food like Lays Chips, M&Mcelebrities


like Justin Bieber and Taylor Swift.
admittedly

be characterized by: be labeled as, be marked as: In a


society characterized by Information Era
Big Bang Disruption:
www.forbes.com

exponentially:
for instance : Last October, for example, a leather products manufacturer announced its decision to
cooperate with Sichuan government and move its headquarter to Pixian County, Sichuan, via Weibo, the Chinese hybrid of
Twitter and Instagram.
in contrast : As opposed to high school students who are typically required to prepare for Gaokao, SATs
Chinese counterpart, and therefore less politically engaged, college school students, in contrast, plays a more significant
role in shaping our future development.
counterpart: / Tang Xianzu, Shakespeares
Chinese counterpart Taobao, Amazons Chinese counterpartWeibo, Twitters Chinese counterpart
counterpart Task1 & 2 describe the city you live in describe two different singers and tell about
their differences

counterpart , TPO21 genetically modified trees are


more likely to survive than their unmodified counterparts.

hybrid: Task1&Task2
WeChat Moment, the Chinese hybrid of
Instagram and mobile Facebook
shape: shape our values

()

2013.01.26
Agree or disagree, young people today are more likely to help others than they were in the past?
2013.03.03
Agree or disagree, young people today are more likely to give time and effort to improving the world than young people
were in the past?
2013.07.13
Agree or disagree, people today rely less on neighbors than people did in the past?
2013.07.14
Agree or disagree, people in the past are friendlier than they are now?
2013.12.15A
Agree or disagree, the food people eat today is healthier than it was in the past?
2013.12.15B
Agree or disagree, students today do not respect their teachers as much as they did in the past?
2014.03.02
Agree or disagree, the rules that the society expects young people to follow and obey today are too strict?
2014.03.16
Agree or disagree, it is easier for people today to receive education than they were in the past?
2014.03.22
Agree or disagree, young people today are less dependent on their parents than they were in the past.
2014.08.23 ( 2013.03.03 )
Agree or disagree, young people today are more likely to give time and effort to improving the world than young people
were in the past?
2014.09.27A 2013.01.26
Agree or disagree, young people today are more likely to help others than they were in the past?
2014.12.13
Agree or disagree, students today are more interested in politics than they were in the past?
2014.12.27 2013.01.26 2014.07.27A
Agree or disagree, young people today are more likely to help others than they were in the past?
2015.01.31 2014.03.02
Agree or disagree, the rules that the society expects young people to follow and obey are too strict?
2015.03.07B
Agree or disagree, people will spend less time on cooking food in twenty years than they do now?

Task2 ()
2014.02.22
Agree or disagree, people in the future will read fewer than they are now
2014.12.21
Agree or disagree, young people today know more about the world than their parents were at the same age.
2014.03.22
Agree or disagree, university education is more important than it used to be?

Level 4 Level1-Level5

TPO 33
TPO33-

TOPIC

1. The carved stone balls may be used as weapons in


hunting or fighting.

1. People do not find signs of use or wear from the carved


stone balls and they are perfectly preserved; however, other
weapons back that time show signs of use and damage.

2. The carved stone balls may be used as a system for


measuring weight.

2. Carved stones balls were made of different types of


stones. Some stone balls made of different materials may be
same in size; however, their density varies from one to
another. Accordingly, the mass of stone balls may be
different and therefore not suitable for measuring.

3. The carved stone balls may be used for some social


purpose rather than pragmatic use.

3. For one thing, some stone balls were made intricate while
others were so simple that people can hardly find symbols
from the simple ones. For another, some rich people might
be buried with their possessions in tomb after death. But
the stone balls were not found in tombs.

Sample Answer
The reading passage introduces three theories about the purpose and meaning of craved stone balls found in Scotland.
The speaker in the listening, however, believes none of the three theories in the reading is convincing and refutes the
claims one by one.
To begin with, the speaker indicates that weapons for hunting or fighting may show signs of use or wear. Other weapons
back that time showed the signs of wear or damage. The stone balls, on the other hand, were perfectly preserved, and
they were neither broken nor cracked. Now that other weapons like arrows were damaged, the stone balls are not likely
to be weapons for hunting or fighting purposes. The speakers statement, therefore, is a sharp contrast to the point made
in the reading saying that carved stone balls were used as weapons for hunting and fighting.
Additionally, the speaker further explains that stone balls were made of different types of stones. The density of each
material varies from one to another. Some stone balls might be same in size; however, their mass could be different. A
handful of one stone balls might be heavier than a handful of another stone balls made of other material. The stone balls
were not desirable for measuring weight. What the speaker says, undoubtedly, contradicts the point made in the reading
maintaining that carved stone balls may be used for measuring weight.

Finally, the speaker believes that some stone balls were made intricate while others were so simple that people could
hardly find symbolic meaning from the simple ones. Besides, some rich people might be buried with their possessions in
tombs or graves after death; however, no stone balls were found in tombs. In this way, the speakers indicates the stone
balls were not for showing social status, opposing the point made in the reading suggesting that stone balls were used for
some social purpose rather than pragmatic use.
318

Level 3 Level1-Level5
carved stones

TPO
150-225 OG 199
150~225
275 good
/
believe, state, claim, maintain, illustrate, indicate, suggest

oppose, contradictoppose
however, in contrast, is a sharp contrast to

TPO33-
Do you agree or disagree with the following statement? When teachers assign projects on which students
must work together, the students learn much more efficiently than when they are asked to work alone on projects.
Use specific reasons and examples to support your answer.

A B AB

AB working togetherwork alone

more efficiently less efficiently

AB AB

which way do you prefer, to work alone on projects assigned, or work together with other
students? , AB . work together
TS1Working together with other students can better help students save time.
TS2Working together with other students can better help students make friends with other students.
TS3Working together with other students can better help people ensure the quality of projects.
work togetherwork alone efficiently efficiently
work together
AB
/ work together
work alone efficiently

Agree 1
TS1: Students work more efficiently when they must work together on academic projects than when they are asked to
work alone.
TS2: High school students work more efficiently when they must work together on out-of-school projects than when they
are asked to work alone.
Agree 2
TS1: Primary school students work more efficiently when they must work together on projects than they are asked to work
alone.
TS2: High school students work more efficiently when they must work together on projects than they are asked to work
alone.
TS3: College school students work more efficiently when they must work together on projects than they are asked to work
alone.
PS: first second
Disagree 1
TS1: Students work less efficiently when they must work together on academic projects than when they are asked to work
alone.
TS2: High school students work less efficiently when they must work together on out-of-school projects than when they
are asked to work alone.
Disagree 2
TS1: Primary school students work less efficiently when they must work together on projects than they are asked to work
alone.
TS2: High school students work less efficiently when they must work together on projects than they are asked to work
alone.
TS3: College school students work less efficiently when they must work together on projects than they are asked to work
alone.

Sample Answer
Many parents and education experts are convinced that children will work more efficiently when they work independently
on projects. People holding believe group work will take long hours to reach consensus; some children taking a free ride
will make the process more time-consuming. From my perspective, however, students work more efficiently when they
must work together on projects than when they are asked to work independently
First, students work more efficiently when they must work together on academic projects than when they are asked to
work alone. When required to do assignments, students often gain more insights in group discussions as brainstorming
help individuals view things from different points of view. Last October when I was doing SAT homework, for example, I
really hoped that I could finish the assigned essays within twenty five minutes. Unfortunately, it always took me long hours
to find appropriate celebrities examples to essay assignments. I could work on for an assignment Is Deception ever
justified? for an entire Saturday night and ended up finding nothing meaningful. Having realized it was hard for students
like me to finish the essay assignments in time, my English teacher divided us into six study groups and encouraged us to
exchange our ideas. The next Tuesday when group members and I brainstormed for the same assignment, my classmate
Morrison advised us to read a novel The Red Badge of Courage as the protagonist Henry was an ideal example for
deception. Another classmate Lucia advised us to read the stories about how Prussian ruler Fredrick the Great used a lie to
urge his people adopt potato-planting and save his people from starvation. After attending the group discussion, I soon got
inspired and finished my assignment. In this way, students work more efficiently when they work together on academic
projects than when they work independently.
Aside from academic projects, students work more efficiently when they must work together on out-of-school projects
than they are asked to work alone. Students today are always advised to do some social activities like civic engagement
activities. Working together on these social activities enables people save time as a group of people have wider range of
knowledge, expertise and skills. When my elder sister Silvia was a third year college student, for example, she was required
to do a research about peoples reactions to waste batteries recycling among different age groups or income levels. At the
very beginning, she was too shy to invite strangers to do questionnaires and wasted a lot of time. After she joined a group,
however, things went smoothly as male team members were willing to do the questionnaire job. Only within a week, she
finished her task by showing her teachers pies, charts, and tables illustrating how recycling behavior vary from one age
group to another In this way, students work more efficiently when they work together on out-of-school projects than they
are asked to work independently.
Admittedly, working alone on projects keep students away from distractions. Working together with others, however,
indeed make it easier for students to finish assignments in time.
512

2015.01.24 SAT essay11 11 2014 SAT


SAT SAT CB ETS
SAT essay native speaker
SAT
with the development ofsome people believe while others believe
good

exchange ideas insights

The Red Badge of Courage,


Prussian ruler Fredrick the Great.

like food like Lays Chips, M&Mcelebrities


like Justin Bieber and Taylor Swift.
admittedly 30

are convinced that


reach consensus
appropriate
end up
adopt
civic engagement activities
expertise
for instance
different age groups
vary from oneto another:

2013.09.14
Students who keep their rooms neat and organized will be more likely to succeed than students who do not.
2014.02.22
People who develop different skills are more successful than people who focus on only one skill.
2014.06.21
People who cannot accept criticism from others will not be successful at working in a group.

SAT
(2011.6) Do groups that encourage nonconformity and disagreement function better than those that discourage it?
(2010.11) Is a group of people more likely than an individual leader to bring about significant change?

Level 4 Level1-Level5

1. AB working togetherwork alone

more efficiently less efficiently

working together can help students


make friends
/ work together

2. work togetherwork alone

TPO 34
TPO34-
sea cow 3

TOPICsea cow

1.
sea cow

1. sea cow
sea cow 9 10

2. sea cow
kelp

2.
sea cow

kelp
sea cow

3.
sea cow

3.sea cow
sea cow

Sample Answer
The lecture explains why the three theories about the cause of sea cow extinction in the reading are not reasonable. The
reasons will be listed as follows.
First, according to the lecture, although the Siberian people did hunt sea cows, this is not likely to drive its population to
the point of extinction. This is because sea cows are massive creatures, about 9 meters long and 10 tons in weight. A
couple of sea cows could be enough for people in a Siberian village to eat for a month. Moreover, the Siberian population
was not so large, which means they would consume too many the animals. Therefore, human hunters may not be the
cause for the extinction. This is different from the point in the reading that it was human hunters who should be blamed
for the extinction of sea cows.
Second, the lecture states that the whole ecosystem may not change so much as the passage indicates. If the ecosystem
did experience so great changes, it would influence other parts of the system besides kelp, the main food for sea cows. For
example, the number of other marine animals, such as whales, should decline, too. But actually, no evidence was reported
about the decline of whales. Therefore, maybe no great ecosystem disturbance happened there and kelp might grow well,
and sea cows might not have food shortage. Again this changes the point in the reading that greater changes in ecosystem
could be the cause for the extinction.

Finally, contrary to the point in the reading that European fur traders could be the reason for the extinction, the professor
in the listening believes that sea cow population had already been small before the traders came. And there is evidence
showing that the sea cow population peaked hundreds of years earlier than the arriving of the Europeans, so there must
have been a reason for the extinction rather than the Europeans, although we do not know what it is.

Level2 Level1-Level5
TPO 34-
Do you agree or disagree with the following statement? Educating children is a more difficult task today than it
was in the past because they spend so much time on cell phone, online games, and social networking Web site. Use
specific reasons and examples to support your answer.

Sample Answer
It is commonly believed that technological advancement is the engine of society, and changes nearly every aspect of
peoples life, children education included. The widely use of the Internet, cell phones or other advanced devises like those
has facilitated children education nowadays, as it creates new methods for both students and teachers. The reasons will be
further illustrated as follows.
The advanced tools for children education make it easier for children to learn by themselves. The primary reason for this is
that the cell phones or social networking websites provide many useful tools that do not exist before to whip up students
interest in self-teaching. For instance, some programming engineers have developed an app on cells to help young
students to learn new English words, and in this app, words display themselves in a short cartoon movie and each word is
characterized by a person, introducing itself in an interesting way that attracts young learners. More specifically, the word
zoo is interpreted as the twin brother of the number 200, which is quite effective for children to lean. Moreover, in some
social networking websites, such as the Moments in WeChat, the most popular instant messenger in China, many articles
about ways of learning and some punchy words can be shared by teachers more often, which also are helpful for the
self-teaching for children. These two examples suggest that those tools do contribute more to the children education.

Besides benefiting the students, those tools also have a positive effect on teachers when they teach children in classroom.
In the past, most teachers did not have access to many tools to make their lectures intriguing for young students. For
example, the most-used teaching tools in classroom were chalks and blackboard and their voice. The limited ways of
teaching made it harder for young children to follow. However, things have drifted apart from what they used to be.
Nowadays plenty of modern tools greatly help the teachers to free their imagination in teaching. For instance, when
lecturing on the solar system, Mr. Tom, my teacher on astronomy, played a video for us to fully understand the orbit of the
sun and other planets. Furthermore, the social networking webs connect teachers and children well, and by using those,
teachers can assign their homework or other materials on the websites, and can get feedback soon from students.
It has to be admitted that cell phones or other things like those do distract students some time. For example, some online
games may waste students time in study. However, we do have some ways to prevent this. Teachers or parents could
guide them to use the Internet properly or supervise their children. In addition, some technological ways can be helpful,
such as restrictions to certain websites for entertainment.

2015/3/7
Do you agree or not? Why?
Taking children to field trips ( for example, to a museum )is a better way of education than learning at school in a
classroom?
2014/3/16
Do you agree or disagreeIt is easier to be well-educated today than it is in the past.
2013/12/28
Should children spend more time playing or studying, or they should be required to help the family with household chores,
like cooking and cleaning?
2013/11/9
(1) Parents learn more from children than children learn from parents.
2013/7/20
Parents now spend too much time on determining the futures of their children. Children should be allowed to make their
own choices.
2013/3/24
It is easier to raise children today than it was ten years ago.

Level3 Level1-Level5

TPO 35
TPO35-

1500 Middle Miocene 11

TOPIC

1. pollen grains 1. pollen grains


pollen grain

pollen grains

2. Middle Miocene

2.

3.Deuterium
deuterium deuterium

3. deuterium
deuterium
deuterium

deuterium

Sample Answer
The lecture explains why the three reasons about higher temperature in Antarctic area are not convincing. The claims are
listed as follows.
First, pollen grains found in trees in Antarctica may originate from other areas. Because pollen grains are light, they might
be blown by wind or flow in the ocean. Scientists have found that pollen grains may come from the warmer continents
nearby, such as South America or New Zealand. This is different from the point in the reading that pollen grains are the
evidence for more trees and more trees indicated higher temperature.
Second, more volcanic activities may not necessarily mean higher level of CO2, which could trap the warmth, in the
atmosphere. Recently, scientists have improved the method of measuring the CO2 and find that although there were more
volcanic activities, they did not add more CO2 into the atmosphere. The level of CO2 in the past was even lower than that
of today. This finding weakens the point in the reading that high level of CO2 contributed by volcanic activities can prove
that the temperature in Antarctica in the past was higher than it was now.
Finally, high level of deuterium may not necessarily indicate the more rainfall. This is because different types of plants have
different ability in absorbing and keeping the deuterium; some plants could absorb more and some less. Thus, the level of
deuterium cannot determine the rainfall. Maybe the rainfall in the past may be the same as that today, but the plants

found in Antarctica may be those with stronger ability in absorbing and keeping deuterium. This challenges the claim in the
reading that deuterium and rainfall are evidence for higher temperature in Antarctica in the past.

Level4 Level1-Level5
TPO 35-
Do you agree or disagree with the following statement? In twenty years there will be fewer cars in use than they
are today. Use specific reasons and examples to support your answer.

20
1

20
3

Sample Answer
The invention of automobiles has ended the time of horse and cart and embarked on the new era for peoples mobility.
With the help of cars, people could travel to farther places and move around easily. However, the booming of autos seems
to be ended due to environmental risks, energy crisis as well as serious traffic congestions. Therefore, I note that in 20
years there will be fewer cars.
The number of cars will shrink in the future because of the serious air pollution. Air pollution has become one of the
greatest headaches for human beings, as it has been reported that an increasing number of people living in cities suffer
from the lung cancer, which is closely related the wasted gases from vehicles. According to the statistics from the
municipal government of Beijing, the past decade has witnessed the jumping number of cars, about 20 million cars in the
city. Surprisingly enough, the number of victims of lung illnesses has also rocketed by 40 percent. In order to clean the air,
the government of Beijing has taken some measures to address the problem, such as limited the number of cars. In the
year of 2014, the government has reduced the number of cars by 10 percent. If this goes on, there would be fewer cars in
cities with the purpose of improving the air quality.
The intensifying energy crisis will limit the number of vehicles, too. Currently, fossil fuels, the non-renewable energies, still
take the main part of energy sources for vehicles, and one day, those will be depleted. Take China as an example, about 80
percent of cars are motivated by gas and other fossil fuels in 2014 and oil in China maybe only last for 10 years if there is

no new and big mines found. Although there exit some renewable energies, such as solar power, or wind power, most of
them are not as practical as the fossils, some of which are restricted to weather fluctuations. The effort to find reliable
energy sources has not been successful so far. In order to have a sustained development of cars, governments will have to
limit the number of cars in 20 years.
Another factor to cut the quantity of vehicles is traffic jams. It is not uncommon to find that traffic congestion has been so
severe that people have to spend extremely long time on the way of commuting. For example, last month I went a
business trip to Beijing. In the rush hours in the morning, it took me more than 2 hours and a half to get to my destination
and I had to spend another hour and a half back to my hotel. Worse, the residents in Beijing have to suffer the long and
long commuting journey nearly every day. If this happens, life in cities could be tougher than before. In order to reduce the
traffic congestion, city governments of Beijing have taken the policy of limiting the car numbers and development of public
transportation.
Based on the reasons discussed above, I believe that there will be fewer cars in the future.

2013/3/30
Even though many people from different countries care about environmental problems(pollution and global warming), the
environmental problem cannot be improved in the future.
2013/1/26
Do you agree or disagree with the following statement? Young people today are more likely to help others than young
people in the past.

Level2 Level1-Level5

You might also like